You are on page 1of 57

Difficulty लेवल एवं Chapter वाइज

ssc
सामा ान (जीके) - हदी

चै र वाइज
वी डयो लेसन
प रचय

ा आप एसएससी पदा भलाषी ह, जो इस उलझन म ह िक िकन का अ ास करना है और कह से अ ास करना है ? पर ा के


लए अ ास करने के लए अ थय को अ र सही े न बक का चयन करने म बहत
ु परे शानी का सामना करना पड़ता है।
इस लए, अ थ के तैयार क या ा को आसान बनाने के लए हमने ाटबुक लॉ क है , जो अगली पीढ़ का ाट े न बक है।
यह ाट े न बक सीखने का एक नया तर का है , जो के अ ास क पारं प रक शैली से थोड़ा अलग है।
इस ाटबुक म 1 लाख से अ धक के पूल से चुने गए सव े 4000 ह। का चयन और 3 अलग-अलग क ठनाई रम
इसका वभाजन, ेक पर उपयोगकत के यास और दशन डेटा पर आधा रत है।टे बुक क डेटा साइंस टीम ने ेक पर
छा क ति याओ ं से उ र देने क ग त, उ र देने म अ धकतम समय, ेक पर सटीकता, टॉपस और औसत छा के दशन
ु ं को नकाला और संसा धत िकया है।िफर उ
आ द जैसे डेटा बदंओ ने आपके लए एक अ त िकताब बनाई है।

ाट े न बक ल?

उ ीदवार इस ाट े न बक क सहायता से सम ा-समाधान से संबं धत मुख ववरण पर ान देना सीखगे।टे बुक ने


-समाधान के ि कोण म सुधार के लए ाट े न बक तैयार क है।इस ाटबुक क मु वशेषताएं इस कार ह :
इस ाटबुक म शा मल सव े 4000 को टे बुक के ऑनलाइन ेटफॉम पर पढ़ रहे छा के दशन डेटा के आधार
पर शॉट ल िकया गया है।
मशीन ल नग तकनीक का उपयोग क ठनाई र और के उ र देने के लए आदश समय (TTA) क गणना के लए िकया
गया है।

ेक पर तुलना क दशन डेटा का व ेषण करने के लए ाट उ र कुंजी दान क जाती है।

अ ास शु करने से पहले अवधारणाओ ं और िट को दहराने


ु के लए अ ाय-वार वीिडयो लेसन दए गये ह, ज QR
कोड को ै न करके ए ेस िकया जा सकता है।

हल करने के बाद ाट उ र कुंजी क ज च कर :

-वार क ठनाई र के साथ-साथ उन छा के तशत क भी ज च कर, ज ने इन को सही ढंग से एटे ट िकया है


या िफर को प करते हए
ु आगे बढ़ ह।
ेक अ ाय के अंत म सभी के व ृत और ापक समाधान देख।
इस ाटबुक का उपयोग कैसे कर?

वीिडयो लेसन SCAN QR


code for FREE video

अ थ पीडीएफ नोट् स के साथ वीिडयो पाठ तक भी पहंु च सकते ह और इस ाटबुक म ेक अ ाय के


lessons on this chapter

लए का अ ास कर सकते ह। यह ेक अ ाय क शु आत म दए गए QR कोड को ै न करके


िकया जा सकता है।

लेवल 1 : इस लेवल म बु नयादी मूलभूत अवधारणाओ ं के शा मल ह। छा को लेवल 1 से शु करना चा हए। इस लेवल म शा मल


सू के सीधे उपयोग पर आधा रत ह गे। इस कार के को ज ी से हल िकया जा सकता है और SSC CGL टीयर I, SSC MTS,
SSC GD क ब
े ल, SSC IMD (पाट I), SSC सले न पो (मैिटक र), आ द जैसी पर ाओ ं के लए सहायक ह गे।

लेवल 2 : इस लेवल म म म क ठनाई लेवल वाले शा मल ह। इन म वषय क अनु यु कृ त शा मल होगी। इस कार के


म वशेष ता उ ीदवार को SSC CHSL, SSC CPO, SSC सले न पो (इंटरमीिडएट लेवल ), आ द जैसी पर ाओ ं म मदद
करे गी।

लेवल 3 : ये अवधारणा और गणना दोन ही ि से क ठन ह। इस लेवल के SSC सले न पो ( ेजुएट लेवल) जैसी
पर ाओ ं के लए उपयोगी ह। जो मु ल और गणना वाले को हल करने के कौशल को बढ़ाने म मददगार होता है।

के लेवल को प रभािषत करने का यह ि कोण सव े चय नत 4000 के ज रए नए अ थ से लेकर अनुभवी अ थ तक के


सफर को सु न त करता है। के लेवल म मक वृ सभी कार के को हल करने क मता को सुगम बनाएगी। यह िकताब
को हल करने म लगने वाले समय क बचत भी करना सखाएगी।
SSC पर ाओ ं के लए वशेष के सामा ान तैयार सुझाव
एसएससी पर ाओ ं म सबसे अ धक अंक दान करने वाला अनुभाग सामा ान है। सामा ान अनुभाग म अ ा ोर करने के
लए उ ीदवार को नवीनतम करट अफेयस, त , ग त व धय आ द से अपडेट रहने क आव कता है :
अ स ट से न ऑिफसर, आयकर नर क आ द जैसे ति त पद पर भत एसएससी सीजीएल पर ाओ ं के ज रए क जाती है।
SSC CGL िटयर 1 म 50 अंक के लए सामा जाग कता अनुभाग है।

SSC CHSL पर ा डाक सहायक, लोअर िडवीजनल क, कोट क, सॉिटग अ स ट, डाटा एं टी ऑपरेटर आ द के पद के लए
उ ीदवार क भत के लए आयो जत क जाती है।य द आप SSC CHSL िटयर 1 के लए ल बना रहे ह तो आपको पता होना चा हए
िक सामा जाग कता अनुभाग 50 अंक का वेटेज रखता है।

एसएससी, SSC Selection पो , एसएससी जीडी क ब े ल, SSC CPO, आ द के लए उ ीदवार क भत के लए पर ा


आयो जत करता है। एसएससी जीडी क ब े ल पर ा के लए 25 अंक और SSC Selection पो के लए 50 अंक का सामा
जाग कता अनुभाग का वेटेज होता है।

जैसा िक अब आप जानते ह, एसएससी भत म सीट पाने के लए उ ीदवार के पास सामा ान पर एक मजबूत पकड़ होनी चा हए।
SSC ाट े न बक बुक सभी मह पूण चीज को याद रखने म आपक मदद करने के लए एक आदश गाइड है।

पर ा क ठनाई र : मैिटकुलेशन

SSC CGL टीयर I, SSC MTS, SSC GD क ब


े ल, SSC IMD (पाट I) और SSC Selection पो

SSC CGL टीयर I, SSC MTS, SSC GD क ब े ल, SSC IMD (पाट I) और SSC Selection पो (मैिटक र)जैसी सभी
पर ाओ ं म जीए से न होता है। पर ाओ ं म पूछे जाने वाले का र मैिटक या न 10व के र का होता है। पर ा के
क ठनाई र का अंदाजा लगाने के लए, उ ीदवार को NCERT क बु को पढ़ना चा हए। मैिटक र क एसएससी
पर ाओ ं को ै क करने के लए यह कुछ ए पट-रेकमडेड िट दए गए ह :

दै नक समाचार प पढ़, िक यह आपको द ु नया क वतमान घटनाओ ं से अवगत कराएगा।


भारतीय इ तहास और भूगोल पर ान द, िक उनसे अ र पूछे जाते ह।
दे श क स राजनी तक ह य और आंदोलन के बारे म जान।
ु ं को याद रखने के लए ह
उ ीदवार को मह पूण जीके बदंओ ल खत नोट् स बनाने चा हए।

पर ा क ठनाई र : इंटरमीिडएट

SSC CHSL, SSC CPO और SSC Selection पो

SSC CHSL, SSC CPO, और SSC चयन पद जैसी पर ाओ ं म क ठनाई का र अ धक होता है और उ ीदवार को भत
ि या म अपनी सीट को िफ करने के लए वा व म अ तैयार करने क आव कता होती है।

पर ा म भारतीय सं वधान और मह पूण संवैधा नक संशोधन पर पूछे जाते ह।


उ ीदवार को सभी मह पूण और हाल ही म पा रत बल को याद रखना चा हए।
उ ीदवार को भारतीय राजनी तक संरचना और मंि य के बारे म जानकार होनी चा हए।
उ ीदवार को मह पूण भारतीय संगठन और उनके अ के उ े के बारे म भी पता होना चा हए।
पर ा क ठनाई र : ेजुएशन

SSC Selection पो

SSC Selection पो पर ा के लए उप त होने वाले उ ीदवार को सामा जाग कता पर आधा रत का यास
करना होगा।नीचे हमने ान क त करने के लए कुछ मह पूण वषय का उ ेख िकया है :

उ ीदवार को संयु रा संगठन और उनके मु ालय के बारे म पता होना चा हए।


भारत और पड़ोसी दे श के साथ उसके संबंध के बारे म भी सवाल पूछे जाते ह।
उ ीदवार को मह पूण सरकार योजनाओ ं के बारे म भी पढ़ना चा हए।

सामा ान के लए ाट िट
उ ीदवार एसएससी पर ा के लए कुछ ए पट िट और िट पा सकते ह।

पर ा पैटन और पा म क ज च कर
उ ीदवार को नवीनतम एसएससी पा म और पर ा पैटन के अनुसार स ी से तैयार करनी चा हए।
उ ीदवार को अंकन योजना और नकारा क अंकन मानदं ड के बारे म भी पता होना चा हए।
उ ीदवार को उ तम अंक से जुड़े वषय पर अ धक ान देना चा हए।

करट अफेयस क व भ े णय पर ान द
करट अफेयस क मुख े णय पर मु ान द। रा ीय और अंतर ीय समाचार, खेल, व ान और तकनीक और कला
और सं ृ त, जैसे से न से अ धक को पूछे जाने क संभावना होती है इस लए इ सबसे पहले तैयार कर।
इन को हल करने के लए, उ ीदवार को िपछले 6 महीने के करट अफेयस के बारे म अ तरह से अपडेट होना

चा हए। वै क घटनाओ पर ान देने से आपको इस अनुभाग के को आसानी से हल करने म मदद मलेगी।
समय बंधन का यास कर
उ ीदवार को अपने समय बंधन कौशल म सुधार पर ान देना चा हए।
उ ीदवार को अ ास करते समय उनके ारा लए गए समय पर ान देना चा हए।

करट अफेयस के बारे म जाग क रह


र ज नग
ं और मा ा क यो ता (Quantitative Aptitude) के साथ-साथ सामा जाग कता पर भी ान देने क ज रत है।
अगर करट अफेयस को गंभीरता से लया जाए तो यह आपको ादा अंक ा करने म मददगार सा बत हो सकता है :

कम समय म अ धक अंक : चूंिक सामा जाग कता म कम गणना आधा रत होते ह, आप पूरे सामा जाग कता
अनुभाग को ज ी से हल कर सकते ह। व ान और तकनीक, खेल और करट अफेयस अनुभाग को ठ क से तैयार करने से
आपको कम से कम 20 अंक ा करने म मदद मल सकती है।

नकारा क अंक पाने क कम संभावना : चूंिक सामा जाग कता म हल करने के लए कुछ भी नह है , आपको हमेशा
पता चलेगा िक आपको उ र पता है या नह ।इसका मतलब है िक इस बात क कोई संभावना नह है िक आप जानबूझकर िकसी
उ र को गलत च त कर।
वषय-सूची

चै र सं ा. चै र लेवल पृ सं ा
1. ाचीन इ तहास लेवल I 1

लेवल II 4

लेवल III 7

ाट आंसर क 12

सॉ ुशन 14

2. म कालीन इ तहास लेवल I 41

लेवल II 44

लेवल III 47

ाट आंसर क 52

सॉ ुशन 54

3. आधु नक इ तहास लेवल I 84

लेवल II 87

लेवल III 90

ाट आंसर क 95

सॉ ुशन 97

4. भारतीय भूगोल लेवल I 123

लेवल II 127

लेवल III 132

ाट आंसर क 138

सॉ ुशन 140

5. व का भूगोल लेवल I 172

लेवल II 174

लेवल III 178

ाट आंसर क 183

सॉ ुशन 185
चै र सं ा. चै र लेवल पृ सं ा
6. राजनी त लेवल I 213

लेवल II 222

लेवल III 230

ाट आंसर क 239

सॉ ुशन 242

7. अथशा लेवल I 298

लेवल II 300

लेवल III 303

ाट आंसर क 310

सॉ ुशन 312

8. पय वरण और पा र तक लेवल I 336

लेवल II 338

लेवल III 341

ाट आंसर क 345

सॉ ुशन 347

9. भौ तक व ान लेवल I 368

लेवल II 372

लेवल III 376

ाट आंसर क 380

सॉ ुशन 382

10. रसायन शा लेवल I 409

लेवल II 412

लेवल III 415

ाट आंसर क 420

सॉ ुशन 422
चै र सं ा. चै र लेवल पृ सं ा
11. जीव व ान लेवल I 447

लेवल II 452

लेवल III 457

ाट आंसर क 464

सॉ ुशन 467

12. कं ूटर जाग कता लेवल I 507

लेवल II 510

लेवल III 513

ाट आंसर क 518

सॉ ुशन 520

13. Static GK लेवल I 547

लेवल II 553

लेवल III 559

ाट आंसर क 567

सॉ ुशन 570
CHAPTER:
1
FREE CHAPTER Scan
QR
code
for

ाचीन
इ
Free
Video
Lessons तहास
ाचीन
इ तहास on
this
Chapter.

LEVEL
1 1
-
86
Questions
TTA
:
14
Seconds
A)
समु गु B)
कठोर
1.
 न ल खत
म
से
कौन
गलत
 मलान
ह? C)
अशोक D)
पुलके शन
II
TTA
:
21
Seconds 14.
 कताब-उल- हद
पु क
क 
रचना
 कसने
क 
ह?
A)
अथव-वेद-जा ुई
सू 
का
 ान B)
ऋ ेद
-
 ु त
के
भजन 
का
 ान TTA
:
17
Seconds
C)
यजुर
-वेद
य ीय
सू 
का
 ान D)
साम
-वेद
-
 व ान
का
 ान A)
अल
मासुदी B)
अल-ब नी
2.
मोहन
जोदड़ो
 कस
नदी
के
तट
पर
 त
ह? C)
सुलेमान D)
 फरदौसी
TTA
:
9
Seconds 15.
रावी
नदी
का
वै दक
नाम
 ा
ह?
A)
रावी B)
 सधु TTA
:
10
Seconds
C)
घ र D)
सर ती A)
अ नी B)
शुतु
3.
 न ल खत
म
से
 कस
 ाचीन
क व
ने
अ भ ानशाकुंतलम
को
 लखा
ह? C)
प षणी D)
का लदी
TTA
:
8
Seconds 16.
इनम
से
कौन
भारतीय
उपमहा ीप
म
 ोटो-ऐ तहा सक
चरण
का
एकमा
A)
भासा B)
शू क उदाहरण
ह?
C)
आयभट D)
का लदास TTA
:
19
Seconds
4.
_______
 च क ा
और
श 
 च क ा
पर
एक
 ाचीन
सं ृ त
 ंथ
ह। A)
वै दक
स ता B)
 सधु
घाटी
क 
स ता
TTA
:
14
Seconds C)
गु 
काल D)
मगध
सा ा
A)
अथशा B)
रघुवंशम् 17.
 न ल खत
म
से
कौन
‘अ ा ायी’
पु क
का
लेखक
ह?
C)
सु ुत
सं हता D)
अ भ ानशाकुंतलम् TTA
:
11
Seconds
5.
" वपास"
___________
नदी
का
ऋ ै दक
नाम
ह। A)
बाणभ B)
पा णनी
TTA
:
12
Seconds C)
शू क D)
पतंज ल
A)
 चनाब B)
 ास 18.
'न चकेता
का
उ ेख
__________
म
ह।
C)
झेलम D)
रावी TTA
:
9
Seconds
6.
भगवान
गौतम
बु 
का
ज 
 ान
ह A)
अथववेद B)
कठोप नषद
TTA
:
8
Seconds C)
शतपथ
 ा ण D)
इनम
से
कोई
नह
A)
लु नी B)
सारनाथ 19.
भारत
म
वै दक
स ता
__________
नदी
के
 कनार
 वक सत
 ई।
C)
बोध
गया D)
वैशाली TTA
:
12
Seconds
7.
इं डका
पु क
 कसने
 लखी
ह? A)
तापी B)
सर ती
TTA
:
7
Seconds C)
नमदा D)
गोदावरी
A)
मेग नीज B)
अर ू 20.
'मोहनजो-दड़ो'
श 
का
अथ
 ा
ह?
C)
चाण D)
इनम
से
कोई
नह TTA
:
7
Seconds
8.
वै दक
धम
को
________
के
 प
म
भी
जाना
जाता
था। A)
पसंदीदा
शहर B)
मुद 
का
टीला
TTA
:
13
Seconds C)
रहने
क 
जगह D)
एक
बाजार
 े
A)
बु 
धम B)
 ा णवाद 21.
वै दक
काल
म,
समाज
को
_______
वग 
म
 वभा जत
 कया
गया
था
 जसे
C)
जैन
धम D)
हडो न वण
कहा
जाता
ह।
9.
1921
म
म टगोमेरी
 जले
म
रावी
नदी
के
तट
पर
 त
 मुख
शहर
कौन
सा TTA
:
12
Seconds
A)
छह B)
तीन
था?
TTA
:
15
Seconds C)
पाँच D)
चार
A)
क ौज B)
हड़ ा 22.
ईसा
पूव
छठी
शता ी
म
मगध
महाजनपद
का
पहला
शासक
था-
C)
वैशाली D)
मगध TTA
:
15
Seconds
10.
चं गु 
 थम
को
उसके
पु 
_______
 ारा
 ानांत रत
 कया
गया
था। A)
महावीर B)
 सेनजीत
TTA
:
13
Seconds C)
 ब सार D)
अजातश ु
A)
समु गु B)
भानुगु 23.
ऋ ेद
क 
अ धकांश
स ताएँ 
 कस
नदी
के
पास
 त
थ ?
C)
 ं दगु D)
पु गु TTA
:
13
Seconds
11.
मृ क टक
________
 ारा
 लखा
गया
ह। A)
नमदा B)
सर ती
TTA
:
9
Seconds C)
गंगा D)
गोदावरी
A)
का लदास B)
सु का 24.
वै दक
काल
के
मह पूण
दवताओं
के
सही
समूह
का
चयन
क जये।
C)
 बलहाना D)
भासा TTA
:
19
Seconds
12.
 स 
गाय ी
मं 
 कसके
 ारा
बनाया
गया
था? A)
 ा,
 जाप त,
ग ड़ B)
शेषनाग,
चं ,
यम
TTA
:
11
Seconds C)
व ण,
 व ु,
 शव D)
अ ,
इं ,
सोम
A)
मनु B)
कौ 25.
 ारं भक
वै दक
काल
के
दौरान
दो
 वधानसभाएं 
 ा
थ ?
C)

व ा म D)
उ ी TTA
:
11
Seconds
13.
गौतम
बु 
का
ज 
लु नी
म
 आ
था
इसक 
पु 
 कसके
 शलालेख
से A)
स म त B)
सभा
होती
ह? C)
स म त
और
सभा D)
न
तो
1
और
न
ही
2
https://amzn.to/3HVqlZK ाचीन
इ तहास

26.
'यजुवद'
म
'यजुर'
श 
का
 ा
अथ
 ा
ह? A)
गुजरात B)
म 
 दश


TTA
:
24
Seconds C)
राज ान D)
उ र
 दश
A)
 जदगी B)
 कृ त
40.
मौय
वंश
का
सं ापक
कौन
था?
C)
ब लदान D)
स
TTA
:
11
Seconds
27.
 न ल खत
म
से
कौन
सा
सबसे
पुराना
वेद
ह? A)
च गु 
 तीय B)
चं गु 
मौय
TTA
:
8
Seconds C)
 व ुगु D)
अशोक
A)
ऋ ेद B)
यजुवद
41.
मगध
महाजनपद
क 
राजधानी
 ा
थी?
C)
सामवेद D)
अथववेद
TTA
:
9
Seconds
28.
ऋ ेद
म
कुल
_________
भजन
ह A)
 ाव ी B)
चंपा
TTA
:
8
Seconds C)
पाट लपु D)
पावा
A)
1028 B)
1208
42.
प व
काल
के
दौरान
 न मत
सात
रथ
मं दर
कहाँ
 त
ह?
C)
1017 D)
1020
TTA
:
12
Seconds
29.
 न ल खत
म
से
 कस
पाठ
को
'वेदांत'
कहा
जाता
ह? A)
ऐहोल B)
महाबलीपुरम
TTA
:
14
Seconds C)
कांची D)
बादामी
A)
वेदांग B)
अयका
43.
 सधु
घाटी
के
घर 
को
________
का
उपयोग
करके
बनाया
गया
था।
C)
उप नषद D)
 ा ण
TTA
:
9
Seconds
30.
 कसके
अधीन,
चौथे
बौ 
प रषद
का
आयोजन
 आ
था? A)
क चड़ B)
 ट
TTA
:
13
Seconds C)
प र D)
बांस
A)
अशोक B)
कालाशोक
44.
 स 
महाबो ध
मं दर
कहाँ
 त
ह?
C)
क न D)
अजातश ु
TTA
:
11
Seconds
31.
हषवधन
क 
जीवनी,
'हषच रत'
एक
सं ृ त
क व
______
 ारा
 लखी
गई A)
कुशीनगर B)
लु नी
थी
जो
थानेसर
के
साथ
उनके
जुड़ाव
का
वणन
करता
ह। C)
बोध
गया D)
सारनाथ
TTA
:
9
Seconds
45.
अशोक
का
संबंध
 कस
राजवंश
से
था?
A)
भासा B)
का लदास
TTA
:
11
Seconds
C)
बाणभ D)
दांडी
A)
नंदा B)
गु
32.
सूय
भगवान
को
सम पत
एक
 वशाल
मं दर,
________
म
 त
ह। C)
क D)
मौय
TTA
:
9
Seconds
46.
नंद
वंश
का
अं तम
राजा
कौन
था?
A)
कनाटक B)
उड़ीसा
TTA
:
9
Seconds
C)
गुजरात D)
म 
 दश
A)
कैवत B)
धना
नंद
33.
‘अ 
महा ान’
बु 
के
जीवन
से
संबं धत
आठ
मह पूण
 ान 
को
कहा C)
पा ु का D)
गो वषाणक
जाता
ह।
 न ल खत
म
कौन
सा
उन
 ान 
म
से
एक
नह 
ह?
47.
गाय ी
मं 
 कस
वेद
म
सि 
म लत
ह?
TTA
:
14
Seconds
TTA
:
12
Seconds
A)
सारनाथ B)
रायगड़
A)
अथववेद B)
सामवेद
C)
लुं बनी D)
बोध
गया
C)
यजुवद D)
ऋ ेद
34.
इं डका
 कसके
 ारा
 लखा
गया
था:
48.
बौ 
धम
क 
न व
____
आय
स 
और
_____
माग
ह।
TTA
:
5
Seconds
TTA
:
17
Seconds
A)
 बो B)
मेग नीज
A)
छह,
चार B)
आठ,
छह
C)
ज न D)
 नी
C)
दो,
आठ D)
चार,
आठ
35.
"जातक"
 कस
धम
के
लोग 
का
प व 
 
ह?
49.
 ु नया
म
सबसे
बड़ा
महाका 
कौनसा
ह:
TTA
:
9
Seconds
TTA
:
7
Seconds
A)
 ह ु B)
जैन
A)
बाइबल B)
महाभारत
C)
 त त D)
बौ
C)
जांडा
अवे ा D)
कुरान
36.
 दयोफर
_________
म
 त
एक
पुराता क
 ल
ह।
50.
 न ल खत
म
से
 कस
शासक
के
शासनकाल
म
 स 
 ीक
राज ू त
TTA
:
6
Seconds
A)
 मजोरम B)
असम
मेग नीज
भारत
आए
थे?
TTA
:
15
Seconds
C)
मेघालय D)
नगालड
A)
अशोक B)
हषवधन
37.
बु 
ने
अपना
पहला
उपदश
कहाँ
 दया
था? C)
कुमारगु 
 थम D)
चं गु 
मौय
TTA
:
9
Seconds
51.
मौय
वंश
के
शासक
से
पहले,
स ाट
अशोक
________
के
गवनर
थे।
A)
काशी B)
सारनाथ
TTA
:
13
Seconds
C)
कुशीनगर D)
बोध
गया
A)
उ ैन B)
द तया
38.
हड़ ा
क 
खुदाई
________
 ारा
क 
गई
थी। C)
इं दौर D)
भोपाल
TTA
:
12
Seconds
52.
 सधु
घाटी
स ता
के
लोग
________
क 
पूजा
करते
थे।
A)
राखाल
दास
बनज B)
ई.
जे.
एच.
मैके
TTA
:
7
Seconds
C)
राय
बहा ुर
दया
राम
साहनी D)
आर.एस.
 ब
A)
हनुमान B)
काली
39.
लोथल
 ाचीन
 सधु
घाटी
स ता
का
एक
शहर
ह
जो
________
म
 त C)
अय ा D)
पशुप त
ह।
53.
 न ल खत
म
से
कौन-सी
जगह
गौतम
बु 
के
ज 
से
जुड़ी
ह?
TTA
:
10
Seconds
https://amzn.to/3HVqlZK ाचीन
इ तहास

TTA
:
11
Seconds A)
 ब ुसार B)
दशरथ


A)
लु नी B)
बोध
गया C)
धना
नंद D)
बृह थ
C)
सारनाथ D)
कुशीनगर
67.
गढ़
और
 नचला
शहर
श 
 कस
शहर
से
जुड़
ह?
54.
गु 
वंश
का
सं ापक
________
था। TTA
:
12
Seconds
TTA
:
9
Seconds A)
ह ी B)
मगध
A)
कुमारगु B)
चं गु 
 थम C)
हड़ ा D)
मथुरा
C)
 ी
गु D)
समु गु
68.
बौ 
धम
म
" -र "
 ा
दशाता
ह?
55.
मेवाड़
के
उस
यो ा
का
नाम
बताइए
 जसके
शरीर
पर
80
चोट
के
 नशान TTA
:
12
Seconds
थे? A)
बु ,
ध ,
संघ B)
सारनाथ,
लु नी,
बोध
गया
TTA
:
11
Seconds C)
 ार,
क णा,
दया D)
स ,
अ हसा,
दया
A)
राणा
सांगा B)
लखा
 सह
69.
मौय
सा ा 
क 
राजधानी
 ा
थी?
C)
अमर
 सह D)
ब ा
रावल
TTA
:
10
Seconds
56.
 न ल खत
म
से
 कस
उप नषद
म
'स मेव
जयते'
क 
उ 
 ई
ह? A)
पाट लपु B)
वैशाली
TTA
:
7
Seconds C)
इं D)
कुशीनगर
A)
केना B)
कथा
70.
 कस
गु 
शासक
को
भारत
का
नेपो लयन
कहा
जाता
ह?
C)
मुंडका D)
मै ी
TTA
:
9
Seconds
57.
कालीबंगा
 सधु
घाटी
 ल
 न ल खत
म
से
 कस
भारतीय
रा 
म
 त A)
रामगु B)
 गु
ह? C)
चं गु 
 थम D)
समु गु
TTA
:
12
Seconds
71.
चीनी
या ी
 न
 ांग
ने
 कस
राजा
के
शासन
के
दौरान
भारत
का
दौरा
 कया
A)
म 
 दश B)
कनाटक
था?
C)
पंजाब D)
राज ान
TTA
:
7
Seconds
58.
ह रयाणा
म
कौन
सा
पुराता क
 ल
ह? A)
पुलके शन B)
हषवधन
TTA
:
13
Seconds C)
राजवधन D)
क तवमन
A)
लोथल B)
धोलावीरा
72.
______
को
'ए शया
के
 काश'
के
 प
म
भी
जाना
जाता
ह।
C)
राखीगढ़ी D)
महरौली
पाक
TTA
:
15
Seconds
59.
 कस
धम
के
 भ ुओ
ं के
 लए
चै 
और
 वहार
का
 नमाण
 कया
गया
था? A)
 मी B)
बु
TTA
:
14
Seconds C)
गांधी D)
 ामी
 ववेकानंद
A)
य दी
धम B)
ईसाई
धम
73.
 न ल खत
म
से
अ ा ायी
 कसके
 ारा
र चत
ह?
C)
बौ 
धम D)
 ह ू 
धम
TTA
:
9
Seconds
60.
'रामच रतमानस’
 कसने
 लखी? A)
का लदास B)
वेद ास
TTA
:
8
Seconds C)
चाण D)
पा ण न
A)
तुलसीदास B)
वेद ास
74.
इनम
से
 ा
 सधु
घाटी
स ता
का
एक
मह पूण
 ल
नह 
ह?
C)
का लदास D)
पा ण न
TTA
:
9
Seconds
61.
संगम
युग
म
तोलका यम
______
सा ह 
क 
सबसे
बड़ा
रचना
ह। A)
लोथल B)
 ड ूगढ़
TTA
:
12
Seconds C)
मोहनजोदड़ो D)
हड़ ा
A)
त मल B)
तेलगु
75.
धना
नंद
को
हराने
के
बाद
मौय
वंश
क 
 ापना
 कसने
क ?
C)
सं ृ त D)
क ड़
TTA
:
11
Seconds
62.
हषच र 
पु क
के
लेखक
कौन
ह? A)
 ब ुसार B)
अशोक
TTA
:
11
Seconds C)
कुणाल D)
चं गु
A)
का लदास B)
शू क
76.
हषवधन
दरबार
का
दौरा
करने
के
बाद
शी-यू-क 
’पु क
 कसने
 लखी?
C)
बाणभ D)
हॉल
TTA
:
11
Seconds
63.
वह
पहला
दश
 जसम
भारत
के
बाहर
बौ 
धम
का
 चार
 कया
गया
था A)
पु
स ग लग B)
 न
 ांग-
TTA
:
12
Seconds C)
झांग
 पग D)
बाणभ
A)
जापान B)
चीन
77.
 कस
स ता
से
लोथल
के
बंदरगाह
शहर
का
पता
लगाया
जा
सकता
ह?
C)
द ण
को रया D)
 ीलंका
TTA
:
12
Seconds
64.
शुंग
वंश
क 
 ापना
 कसने
क ? A)
 वड़
स ता B)
द न
घाटी
स ता
TTA
:
10
Seconds C)
इं डो- ीक
स ता D)
हड़ ा
स ता
A)
पु म 
शुंग B)
अ म 
शुंग
78.
 थम
तीथकर
कौन
ह?
C)
वसु े 
शुंग D)
वसु म 
शुगा
TTA
:
10
Seconds
65.
सं ृ त
म
रामायण
 कसके
 ारा
 लखी
गई
ह? A)
ऋषभदव B)
ने मनाथ
TTA
:
15
Seconds C)
पा नाथ D)
व मान
A)
वा ी क B)
कबीर
79.
क न 
 कस
वंश
से
संबं धत
था?
C)
तुलसीदास D)
 ास
TTA
:
9
Seconds
66.
मौय
वंश
का
अं तम
राजा
____________
था। A)
मौय B)
प व
TTA
:
10
Seconds C)
कुषाण D)
चोल
https://amzn.to/3HVqlZK ाचीन
इ तहास

80.
राजा
हषवधन
ने
 ान
का
एक
बड़ा
क 
 ा पत
 कया,
 जसे
_____
म 92.
उस
पु क
का
 ा
नाम
ह
 जसम
बौ 
संघ
 ारा
बनाए
गए
 नयम 
का
'भ - वहार'
कहा
जाता
था। उ ेख
 मलता
ह?
TTA
:
16
Seconds TTA
:
15
Seconds
A)
मथुरा B)
काशी A)
 ो B)
 वनय
 पटक
C)
क ौज D)
ह र ार C)
महा भ न मण D)
पंचतं
81.
'दवानाम या
 यदश '
के
नाम
से
 कसे
जाना
जाता
था? 93.
 न 
का
 मलान
क जये:
TTA
:
12
Seconds पु क लेखक
A)
अशोक
मौय B)
 ब ुसार
मौय
1 अ ा ायी A भरत
C)
चं गु 
मौय D)
महाप 
नंदा
2 ना 
शा B वराह म हर
82.
बौ 
 ंथ
म मा
 नकाय
 कसम
ह
3 बृहत्
जातक C पा ण न
TTA
:
13
Seconds
A)
सं ृ त B)
पाली TTA
:
26
Seconds
C)
 ाकृत D)
तेलेगु A)
1(B),
2(C),
3(A) B)
1(B),
2(A),
3(C)
C)
1(C),
2(A),
3(B) D)
1(C),
2(B),
3(A)
83.
 न
 ांग
ने
 न ल खत
म
से
 कस
शासक
के
दरबार
का
दौरा
 कया?
TTA
:
12
Seconds 94.
सुरकोटदा
का
पुराता क
 ल
 कस
रा 
म
 त
ह?
A)
अशोक B)
हषवधन TTA
:
12
Seconds
C)
 ब सार D)
चं गु 
मौय A)
गुजरात B)
कनाटक
C)
राज ान D)
ह रयाणा
84.
 न ल खत
म
से
 कस
अव ध
को
 ह ू 
धम
के
 ण
युग
के
 प
म
जाना
जाता
ह? 95.
कौन-सा
बो धस 
भगवान
इं 
क 
तरह
व 
धारण
करता
ह?
TTA
:
12
Seconds TTA
:
12
Seconds
A)
मौय
काल B)
गु 
काल A)
मंजू ी B)
मै ेय
C)
उ र
गु 
काल D)
उ र

मौय
काल C)
व पाणी D)
अ मताभ
85.
 सधु
घाटी
स ता
का
एक
बंदरगाह
शहर
 न ल खत
म
से
कौन
सा
था? 96.
 न ल खत
म
से
 कस
पुराता क
 ल
म
गत
आवास 
के
 माण
ह?
TTA
:
10
Seconds TTA
:
12
Seconds
A)
राखीगढ़ी B)
धोलावीरा A)
बुजहोम B)
मेहरगढ़
C)
कालीबंगा D)
लोथल C)
राणा
पुंडई D)
पलवोय
86.
चाण 
के
कई
 वचार 
को
 न 
म
से
 कस
पु क
म
 लखा
गया
था? 97.
बौ 
धम
म
" र "
का
 ा
अथ
ह?
TTA
:
11
Seconds TTA
:
14
Seconds
A)
पुराण B)
राजतरं गणी A) पटक B)बु ,
ध 
(धम),
संघ
C)
अथशा D)
महा वा C)स ,
अ हसा,
क णा D)शैल,
समा ध,
संघ
98.
 न ल खत
म
से
 ा
वेदांग
नह 
ह?
TTA
:
12
Seconds
LEVEL
2 87
-
166
Questions A)
क B)
 न
C)
पूवमीमांसा D)
 श ा
87.
'बृह ं हता'
के
लेखक
कौन
थे? 99.
 सधु
घाटी
स ता
का
 ाचीन
बंदरगाह
 न ल खत
म
से
कौन
सा
ह?
TTA
:
16
Seconds TTA
:
11
Seconds
A)
आयभ B)
वराह म हर A)
मोहनजोदड़ो B)
कालीबंगा
C)
अमर सह D)
 गु C)
हड़ ा D)
लोथल
88.
अजंता
क 
 च कला
म
मंजु ी
 कस
धम
से
संबं धत
ह? 100.
'भारतीय
मै कयावेली'
 कसे
कहा
जाता
था?
TTA
:
13
Seconds TTA
:
12
Seconds
A)
बौ 
धम B)
जैन
धम A)
कौ ट B)
 वशाखद
C)
आजीवक D)
उपरो 
म
से
कोई
नह C)
मेग नीज़ D)
चं गु 
मौय
89.
1922
म
 सधु
के
तट
पर
लरकाना
 जले
म
 कस
शहर
क 
खुदाई
क 
गई 101.
24व
जैन
तीथकर
कौन
थे?
थी? TTA
:
10
Seconds
TTA
:
16
Seconds A)
ऋषभदव B)
महावीर
A)
मोहनजोदड़ो B)
धनुषकोडी C)पा नाथ D)
सुम तनाथ
C)
कांची D)
हड़ ा 102.
सां 
 ू ल
ऑफ
 फलॉसफ 
क 
 ापना
________
 ारा
क 
गई
थी।
90.
 ू सरा
बौ 
प रषद
__________
के
शासनकाल
के
दौरान
आयो जत TTA
:
15
Seconds
कया
गया
था। A)
पतंज ल B)
गौतम
TTA
:
14
Seconds C)
कुम रला
भ D)
क पला
A)
उदयभ B)
महाप नंदा
103.
यूना नय 
के
लेखन
म
 कसे
'स ोकोटस'
कहा
जाता
था?
C)
 शशुनाग D)
कालाशोक TTA
:
17
Seconds
91.
प व 
पु क
'भगवद
गीता'
 कसने
 लखी? A)
अशोक B)
 ब ुसार
TTA
:
14
Seconds C)
चं गु 
मौय D)
घनानंद
A)
का लदास B)
वा ी क
104.
 न ल खत
म
से
कौन
सी
पु क
हषवधन
 ारा
नह 
 लखी
गई
थी?
C)
सूरदास D)
वेद
 ास TTA
:
28
Seconds
To Practice 3751 More Questions
From Remaining Chapters

Buy NOW

Available at
&

Unique Features of SmartBook


Time To Answer (TTA)

Smart Answer Key

Level - Wise Difficulty

Best 4000 Questions

Free Video Lessons


https://amzn.to/3HVqlZK ाचीन
इ तहास

A)
र ावली B)
नागानंद 1.
अवं त:
उ ैन


C)
काद री D)
 यद शका 2.
काशी:
बनारस
3.
व :
च ा
105.
गु 
काल
का
सा ह कार
कौन
ह? 4.
व :
कौशा ी
TTA
:
18
Seconds TTA
:
30
Seconds
A)
भैरवी B)
का लदास A)
1,
2
और
3 B)
2,
3
और
4
C)
ह रसेना D)
उपयु 
सभी C)
1,
2
और
4 D)
1,
3
और
4
106.
जातक
कथाएँ 
इनम
से
 कस
सं दाय
से
जुड़ी
ह? 116.
साबी
नदी
के
तट
पर
 त
उस
 ल
का
 ा
नाम
था,
जहाँ
राज ान
क
TTA
:
10
Seconds ाचीन
स ता
बसी
 ई
थी?
A)
 लगायत B)
शैव
धम TTA
:
22
Seconds
C)
जैन
धम D)
बौ 
धम A)
 गलु B)
जोधपुरा
107.
'माल वका-अ म म्'
 कसके
 ारा
 लखा
गया
था? C)
नगारी D)
बरोर
TTA
:
10
Seconds 117.
 ह ू 
पौरा णक
कथाओं
के
संदभ
म,
 न ल खत
म
से
कौन
भगवान
 व ु
A)
बाणभ B)
कबीर के
दस
अवतार 
का
 ह ा
नह 
ह?
C)
का लदास D)
सूरदास TTA
:
18
Seconds
A)
ग ड़ B)
कुम
108.
 न 
म
से
 कस
दश
से
हड़ ा
के
लोग 
ने
कोई
 ापार
नह 
 कया?
TTA
:
15
Seconds C)
वराह D)
म
A)
प सया B)
अफगा न ान 118.
'कोट
दीजी'
स ता
का
स 
रहा
ह?
C)
ओमान D)
मंगो लया TTA
:
16
Seconds
A)
हड़ ा
पूव
क 
स ता
से B)
चीन
क 
स ता
से
109.
 न ल खत
म
से
कौन
सा
 ान
सबसे
पहले
खोजा
गया
था?
TTA
:
11
Seconds C)
 म 
क 
स ता
से D)
सुमे रया
क 
स ता
से
A)
मोहनजोदड़ो B)
लोथल 119.
"स 
क 
खोज"
म
 स ाथ
के
 ान
को
जाना
जाता
ह-
C)
हड़ ा D)
अमरी
नाल TTA
:
17
Seconds
A)
धमच प रवतन B)
महा भ न मण
110.
 न ल खत
का
सही
 मलान
क जये-
C)
 नवाण D)
प र नवाण
A.
ऋ ेद 1)
संगीतमय
भजन
B.
यजुवद 2)
भजन
और
अनु ान
120.
कालीबंगन,
 सधु
घाटी
 ल
________
म
ह।
TTA
:
10
Seconds
C.
सामवेद 3)
तं 
और
मं
A)
राज ान B)
गुजरात
D.
अथववेद 4)
भजन
और
 ाथना C)
म 
 दश D)
उ र
 दश
TTA
:
33
Seconds 121.
"स 
क 
खोज"
म
 स ाथ
का
 ान
को
 ा
कहा
जाता
ह?
A)
A
-
4,
B
-
2,
C
-
1,
D
-
3 B)
A
-
3,
B
-
2,
C
-
4,
D
-
1 TTA
:
18
Seconds
C)
A
-
4,
B
-
1,
C
-
2,
D
-
3 D)
A
-
2,
B
-
3,
C
-
2,
D
-
4 A)
धमच प रवतन B)
महा भ न मण
111.
उस
सा ा 
का
नाम
बताइए
 जसने
सव थम
यु 
म
हा थय 
का
उपयोग C)
 नवाण D)
प र नवाण
कया
था? 122.
ऋ े दक
काल
म,
अर ानी
________
क 
दवी
थी।
TTA
:
12
Seconds TTA
:
15
Seconds
A)
चंपा B)
मगध A)
वन B)
पृ ी
C)
कोशल D)
अवंती C)
वायु D)
अ णोदय
112.
 न ल खत
म
से
कौन
म 
महाजनपद
क 
राजधानी
थी? 123.
 न ल खत
म
से
कौन
सी
चार
 ाचीन
स ताओं
म
सबसे
बड़ी
ह?
TTA
:
13
Seconds TTA
:
13
Seconds
A)
कौशा ी B)
इं A)
मेसोपोटा मया B)
चीन
C)
 वराटनगर D)
मथुरा C)
 म D)
 स ु
घाटी
स ता
113.
उनक 
राजधा नय 
(सूची
II)
के
साथ
महाजनपद
(सूची
I)
का
 मलान
कर: 124.
 न ल खत
म
से
कौन
सा
 ल
यूने ो
का
 व 
धरोहर
 ल
नह 
ह?
सूची
I
(महाजनपद) सूची
II
(राजधानी) TTA
:
17
Seconds
A.
मगध I.
कौशांबी A)
चंपानेर-पावागढ़
पुरात 
पाक B)
लोथल
C)
भीमबेटका
गुफ़ाएँ D)
ताजमहल
B.
व II.
राजगृह

C.
सुरसेना III.
राजपुरा
125.
वै दक
समाज
को
चार
वग 
म
 वभा जत
करने
का
उ ेख
 कस
 ंथ
म
ह?
TTA
:
18
Seconds
D.
क ोज IV.
मथुरा
A)
ऋ ेद
का
पु ष
सू B)
यजुवद
नीचे
 दए
गए
 वक 
म
से
सही
उ र
के
 लए
कोड
का
चयन
कर: C)
मुंडक
उप नषद D)
शतपथ
 ा ण
TTA
:
30
Seconds
126.
सं ृ त
भाषा
का
पहला
 ाकर णक
पाठ
__________
 ारा
 लखा
गया
A)
A
-
II,
B
-
I,
C
-
IV,
D
-
III B)
A
-
II,
B
-
III,
C
-
IV,
D
-
I
था।
C)
A
-
III,
B
-
II,
C
-
IV,
D
-
I D)
A
-
III,
B
-
II,
C
-
I,
D
-
IV
TTA
:
12
Seconds
114.
 न ल खत
म
से
 कस
वेद
म
रोग 
का
उपचार
ह? A)
वा ी क B)
क ण
TTA
:
19
Seconds C)
पा ण न D)
का लदास
A)
यजुवद B)
ऋ ेद
127.
जेपी
जोशी
ने
 न ल खत
म
से
 कस
 ान
क 
खोज
क 
थी?
C)
सामवेद D)
अथववेद
TTA
:
17
Seconds
115.
 न ल खत
म
से
कौन
सा
महाजनपद
अपनी
राजधानी
से
सही
ढंग
से
मलान
होता
ह/ह?
https://amzn.to/3HVqlZK ाचीन
इ तहास

A)
लोथल B)
च दड़ो A)
ऋषभनाथ B)
पा नाथ


C)
धोलावीरा D)
बनवाली C)
अ जतनाथ D)महावीर
128.
परी त
और
जनमेजय
________
सा ा 
के
 स 
शासक
थे। 141.
ना सक
के
 शलालेख 
पर
 कस
राजा
क 
उपल यां
दज
ह?
TTA
:
19
Seconds TTA
:
14
Seconds
A)
म B)
कु A) गौतमी
पु 
शातकण B)
अशोक
C)
य ु D)
पु C)
खारवेल D)
इनम
से
कोई
भी
नह
129.
 न ल खत
म
से
 कसने
'रघुवंशम'
का
संकलन
 कया
ह? 142.
 न ल खत
म
से
 कस
चीनी
बौ 
या ी
ने
राजा
हष
और
राजा
पुलके शन
TTA
:
10
Seconds II
दोन 
का
दौरा
 कया?
A)
सूरदास B)
कबीरदास TTA
:
17
Seconds
C)
का लदास D)
तुलसीदास A)
चीनी
बौ 
 भ ु
 न
 ांग B)
चीनी
तीथया ी
आई- ग
C)
उपयु 
दोन D)
इनम
से
कोई
भी
नह
130.
________
काल
को
भारतीय
इ तहास
के
' ण
युग'
के
 प
म
व णत
कया
गया
ह। 143.
ऐहोल
 शलालेख
 न ल खत
म
से
 कस
शासक
के
साथ
जुड़ा
 आ
ह?
TTA
:
10
Seconds TTA
:
14
Seconds
A)
मुगल B)
मराठा A)
 व मा द B)
पुलके शन
II
C)
गु D)
बु C)
अकबर D)
अशोक
131.
सोना ग र
________
का
एक
तीथ ल
ह। 144.
 न ल खत
म
से
कौन
क 
वंश
के
सं ापक
थे?
TTA
:
11
Seconds TTA
:
13
Seconds
A)
जैन B)
मु म A)
 दामन
I B)
 समुका
C)
बौ D)
 सख C)
खारवेल D)
वासुदव
132.
 न
 ांग
 कसके
शासनकाल
म
भारत
आया
था? 145.
अल- ब नी
के
अनुसार,
थानेसर
के
मं दर
म
___________
 न मत
TTA
:
12
Seconds दवताओं
क 
कई
मू तयां
थ ।
A)
समु गु B)
चं गु 
 व मा द TTA
:
17
Seconds
C)
हषवधन D)
 गु A)
लोहा B)
तांबा
C)
कां D)
चांदी
133.
7व 
शता ी
सीई
म
हषवधन
के
सा ा 
क 
राजधानी
__________
थी। 146.
इ तहासवे ा
 ारा
 ात
कुल
उप नषद 
क 
सं ा
 कतनी
ह?
TTA
:
13
Seconds TTA
:
11
Seconds
A)
क ौज B)
थाने र A)
11 B)
21
C)
पाट लपु D)
मालवा C)
108 D)
1080
134.
 न ल खत
म
से
 कस
 ान
पर
घोड़
क 
ह य 
के
अवशेष
पाए
गए
ह? 147.
 न ल खत
म
से
कौन
सा
 सधु
घाटी
 ल
अपने
खोजकता
से
सही
ढंग
TTA
:
14
Seconds से
सुमे लत
नह 
ह?
A)
सुरकोटदा B)
धोलावीरा TTA
:
32
Seconds
C)
लोथल D)
मोहनजोदड़ो A)
लोथल:
 शकारीपुरा
रंगनाथ
राव B)
कालीबंगा:
ए.
घोष
C)
च दारो:
आर.
डी.
बनज D)
रोपड़:
वाई.
डी.
शमा
135.
इनम
से
कौन
सा
अ भलेख
क लग
के
साथ
अशोक
के
यु 
के
बार
म
ववरण
दता
ह? 148.
व यान
 ू ल
 कस
धम
से
संबं धत
ह?
TTA
:
10
Seconds TTA
:
13
Seconds
A)
 मुख
 शलालेख
13 B)
 
लेख
4 A)
जैन
धम B)
बौ 
धम
C)
 
लेख
5 D)
सू 
 शलालेख
1 C)
 ह ू 
धम D)
उपरो 
म
से
कोई
नह
136.
संगम
युग
क 
एक
 ापक
कृ त,
टो यम
से
संबं धत
ह 149.
भारत
म
 मुख
बौ 
संरचनाओं
म
से
एक,
सारनाथ
म
________
 ूप
का
TTA
:
14
Seconds नमाण
महान
मौय
राजा,
अशोक
 ारा
 कया
गया
था।
A)
आयु
क 
राजनी त B)
त मल
 ाकरण TTA
:
14
Seconds
C)
अव ध
के
यु D)
कर
और
शु 
का
सं ह A)
धौली B)
धमेख
C)
भर त D)
ल लत ग र
137.
गाय ी
मं 
क 
रचना
 न ल खत
म
से
 कसने
क ?
TTA
:
12
Seconds 150.
गु 
क 
राजधानी
_________
म
थी।
A)
गो ामी
तुलसीदास B)
 व ाम TTA
:
14
Seconds
C)
व श D)
इं A)
राजगृह B)
वैशाली
C)
पाट लपु D)
कनौज
138.
 न ल खत
 ाचीन
या य 
म
से
कौन
चं गु 
-
 तीय
क 
अव ध
के
समकालीन
थे? 151.
चं गु 
मौय
के
समय
भारत
कौन
आया
था?
TTA
:
15
Seconds TTA
:
12
Seconds
A)
मेग नीज B)
 डमाकस A)
फ़ै यन
(फ़ा
 हएन) B)
जुआनज़ग
( न
 ांग)
C)
फा- हएन D)
जुआनजांग C)
मेग नीज D)
स ाबो
139.
_______
ने
राजा
हान
हो-टी
के
 खलाफ
लड़ाई
लड़ी,
जो
चीन
के
हान 152.
 ा ु ट स ांत
 न ल खत
म
से
 कसके
 ारा
 लखी
गई
पु क
ह?
वंश
के
राजा
थे,
और
 ू सर
 यास
म
उ 
हरा
 दया। TTA
:
16
Seconds
TTA
:
20
Seconds A)
आयभ B)
भा राचाय
A)
क न B)
 ब ुसार C)
अ नी
कुमार D)
Bramhagupta
C)
चं गु 
मौय D)
अशोक
153.
मौय
 शासन
म
पूर
रा 
से
राज 
सं ह
का
मु 
पयवे क
कौन
था?
140.
जैन
धम
के
23व
तीथकर
कौन
थे? TTA
:
14
Seconds
TTA
:
11
Seconds
https://amzn.to/3HVqlZK ाचीन
इ तहास

A)
अंतपाल B)
कमा का TTA
:
17
Seconds


C)
सं नद D)
समहता A)
अली
आ दल
शाह
I B)
मीर
जुमला
C)
इ ा हम
आ दल
शाह
II D)
इ ाहीम
क़ुतुबशाह
154.
 न ल खत
म
से
 कस
शासक
ने
नालंदा
महा वहार
क 
 ापना
क 
थी?
TTA
:
12
Seconds 166.
वै दक
 स ांत
के
अनुसार
 न ल खत
म
से
कौन
सा
गलत
ह?
A)
समु गु B)
चं गु 
I TTA
:
27
Seconds
C)
कुमारगु D)
 ी
गु A)
 य,
या
यो ा
शासक,
पु षा
क B)
वै ,
या
सामा जन,
पु ष
क
बाह 
से
आए
थे जांघ 
से
आए
थे
155.
ऋ ेद
म
 न ल खत
म
से
 कस
नदी
का
उ ेख
नह 
ह?
C)
 ा ण,
या
पुजारी,
पु ष
के
कान
से D)
शू ,
या
मज ू र
और
नौकर,
पु ष
के
TTA
:
13
Seconds
आए
थे पैर 
से
आए
थे
A)
गंगा B)
यमुना
C)
सर ती D)
नमदा
156.
 तीय
बौ 
प रषद
कहाँ
और
कब
आयो जत
क 
गई
थी? LEVEL
3 167
-
249
Questions
TTA
:
17
Seconds
A)
250
ईसा
पूव
म
पाट लपु , B)
पहली
शता ी
ईसा
पूव
म
 ीलंका
C)
383
ईसा
पूव
म
वैशाली D)
1871
ई.
म
मंड लया 167.
वै दक
सा ह 
का
सबसे
 ाचीन
 ा ण
 न ल खत
म
से
कौन
ह?
TTA
:
13
Seconds
157.
शाही
चोलाओं
के
अधीन
राज 
 शासन
के
संबंध
म
'शालाभोग'
श 
का A)
ऐतरय B)
सां
ा
अथ
ह? C)
शतपथ D)
गोपद
TTA
:
21
Seconds
A)
 कसी
यो ा
को
दान
क 
गई
भू म B)
एक
नया
बसा
 आ
गाँव 168.
 न ल खत
म
से
कौन
गु 
काल
के
दौरान
 च क ा
पर
अपने
काय
के
C)
 व ालय
के
रखरखाव
के
 लए
दान D)
 सचाई
सु वधाओं
के
रखरखाव
के लए
जाना
जाता
ह?
क 
गई
भू म लए
दान
क 
गई
भू म TTA
:
13
Seconds
A)
चरक B)
सु ुत
158.
इनम
से
 कस
हड़ ाकालीन
 ल
ने
एक
जुताई
के
 े 
का
 माण
 दान
C)
सौ मला D)
पा ण न
कया
ह?
TTA
:
13
Seconds 169.
इलाहाबाद
 ंभ
 शलालेख
 कस
राजा
के
शासनकाल
का
 व ृत
 ववरण
A)
हड़ ा B)
मोहनजोदड़ो दता
ह?
C)
कालीबंगन D)
च दड़ो TTA
:
13
Seconds
A)
चं गु 
मौय B)
क न
159.
भारत
म
धा मक
 थाओं
के
संदभ
म,
" ानकवासी"
सं दाय
________
C)
समु 
गु D)
अशोक
से
संबं धत
ह।
TTA
:
22
Seconds 170.
गु 
युग
के
 न ल खत
 शलालेख 
म
से
कौन
सा
 ं दगु 
से
संबं धत
A)
बु 
धम B)जैन
धम नह 
ह
?
C) D)
शैव TTA
:
17
Seconds
वै व
सं दाय A)
जूनागढ़
रॉक B)
एरण
 शला
 ंभ
C)
इं दौर
कॉपर
 ेट D)
 भटारी
 ंभ
160.
7
व 
शता ी
के
 ारंभ
के
साथ,
थाने र
और
क ौज
के
 सहासन
पर
171.
________
को
सबसे
पुराना
खोजा
गया
 सधु
घाटी
स ता
 ल
माना
कौन
बैठा?
TTA
:
18
Seconds जाता
ह।
TTA
:
12
Seconds
A)
कृ दव B)
हषवधन
A)
मोहन
जोदड़ो B)
 भराना
C) राज 
चोल
 थम D)
च गु 
 तीय
C)
अ ाहदीनो D)
राखीगढ़ी
161.
 न ल खत
म
से
कौन
सा
वेद
जा ुई
अनु ान 
और
आकषण
के
बार
म
172.
" चनाब
नदी"
का
ऋग
वै दक
नाम
 न ल खत
म
से
कौन
सा
ह?
बताता
ह?
TTA
:
14
Seconds
TTA
:
15
Seconds
A)
अ नी B)
पु शनी
A)
सामवेद B)
यजुवद
C)
 वत ा D)
 वपाशा
C)
अथववेद D)
ऋ ेद
173.
'कथास र ागर’
 कसके
 ारा
 लखा
गया
ह?
162.
 सधु
घाटी
स ता
का
 न ल खत
म
से
कौन-सा
 ल
लूनी
नदी
के
तट
TTA
:
14
Seconds
पर
 त
ह?
A)
कालीदास B)
भास
TTA
:
18
Seconds
A)
मोहनजोदड़ो B)
हड़ ा C)
जयदव D)
सोमदव
C)
धोलावीरा D)
लोथल 174.
 न ल खत
म
से
कौन
सा
उप नषद
 ाचीनतम
माना
जाता
ह?
TTA
:
10
Seconds
163.
 व 
 ा
ह?
A)
चंडो B)
मुंडका
TTA
:
15
Seconds
A)
एक
 कार
के
भगवान B)
एक
 कार
का
मनोरंजन C)
बृहदार क D)
इनमे
से
कोई
भी
नह
C)
एक
 कार
क 
बंधुआ
मज ू री D)
इनमे
से
कोई
भी
नह 175.
 कसने
अपने
शासनकाल
म
पाट लपु 
को
मगध
क 
राजधानी
बनाया
और
164.
 न ल खत
म
से
 कसने
शक
राजा
नहपान
को
परा जत
 कया
और यहाँ
एक
छोटा
 कला
बनाया?
TTA
:
17
Seconds
सातवाहन
श 
को
पुनज वत
 कया?
A)
 ब सार B)
अजातश ु
TTA
:
16
Seconds
A)
हाला B)
गौतमीपु 
सातकण C)उदा यन D)कालाशोक
C)
पुलुमवी
तृतीय D)
 समुक 176.
 न ल खत
म
से
कौन
हड़ ा
और
मोहनजो-दारो
क 
खुदाई
से
संबं धत
165.
 न ल खत
म
से
 कसके
पास
ड न
के
'अबला
बाबा'
(गरीब 
के
दो ) नह 
ह?
TTA
:
20
Seconds
क 
उपा ध
थी?
To Practice 3751 More Questions
From Remaining Chapters

Buy NOW

Available at
&

Unique Features of SmartBook


Time To Answer (TTA)

Smart Answer Key

Level - Wise Difficulty

Best 4000 Questions

Free Video Lessons


https://amzn.to/3HVqlZK ाचीन
इ तहास

A)
आर.डी.
बनज B)
के.एन.
दी त 189.
कौन
सा
 ाचीन
 ल
भारत
के
 बहार
रा 
म
मौजूद
एकमा 
यूने ो


C)
एम.एस.
व D)
वी.ए.
 थ व 
धरोहर
 ल
ह?
TTA
:
23
Seconds
177.
 न ल खत
म
से
कौन
बौ 
धम
के
अंतगत
 मुख
महान
स 
ह/ह?
A)
शेरशाह
सूरी
का
मकबरा B)
 व म शला
मठ
का
 ाचीन
 ल
1.
 ुःख
C)
महाबो ध
 वहार D)
कुंभार
पटना
2.
समुदाय
3.
 नरोध 190.
पाषाण
काल
के
 न ल खत
काल
म
से
 कसने
सबसे
पहले
जानवर 
के
4.
माग वच 
को
दखा?
TTA
:
22
Seconds TTA
:
21
Seconds
A)
केवल
1,
2
और
3 B)
केवल
2,
3
और
4 A)
पुरापाषाण
युग B)
चालको ल थक
युग
C)
1,
3
और
4 D)
उपरो 
सभी C)
नवपाषाण
युग D)
मेसो ल थक
युग
178.
 कस
गु 
शासक
ने
 ण
आ मण
को
 नर 
 कया? 191.
 न ल खत
म
से
कौन
सा
 सधु
घाटी
का
बंदरगाह
शहर
नह 
ह?
TTA
:
14
Seconds TTA
:
17
Seconds
A)
समु गु B)
च गु 
 तीय A)
सुतकागडोर B)
बालाकोट
C)
 गु D)
कुमारगु C)
धोलावीरा D)
कुंतसी
179.
 कस
 ान
पर
हड़ ा
काल
के
रथ
क 
एक
मू त
पाई
गई
थी? 192.
 सधु
घाटी
स ता
क 
वा ुकला
के
बार
म
कौन
सी
 वशेषता
स 
नह
TTA
:
17
Seconds ह?
A)
लोथल B)
बनवाली TTA
:
31
Seconds
C)
दमाबाद D)
कालीबंगा A)
शहरी
शहर-उ ेखनीय
नगर B)
बड़
शहर
राजा
के
महल,
आम
घर
180.
अशोक
 ारा
 नयु 
 कए
गए
धा मक
अ धकारी
को
 ा
कहा
जाता
था? नयोजन,
और
जल
 नकासी
और और
सामू हक
सभा
 ारक 
के
बीच
TTA
:
24
Seconds
ता
क 
उ ृ 
 व ा तीन
भाग 
म
 वभा जत
होते
ह
A)
ध -महाम ा B)
ध धराजा C)
सड़क -ल लत
जल
 नकासी D)
भवन
 नमाण
साम ी- सधु
शहर 
म
णाली,
अ ी
तरह
से
 व त
पानी कोई
प र
का
घर
नह
C)
ध ा धकारी D)
ध ा ा ा
क 
आपू त
 णाली
181.
 न ल खत
म
से
कौन
जैन
तीथकर
पा नाथ
का
 तीक
ह?
193.
मनु ृ त
म
"एक
युवती
और
उसके
 ेमी
का
 ै क
संघ"
से
 ववाह
 कस
TTA
:
15
Seconds
प
म
होता
ह?
A)
बैल B)
कमल
TTA
:
17
Seconds
C)
सांप D)
शेर A)
आठवाँ
 प B)
पाँचवाँ
 प
182.
"सभी
मनु 
मेर
ब े
ह",
अशोक
ने
 न ल खत
म
से
 कस
 शलालेख
म C)
सातवाँ
 प D)
छठा
 प
यह
 स 
घोषणा
क 
ह? 194.
 न ल खत
म
से
 कसने
 वदह
सा ा 
क 
शु आत
क ?
TTA
:
16
Seconds
TTA
:
12
Seconds
A)
पृथक
क लग
 शला
 ापक
I B)
लु नी
 ंभ
 ापक
A)
 न म
 वदह B)
इ ाकु
C)
 
 ापक
VII D)
लघु
 शला
 ापक
(अहरौरा)
C)
 म थजानक
 वदह D)
इनम
से
कोई
नह
183.
 बहार
म
ब तायत
म
पाए
जाने
वाले
पंच- च त
 स े 
______
 ारा
195.
ऋ ेद
क 
रचना
 कस
काल
म
 ई
थी?
न मत
होते
थे। TTA
:
14
Seconds
TTA
:
13
Seconds
A)
 ारं भक
वै दक
युग B)
उ र
वै दक
युग
A)
सोना B)
चाँदी
C)
 ागै तहा सककाल D)
इनमे
से
कोई
भी
नह
C)
लीड D)
तांबा
196.
 न ल खत
म
से
कौन
सा
कथन
बाद
के
वै दक
काल
के
बार
म
सही
नह
184.
समा ध
मरण
 कस
दशन
से
संबं धत
ह-
ह?
TTA
:
13
Seconds
TTA
:
29
Seconds
A)
बौ 
दशन B)
जैन
दशन
A)
बाद
के
वै दक
काल
म
म हलाओं B)
वंशानुगत
 ापा रय 
का
एक
वग
C)
योग
दशन D)
लोकायत
दशन को
मह पूण
राजनी तक
 वशेषा धकार अथात
वा ा
अ 
म
आया
185.
चंडो 
 ा ण
 कस
वेद
से
संबं धत
ह? ा 
 ए
TTA
:
16
Seconds C)
सोना
और
चांदी
के
 स े 
जैसे
 क D)
बाल
 ववाह
आम
हो
चुके
थे
A)
ऋ ेद B)
सामवेद सतनाम
और
 शनाला
का
उपयोग
C)
यजुवद D)
अथवद व नमय
के
मी डया
के
 प
म
 कया
जाता
था
186.
'कालच '
समारोह
 न ल खत
म
से
 कस
धम
से
संबं धत
ह?
197.
ऋग-वै दक
युग
क 
 न ल खत
न दय 
म
से
 कसे
"न द
तना"
या
ऋ ेद
TTA
:
11
Seconds
A)
जैन
धम B)
बु 
धम क 
सव े 
न दय 
के
 प
म
जाना
जाता
ह?
TTA
:
17
Seconds
C)
 सख
धम D)
 ह ू 
धम
A)
 सधु
नदी B)
कुंभा
नदी
187.
महाराजा धराज
क 
उपा ध
अपनाने
वाला
पहला
गु 
शासक
कौन
था? C)
गंगा
नदी D)
सर ती
नदी
TTA
:
13
Seconds
198.
 ारं भक
वै दक
युग
के
दौरान
जा त- व ा
म
 ा
 न ष 
था
A)
च गु 
I B)
च गु 
II
TTA
:
16
Seconds
C)
समु गु D)
 ीगु
A)
अंतर
जा त
भोजन B)
अंतरजातीय
 ववाह
188.
एक
जैन
गण
के
नेता
को
________
के
 प
म
जाना
जाता
था। C)
दोन D)
इनम
से
कोई
नह
TTA
:
15
Seconds
199.
भारतीय
उपमहा ीप
म
लोह
के
सबसे
पहले
सा ह क
संदभ
 न ल खत
A)
गणप त B)
गणधर
म
से
 कसम
पाये
गए
ह:
C)
गणराज D)
गणक
TTA
:
16
Seconds
https://amzn.to/3HVqlZK ाचीन
इ तहास

A)
ऋ ेद B)
सामवेद TTA
:
23
Seconds


C)
यजुवद D)
 वनय
 पटक A)
चोल B)
सातवाहन
C)
शक D)
पां ा
200.
सुरसेन
महाजनपद
क 
राजधानी
कहाँ
थी?
TTA
:
13
Seconds 212.
सातवाहन
वंश
के
संबंध
म
इम
से
कौन
सा
कथन
सही
नह 
ह?
A)
 वराटनगर B)
त शला TTA
:
20
Seconds
C)
काशी D)
मथुरा A)
सातवाहन
वंश
का
सं ापक B)
उ ने
बौ 
धम
और
 ा णवाद
का
समुका
था। संर ण
 कया
201.
 न ल खत
 सधु
घाटी
 ल 
म
से
 कससे
दोहर
दफन
के
 लए
सबूत
C)
उ ने
क ड़
भाषा
और
सा ह 
का D)
उ ने
कई
चै 
और
 वहार
बनाए।
मले
ह? संर ण
 कया।
TTA
:
16
Seconds
A)
लोथल B)
कालीबंगा 213.
 न 
का
 मलान
क जये
C)
सुरकोटड़ा D)
मोहनजोदड़ो रा तीक

202.
 न ल खत
म
से
कौन
सी
म हला
 व ान
वै दक
युग
से
संबं धत
ह? 1 चोल A मछली


TTA
:
10
Seconds 2 चेरा B चीता
A)
गाग B)
मै ेय 3 पां ा C धनुष
और
बाण।
C)
लोपामु ा D)
उपरो 
सभी
TTA
:
29
Seconds
203.
 न ल खत
म
से
कौन
सा
जानवर
हड़ ा
सं ृ त
क 
मुहर 
और A)
1(B),
2(C),
3(A) B)
1(B),
2(A),
3(C)
टराकोटा
कला
का
 त न ध 
नह 
करता
था? C)
1(C),
2(A),
3(B) D)
1(C),
2(B),
3(A)
TTA
:
17
Seconds
A)
गाय B)
हाथी 214.
महरौली
लौह
 ंभ
 कसने
अपने
शासनकाल
म
बनवाया
था?
TTA
:
10
Seconds
C)
गडा D)
बाघ
A)
 गु B)
समु गु
204.
 ाचीन
भारतीय
दशन
के
अनुसार,
पु षाथ
या
जीवन
के
चार
उ 
म C)
च गु 
II D)
कुमारगु -
I
शा मल
नह 
ह:
TTA
:
20
Seconds 215.
जैन
क त
 
( च ौड़गढ़)
 कस
जैन
तीथकर
को
सम पत
ह?
A)
काम B)
मो TTA
:
15
Seconds
A)
महावीर
 ामी B)
पशावानाथ
C)
अथ D)
यश
C)
अ जतनाथ D)
आ दनाथ
205.
 बहार
के
 कस
शहर
को
बौ 
 ंथ
म मा
 नकाय
म
अंगु ारपा
के
 प
म
उ े खत
 कया
गया
ह? 216.
भारतीय-यूनानी
शासक 
के
बार
म
जानकारी
का
मु 
 ोत
 ा
ह?
TTA
:
20
Seconds TTA
:
13
Seconds
A)
बेगूसराय B)
दरभंगा A)
या ी
खाते B)
 स े
C)
सासाराम D)
बोध
गया C)
बौ 
सा ह D)
जैन
सा ह

206.
महाभारत
के
 कस
पव
से
भगवद
गीता
 नकाली
गई
ह? 217.
खजुराहो
 त
कंद रया
महादव
मं दर
 न ल खत
म
से
 कसके
 ारा
TTA
:
16
Seconds बनाया
गया
था?
A)
भी 
पव B)
शां त
पव TTA
:
14
Seconds
A)
 वधाधर B)
धंगदव
C)
सौ पटका
पव D)
आ द
पव
C)
 वजयपाल D)
जयवमन
207.
बुराइय 
को
समा 
करने
के
 लए
वै दक
दवता
इं 
को
__________
के
प
म
जाना
जाता
था। 218.
जैन
धम
म,
तीन
र 
( र )
 दए
गए
ह
और
उ 
 नवाण
का
माग
कहा
TTA
:
18
Seconds जाता
ह।
वे
 ा
ह?
A)
अ B)
महान
यो ा TTA
:
24
Seconds
A)
सही
बोली,
सही
 ान
और
सही B)
सही
 व ास,
सही
 ान
और
सही
C)
 काश
और
श D)
वायु
आचरण वहार
208.
 न ल खत
म
से
कौन
बौ 
धम
म
 नवाण
क 
अवधारणा
का
सबसे C)
सही
 व ास,
सही
रा ा
और
सही D)
सही
 व ास,
सही
 ान
और
सही
अ ा
वणन
करता
ह? आचरण आचरण
TTA
:
23
Seconds
219.
गु 
काल
के
दौरान,
सोने
के
 स 
को
 न ल खत
म
से
 कस
नाम
से
A)
कामना
क 
लौ
का
 वलु 
होना B)
 यं
का
पूण
अंत
ना मत
 कया
गया
था?
C)

क ाण
और
 व ाम
क 
अव ा D)
सभी
समझ
से
पर
एक
मान सक
TTA
:
12
Seconds
अव ा
A)
 पक B)
टंक
209.
 न ल खत
म
से
 कस
 ल
से
हड़ ा
काल
के
आरंभ
म
 कलेबंदी
के C)
 ाम D)
दीनार
माण
 मले
ह?
TTA
:
18
Seconds 220.
बौ 
धम
के
हीनयान
सं दाय
के
संबंध
म
 न ल खत
म
से
कौन-सा
A)
बनवाली B)
रोपर वक 
सही
ह?
TTA
:
27
Seconds
C)
लोथल D)
अमरी
A)
इस
सं दाय
को
बड़
वाहन
के
 प
म B)
वे
बु 
क 
मू त
पूजा
म
 व ास
210.
दवताओं
क 
 ु त
करने
के
 लए,
वै दक
भजन 
के
भाग
को
_________ भी
जाना
जाता
ह करते
ह
कहा
जाता
ह। C)
हीनयान
सं दाय
म
बो धस 
क D)
हीनयान
 व ान 
ने
जनसाधारण
के
TTA
:
18
Seconds कोई
अवधारणा
नह 
ह साथ
बातचीत
करने
के
 लए
 ाकृत
A)
सं हता B)
वेदांत भाषा
का
उपयोग
 कया
था
C)
 ा ण D)
सां 221.
इनम
से
 कस
शासक
का
उ ेख
स ोकोट् स
के
 प
म
 कया
गया
था?
211.
मौय
सा ा 
के
पतन
के
बाद
द ण
 ावणकोर
के
कुछ
 ह 
के
साथ TTA
:
14
Seconds
म ुर
और
 त ेवे ी
 जल 
पर
 कसने
क ा
 कया?
https://amzn.to/3HVqlZK ाचीन
इ तहास

A)
अशोक B)
चं गु 
मौय TTA
:
29
Seconds


C)
 ब ुसार D)
कुणाल A)
यम,
इं ,
व ण
और
कुबेर B)
इं ,
व ण,
 
और
कुबेर
C)
इं ,
व ण,
यम
और
 ा D)
यम,
 शव,
कुबेर
और
इं
222.
 सल ा दकारम
’एक
त मल
महाका 
ह
 जसे
______
 ारा
 लखा
गया
था। 234.
ध ंतरी
भारत
म
 कस
शासक
के
नवर 
(नौ
र )
म
से
एक
थे?
TTA
:
11
Seconds TTA
:
15
Seconds
A)
एववाइयर B)
 थ व ुअर A)
बाबर B)
च गु 
II
C)
स ानर D)
इलंगो
आ दगल C)
कृ दवराय D)
अकबर
223.
कामंदक
के
नी तसार
का
योगदान
 कस
 वषय
म
ह? 235.
 थम
जैन
सभा
का
आयोजन
कहाँ
 कया
गया
था?
TTA
:
19
Seconds TTA
:
12
Seconds
A)
तकशा 
और
दशन B)
ग णत A)
पाट लपु B)
वैशाली
C)
राजनी तक
नै तकता D)
 ाकरण C)
राजगृह D)
व वी
224.
जैन
धम
के
संबंध
म,
 न ल खत
म
से
कौन
सा
श 
"अचौय"
को 236.
 वनय
 पटक
एक
पु क
से
संबं धत
ह:
संद भत
करता
ह? TTA
:
12
Seconds
TTA
:
17
Seconds A)
सं ृ त
 ाकरण B)
महावीर
के
उपदश
A)
स B)
बेघर C)
जो 
के
 वचार D)
बौ 
संघ
के
 नयम
C)
अ ेय D)
अ ववा हत
जीवन 237.
सं ध व ह,
गु कालीन
 शलालेख 
म
उ े खत
एक
उ 
अ धकारी,
का
225.
 न ल खत
म
से
राज ान
के
 कस
रा 
का
अ धकतम
भाग
वधना उपयोग
 कया
गया
था
रा 
के
राजा
 ारा
 ा 
 कया
गया
ह
?
 TTA
:
21
Seconds
TTA
:
20
Seconds A)एक
 वदश
मं ी B) ांतीय
रा पाल
A)
यशोवधन B)
 भाकर
वधन C)सेना
के
कमांडर-इन-चीफ D)इनमे
से
कोई
नह
C)
महासेन
गु ा D)
हषवधन 238.
अशोक
के
आ मण
के
दौरान
क लग
(पूव 
 ांत)
क 
राजधानी
कौन-सी
226.
 न ल खत
म
से
कौन
सा
मं दर
चालु 
सा ा 
 ारा
 न मतनह 
ह? थी?
TTA
:
14
Seconds TTA
:
17
Seconds
A)
लाद
खान
मं दर B)
 चीम ीगुड़ी
मं दर A)
त शला B)
पाटलीपु
C)
मेगुती
का
जैन
मं दर D)
कृ े र
मं दर C)
उ ैन D)
तोशाली
227.
 ाचीन
भारत
म
 न ा
 ा
था? 239.
इ तहास
के
 ाचीन
और
म युगीन
काल
के
राजाओं
म
से
कौन
सा
जोड़ा
TTA
:
22
Seconds और
उनके
 ारा
 लखी
गई
रचनाओं
का
सही
 मलान
 कया
गया
ह?
A)
मौय 
का
 ण
का
 स ा B)
गु 
का
चाँदी
का
 स ा TTA
:
25
Seconds
C)
कुषाण
का
सोने
का
 स ा D)
हष
का
चाँदी
का
 स ा A)
कृ दवराय
-
समरांगणसु धरा B)
मह वमन
-
मटा वलसा
 हसन
C)
भोजदव
-
मानसोलासा D)
सोमे रा
-
अमुकतामलीदा
228.
उपसा स
एक
पाठ
 कस
से
संबं धत
ह?
TTA
:
16
Seconds 240.
चीनी
बौ 
तीथया ी
ज़ुआन
ज़ांग
______
के
दरबार
म
आया
था।
A)
शैव B)
वै व TTA
:
16
Seconds
C)
जैन
धम D)
बु 
धम A)
चं गु 
मौय B)
च गु 
 तीय
C)
हषवधन D)
समु गु
229.
 न ल खत
म
से
कौन
बौ 
सा ह क
 ंथ 
से
संबं धत
ह?
1.
 म लदा
प ा 241.
का लदास
के
कुमारसंभवम्
म
 कस
पौरा णक
च र 
के
ज 
क 
कहानी
2.
अ भधममो व णत
ह?
3.
उवस हरं
 ो TTA
:
19
Seconds
4.
महाव ा A)
सन ु मार B)
का तकेय
5.
समसयारा C)
 ु D)
अ भम ु
TTA
:
26
Seconds
A)
केवल
2,
3
और
5 B)केवल
1,
2
और
4 242.
जैन
धम
म
 श ा
का
अं तम
ल 
ह
C)केवल
1,
3
और
4 D)1,
2,
3,
4
और
5 TTA
:
12
Seconds
A)
दान
पु B)
दया
और
 ाग
230.
 न ल खत
म
से
 चनाब
का
ऋ ै दक
कालीन
नाम
 ा
ह? C)
मु D)
अ हसा
TTA
:
15
Seconds
A)
 वत ता B)
प ष ण 243.
क 
वंश
क 
राजधानी
____
थी।
C)
 वपश D)
अ नी TTA
:
13
Seconds
A)
पूहर B)राजगीर
231.
कुतुब
मीनार
के
पास
 त
महरौली
लौह
 ंभ
 कसक 
अव ध
के
अंतगत C)पाट लपु D)त शला
आता
ह:
TTA
:
17
Seconds 244.
उस
स ाट
का
नाम
बताइए
 जसने
बेसनगर
को
शुंग
वंश
क 
राजधानी
के
A)
चं गु 
 व मा द B)
चं गु 
मौय प
म
 ा पत
 कया।
C)
समु गु D)
अशोक TTA
:
20
Seconds
A)
वसु म ा B)
दवभुटी
232.
मौय
सा ा 
के
नगर
 शासन
म,
शु ाका 
श 
 न ल खत
म
से C)
भगभा ा D)
पु ल क
कससे
संबं धत
ह?
TTA
:
20
Seconds 245.
वै दक
स ता
________
नदी
के
साथ
 वक सत
 ई।
A)
वा ण 
अधी क B)
बाजार
के
अधी क TTA
:
11
Seconds
C)
टोल 
के
सं ाहक D)
खान
अधी क A)
 स ु B)
यमुना
C)
सर ती D)
गंगा
233.
इन
भगवान 
म
से
 कनको
लोकपाल
या
 व 
के
संर क
के
 प
म
भी
जाना
जाता
ह? 246.
पाँचव 
शता ी
म
भारत
आने
वाले
चीनी
तीथया ी
का
नाम
बताइए।
https://amzn.to/3HVqlZK ाचीन
इ तहास

TTA
:
15
Seconds 249.
 न ल खत
म
से
कौन
महायान
बौ 
धम
और
हीनयान
बौ 
धम
के
बीच


A)
इ ग B)
 न
सांग मूलभूत
अंतर
का
 त न ध 
करता
ह?
C)
फा ान D)
इनम
से
कोई
नह TTA
:
20
Seconds
A)अ हसा
पर
जोर B)जा त वहीन
समाज
247.
 न ल खत
म
से
कौन
तीसर
जैन
तीथकर
थे?
TTA
:
16
Seconds C)
दवी-दवताओं
क 
मू त
क 
पूजा D) ूप
क 
पूजा
A)
ऋषभनाथ B)
अ जतनाथ
करना
C)
संभवनाथ D)
सु मतनाथ
248.
मेग नीज
के
अनुसार
मौय
काल
म
भारतीय
समाज
म
 न 
म
से
कौन-
सी
जा त
अनुप त
थी?
TTA
:
17
Seconds
A)
दाश नक B)
दास
C)
यो ा D)
 कसान
To Practice 3751 More Questions
From Remaining Chapters

Buy NOW

Available at
&

Unique Features of SmartBook


Time To Answer (TTA)

Smart Answer Key

Level - Wise Difficulty

Best 4000 Questions

Free Video Lessons


FREE CHAPTER
CHAPTER:
1
Scan
QR
code
for

Free
Video
Lessons
ाचीन
इ तहास on
this
Chapter.

Correct Correct Correct Correct Correct Correct Correct


Q. Ans Q. Ans Q. Ans Q. Ans Q. Ans Q. Ans Q. Ans
Skipped Skipped Skipped Skipped Skipped Skipped Skipped
80% 53% 27% 27
% 14
% 95% 1%
1 D 29 C 57 D 85 D 113 A 141 A 169 C
11
% 28
% 20% 16% 58% 1% 17%
33% 2% 91% 17
% 13
% 4% 2%
2 B 30 C 58 C 86 C 114 D 142 A 170 B
7
% 22
% 2% 33% 16% 29% 56%
24% 5% 29% 8
% 2
% 97% 79%
3 D 31 C 59 C 87 B 115 C 143 B 171 B
40
% 62
% 48% 56% 44% 2% 2%
74% 61% 89% 3
% 30
% 34% 65%
4 C 32 B 60 A 88 A 116 B 144 D 172 A
11
% 12
% 1% 36% 23% 26% 15%
13% 23% 26% 20
% 45
% 10% 2%
5 B 33 B 61 A 89 A 117 A 145 C 173 D
35
% 61
% 46% 27% 24% 44% 49%
40% 54% 83% 1
% 51
% 64% 12%
6 A 34 B 62 C 90 D 118 A 146 C 174 C
5
% 42
% 4% 44% 28% 17% 29%
10% 27% 39% 19
% 10
% 15% 8%
7 A 35 D 63 D 91 D 119 B 147 C 175 C
33
% 4
% 7% 10% 19% 44% 12%
56% 87% 64% 12
% 60
% 85% 18%
8 B 36 B 64 A 92 B 120 A 148 B 176 D
16
% 10
% 20% 42% 9% 4% 23%
57% 46% 85% 2
% 2
% 25% 2%
9 B 37 B 65 A 93 C 121 B 149 B 177 D
34
% 12
% 4% 44% 24% 42% 55%
22% 45% 47% 27
% 85
% 9% 2%
10 A 38 C 66 D 94 A 122 A 150 C 178 C
32
% 4
% 15% 44% 7% 13% 33%
43% 46% 78% 2
% 60
% 6% 31%
11 B 39 A 67 C 95 C 123 D 151 C 179 C
33
% 5
% 14% 75% 9% 17% 30%
12% 12% 18% 27
% 53
% 7% 15%
12 C 40 B 68 A 96 A 124 B 152 D 180 A
26
% 32
% 10% 59% 4% 65% 18%
3% 46% 8% 4
% 50
% 11% 1%
13 C 41 C 69 A 97 B 125 A 153 D 181 C
61
% 22
% 14% 8% 6% 71% 18%
4% 9% 22% 5
% 25
% 23% 8%
14 B 42 B 70 D 98 C 126 C 154 C 182 A
43
% 44
% 11% 61% 12% 36% 57%
32% 71% 30% 26
% 79
% 87% 22%
15 C 43 B 71 B 99 D 127 C 155 D 183 B
31
% 5
% 41% 6% 9% 3% 35%
11% 78% 6% 6
% 89
% 9% 3%
16 B 44 C 72 B 100 A 128 B 156 C 184 B
52
% 2
% 23% 38% 9% 35% 38%
6% 54% 14% 38
% 41
% 26% 84%
17 B 45 D 73 D 101 B 129 C 157 C 185 B
50
% 10
% 27% 33% 30% 63% 5%
46% 8% 89% 22
% 58
% 40% 33%
18 B 46 B 74 B 102 D 130 C 158 C 186 B
28
% 72
% 4% 53% 8% 24% 24%
10% 28% 69% 3
% 3
% 79% 5%
19 B 47 D 75 D 103 C 131 A 159 B 187 A
42
% 26
% 2% 36% 43% 6% 12%
70% 88% 9% 3
% 3
% 58% 7%
20 B 48 D 76 B 104 C 132 C 160 B 188 B
5
% 4
% 41% 22% 41% 8% 47%
85% 71% 83% 7
% 30
% 22% 7%
21 D 49 B 77 D 105 D 133 A 161 C 189 C
5
% 5
% 9% 26% 29% 11% 22%
15% 11% 65% 21
% 28
% 12% 9%
22 C 50 D 78 A 106 D 134 A 162 C 190 D
26
% 17
% 20% 16% 26% 28% 23%
15% 9% 35% 19
% 81
% 62% 67%
23 B 51 A 79 C 107 C 135 A 163 C 191 C
18
% 68
% 13% 53% 12% 29% 3%
7% 85% 6% 79
% 10
% 10% 14%
24 D 52 D 80 C 108 D 136 B 164 B 192 B
31
% 6
% 60% 4% 63% 38% 31%
9% 89% 34% 50
% 68
% 4% 39%
25 C 53 A 81 A 109 C 137 B 165 C 193 D
45
% 2
% 21% 20% 14% 83% 34%
39% 84% 28% 14
% 6
% 73% 41%
26 C 54 C 82 B 110 A 138 C 166 C 194 A
30
% 2
% 25% 25% 19% 20% 31%
76% 73% 12% 67
% 46
% 42% 27%
27 A 55 A 83 B 111 B 139 A 167 C 195 A
12
% 8
% 33% 24% 45% 27% 14%
91% 48% 22% 4
% 32
% 3% 12%
28 A 56 C 84 B 112 C 140 B 168 B 196 A
6
% 24
% 12% 36% 7% 11% 37%
Correct Correct Correct Correct Correct Correct Correct
Q. Ans Q. Ans Q. Ans Q. Ans Q. Ans Q. Ans Q. Ans
Skipped Skipped Skipped Skipped Skipped Skipped Skipped
85% 57%
197 D 225 B
4
% 8
%
4% 51%
198 D 226 D
16
% 36
%
1% 87%
199 C 227 A
54
% 8
%
1% 4%
200 D 228 C
60
% 66
%
58% 4%
201 A 229 B
16
% 63
%
70% 15%
202 D 230 D
14
% 27
%
73% 50%
203 A 231 A
6
% 18
%
38% 8%
204 D 232 C
22
% 64
%
13% 80%
205 A 233 A
8
% 8
%
11% 43%
206 A 234 B
50
% 14
%
42% 1%
207 B 235 A
16
% 27
%
1% 44%
208 A 236 D
31
% 13
%
29% 30%
209 A 237 A
36
% 51
%
69% 84%
210 A 238 D
6
% 1
%
15% 16%
211 D 239 B
40
% 70
%
11% 3%
212 C 240 C
43
% 29
%
8% 5%
213 A 241 B
52
% 60
%
24% 12%
214 C 242 C
31
% 22
%
4% 12%
215 D 243 C
35
% 46
%
79% 14%
216 B 244 C
6
% 26
%
44% 62%
217 B 245 C
13
% 3
%
8% 5%
218 D 246 C
19
% 20
%
34% 43%
219 D 247 C
17
% 36
%
27% 67%
220 C 248 B
19
% 10
%
39% 24%
221 B 249 C
29
% 33
%
25%
222 D
58
%
97%
223 C
2
%
24%
224 C
41
%
https://amzn.to/3HVqlZK ाचीन
इ तहास

LEVEL
1 1
-
86
Questions

Sol
1. Sol
5.
सही
उ र
 वक 
2)
यानी
 ास
ह।
यजुर
वेद
-
इसम
य 
या
य 
अनु ान
करते
समय
कह
जाने
वाले
मं आधु नक-समय
का
नाम ऋ ै दक
नाम
शा मल
ह।
इसम
पूजा
अनु ान
के
 लए
भजन
शा मल
ह।
साम
वेद
-
यह
धुन 
और
मं 
का
वेद
ह।
इसम
 ु नया
के
सबसे
पुराने
मं चनाब अ नी
और
अनु ान
संबंधी
धुन
शा मल
ह। रावी पु शनी
अथववेद
-
तनाव,
 चता
और
अ 
सम ाओं
को
 ू र
करने
के
 लए
जा ुई
झेलम वता ा
सू 
शा मल
ह।
हालां क
इसम
द नक
जीवन
के
 लए
गैर
जा ुई
नु 
े भी
शा मल
ह। ास वपास
ऋ ेद
-
सबसे
पुराना
 ात
वेद।
यह
दवताओं
क 
 शंसा,
 ांड
 व ान,
दाश नक
और
का नक
 
जैस
े वषय 
से
संबं धत
ह। Sol
6.
सही
उ र
लु नी
ह।
Sol
2. Key
Points
मोहन
जोदड़ो
पा क ान
के
 सध
 ांत
के
लरकाना
 जले
म
 सधु
नदी
के
तट
पर
 त
ह। भगवान
बु ,
 स ाथ
गौतम,
का
ज 
623
ई.पू.
लु नी
के
 स 
उ ान
Key
Points म
 आ
था,
जो
ज 
ही
एक
तीथ ल
बन
गया।
तीथया य 
म
भारतीय
स ाट
अशोक
भी
थे,
 ज ने
वहां
अपना
एक
 ारक
यह
1922
म
 ी
आर
डी
बनज 
 ारा
खोजा
गया
था
और
इसे
1980
म ंभ
बनवाया
था।
यूने ो
के
 व 
 वरासत
 ल
के
 प
म
ना मत
 कया
गया
था।
यह
 ाचीन
समय
म
 ु नया
क 
सबसे
पुरानी
ब य 
म
से
एक
ह
और
यह Additional
Information
सधु
घाटी
स ता
म
ब ी
का
सबसे
बड़ा
 ल
ह।
एक
 वशाल
लकड़ी
के
अ धरचना
के
साथ
एक
बड़ी
इमारत
जो
 ेट
 ैनरी भारत
म
बौ 
धम
(अनाज
भंडारण
बेस)
लगती
थी,
सर
मो टमर
 ीलर
 ारा
खोजा
गया
था भारत
म
बौ 
धम
क 
शु आत
2,600
वष
पहले
एक
जीवन-
और
साथ
ही
बड़
सावज नक
 ान
को
भी
पास
म
खोजा
गया
था
और प त
के
 प
म
 ई
थी
 जसम
 
को
बदलने
क 
 मता
थी।
बृहत ानागार
कहा
जाता
था। यह
द ण
और
द ण-पूव 
ए शयाई
दश 
के
मह पूण
धम 
म
से
एक
ह।
Sol
3. धम
अपने
सं ापक
 स ाथ
गौतम
क 
 श ाओं,
जीवन
के
अनुभव
सही
उ र
कालीदास
ह। पर
आधा रत
ह।
Key
Points उनका
ज 
शा 
वंश
के
शाही
प रवार
म
 आ
था,
जो
लुं बनी
म
क पलव 
ु से
शासन
करते
थे,
जो
भारत-नेपाल
सीमा
के
पास
का लदास
 ाचीन
भारत
के
महान
शा ीय
सं ृ त
क वय 
और
नाटककार त
ह।
म
से
एक
ह। 29
वष
क 
आयु
म,
गौतम
ने
घर
छोड़
 दया
और
धन
के
अपने
उनके
लेखन
से
पता
चलता
ह
 क
वह
उ ैन
के
एक
प व 
 ा ण
थे,
और जीवन
को
अ ीकार
कर
 दया
और
जीवनशैली
या
चरम
आ -
उ ने
 ा णवादी
सीखने
क 
 व भ 
शाखाओ
ं का
 ान
 ा 
 कया
था। अनुशासन
को
अपनाया।
उनक 
का 
 तभा
उनके
सभी
काय 
म
ब त
अ ी
तरह
से
प रल त
होती लगातार
49
 दन 
के
 ान
के
बाद,
गौतम
ने
 बहार
के
एक
गांव
ह,
जो
माल वका म ,
 व मोवशीय,
अ भ ान-सकुंतला,
और
रघुवंश, बोधगया
म
एक
पीपल
के
पेड़
के
नीचे
बो ध
( ानोदय)
 ा
कुमारसंभव
और
मेघ ू त
जैसी
क वताएँ 
ह। कया।
बु 
ने
अपना
पहला
उपदश
यूपी
के
बनारस
शहर
के
पास
सारनाथ
Important
Points गाँव
म
 दया
था।
इस
घटना
को
धम-च -पालन
(कानून
का
प हया
कृ 
 पायन,
 जसे
भासा
और
वेद
भासा
के
नाम
से
भी
जाना
जाता
ह।
मोड़ना)
के
 प
म
जाना
जाता
ह।
भासा उ 
पारंप रक
 प
से
महाभारत
के
लेखक
के
 प
म
जाना
जाता
ह।
उनक 
मृ 
ु 803
वष
क 
आयु
म
483
ईसा
पूव
म
यूपी
के
एक
उ 
वेद 
और
पुराण 
का
 त प
भी
माना
जाता
ह।
क 
े कुशीनगर
नामक
 ान
पर
 ई
थी।
घटना
को
महाप र न न
के
 प
म
जाना
जाता
ह।
चार
महान
स :
शु का
एक
सव े 
 च कूट
के
 लए
जाना
जाता
ह
 जसका
शीषक
दशकुमारच रत
(दस
राजकुमार 
का
रोमांच)
ह। ु ख
( ुःख)
संसार
का
सार
ह।
शू क हर
 ुख
का
एक
कारण
होता
ह
-
समुदय।
उनके
 ारा
तीन
सं ृ त
नाटक
मृ क टका
(छोटी
 म ी
क 
गाड़ी),
वनवासवद ,
और
एक
भान,
प भृतक
ह। पी ड़त
को
बुझाया
जा
सकता
था
-
 नद ष।
इसे
अथांग
म ा
(आठ-गुना
पथ)
का
पालन
करके
 ा 
 कया
जा
सकता
ह।
वह
ग णत 
और
खगोलशा ी
के
 प
म
 स 
ह।
उनके
काम 
म
कई
तरह
के
 वषय
शा मल
ह,
जैसे
 क
वगमूल Sol
7.
आयभ सही
उ र मेग नीज ह।
नकालना,
 घात
समीकरण 
को
हल
करना
और
 हण
क
भ व वाणी
करना। Key
Points

मेग नीज
एक
 ीक
या ी
और
भूगोलवे ा
था,
 जसके
बाद
के
 ीक लेखक


Sol
4. मु 
 प
से
भारत
के
 लए
ऋणी
थे।
यूनानी राजा
से ूकस
 थम
ने
उ 
भारत
म
राजा
चं गु 
मौय
के
दरबार
सु ुत
सं हता
 च क ा
और
श 
 च क ा
पर
एक
 ाचीन
सं ृ त
 ंथ म
राज ू त
के
 प
म
भेजा।
ह। भारत
क 
सं ृ त,
इ तहास
और
धम
के
बार
म
उनका लेखन भारत
के
बार
म
सु ुत
का
संकलन
आयुवद
के
मूल
 ंथ 
म
से
एक
माना
जाता
ह। प मी
 ान
का
आधार
था।
यह
भारत
म
 ाचीन
स ता
से
बचे
 च क ा
 े 
के
दो
मूल
 ह ू 
 ंथ 
म
से
एक
माना
जाता
ह। Sol
8.

वै दक
धम
को
 ा णवाद
के
 प
म
भी
जाना
जाता
था।


https://amzn.to/3HVqlZK ाचीन
इ तहास

यह
 ह ू 
धम
का
एक
 प
ह
जो
वेद 
 ारा
 ुत
क 
गई
र ी
 वचारधाराओं अबू
रहान
अल-ब नी
एक
ईरानी
 व ान
थे।
और
पौरा णक
 
के
इद- गद
घूमता
ह। उ 
 व भ 
नाम 
से
जाना
जाता
ह:
-
बौ 
धम
क 
 ापना
 स ाथ
गौतम
ने
क 
थी। भारत- व ा
के
जनक
जैन
धम
क 
 ापना
महावीर
ने
क 
थी। तुलना क
धम
के
जनक
हडो न 
क 
 ापना
अ र पस
 ारा
क 
गई
थी।यह
एक
ऐसा
दशन
ह आधु नक
भूग णत
के
जनक
जो
मानता
ह
 क
सुख
और
दद
ही
एक
ऐसी
चीज
ह
जो
इं सान
क 
शारी रक पहला
मानव व ानी
भलाई
के
 लए
मायने
रखती
ह। उ ने
 कताब-उल- हद
पु क
क 
रचना
क ।
अलब नी
(अबू
र म
ब नी)
एक
फारसी
 व ान
था
जो
1017
म
गजनी
Sol
9. के
महमूद
के
साथ
भारत
आया
था।
हड़ ा: उ ने
भारत
 व ा,
 ह ू 
धा मक
मा ताओं,
री त- रवाज 
और
सामा जक
संगठन
पर
 ट णी
क ।
1921
म,
दया
राम
साहनी
ने
 टश
भारत
के
पंजाब
 ांत
के
म टगोमरी
 जले
म
हड़ ा
क 
स ता
का
खुलासा
 कया। Additional
Information
हड़ ा
के
पुराता क
 न ष
एक
पं 
म
अ 
भंडार
थे,
मातृ
दवी
क
म ी
क 
मू त,
तांबे
के
पैमाने,
 लग
के
प र
के
 च ,
आ द। अल-मसुदी
क 
पु क-
द
मीडोज
ऑफ
गो
सुलेमान
क 
पु क
-
भारत
और
चीन
के
 ाचीन
लेख
Sol
10. फरदौसी
क 
पु क
-
शाहनामे
सही
उ र समु गु 
ह।
Sol
15.
चं गु 
 थम
को
उसके
पु 
समु गु   ारा
 ानांत रत
 कया
गया
था।
रावी
नदी
का
वै दक
नाम
पा शनी
था।
Key
Points ऋ ेद
म
स - सधु
के
नाम
से
जानी
जाने
वाली
7
न दय 
के
समूह
का
उ ेख
ह।
समु गु 
(335
ई 
-
380
ई ): पांच
न दय 
के
साथ
सर ती
नदी
उन
7
न दय 
म
से
6
नदी
बनाती
ह,
समु गु 
गु 
राजवंश
का
सबसे
महान
राजा
था। हालां क
7व 
क 
अभी
भी
पु 
नह 
 ई
ह।
उसके
शासनकाल
का
सबसे
 व ृत
और
 ामा णक
अ भलेख यमुना
को
का लदी
भी
कहा
जाता
ह
जो
भारतीय
रा 
प म
बंगाल
म
याग
 श /इलाहाबाद
 ंभ
 शलालेख
म
संर त
ह,
जो सुंदरवन
के
आसपास
क 
एक
नदी
ह।
उसके
दरबारी
क व
ह रसेन
 ारा
र चत
ह।
समु गु 
के
सै 
अ भयान
को
वी.ए.
 थ
 ल खत
भारत
के Sol
16.
नेपो लयन
के
 प
म
उसके
वणन
को
 ायसंगत
मानते
ह।
शीषक:
क वराज,
परम
भागवत,
अ मेध प र मा,
 व म
सव- सधु
घाटी
क 
स ता
भारतीय
उपमहा ीप
म
 ोटो-ऐ तहा सक
चरण
का
राजो े ा केवल
गु 
शासक
के
पास
सव-राज-ओ ेता
क एकमा 
उदाहरण
ह।
उपा ध
थी। हम
इस
चरण
को
 ोटो-ऐ तहा सक
मानते
ह
 क
आधु नक
इ तहासकार
इलाहाबाद
 ंभ
के
 शलालेख 
म
शीषक
धम
 चारक
बंध
ु का ने
उस
अव ध
क 
 ल पय 
क 
खोज
क 
ह,
ले कन
उ ने
इसे
 ड 
नह
उ ेख
ह,
अथात
वह
 ा णवादी
धम
के
 वतक
था। कया
ह।

Sol
11. म
सं ा चरण लप ड 
या
नह


सही
उ र
सु का
ह।
Key
Points 1 पूव
ऐ तहा सक कोई
 ल प
नह ड 
नह
मृ क टका
( म ी
क 
गाड़ी): 2 ोटो
ऐ तहा सक ल प
मौजूद ड 
नह

यह
सु का
(248
A.D.)
 ारा
 लखा
गया
ह। 3 ऐ तहा सक ल प
मौजूद ड


यह
गंभीर
वा वकता
के
 श
के
साथ
एक
उ ेखनीय
सामा जक
नाटक
ुत
करता
ह। Sol
17.
पा 
को
समाज
के
सभी
 र 
से
 लया
गया
ह,
 जसम
चोर
और
जुआरी, सही
उ र
पा णनी
ह।
बदमाश
और
आलसी
 ,
दरबारी
और
उनके
सहयोगी,
पु लस
कां ेबल, Key
Points
भ क ु 
और
राजनेता
शा मल
होते
ह।
अ ा ायी,
सं ृ त
अवधी
("आठ
अ ाय"),
भारतीय
 ाकरण
पा णनी
Sol
12. ारा
छठी
से
पांचव 
शता ी
ईसा
पूव
म
 लखा
गया
एक
सं ृ त
 ाकरण
सही
उ र
 वक 
3
ह,
अथात
 व ाम । ंथ।
शा ीय
सं ृ त
के
 लए
भाषाई
मानक
इस
काय
 ारा
 नधा रत
 कए
गए
थे।
वै दक
धम
म
 वक सत
 कए
गए
 ा कता
और
 ाकरण
के
गाय ी
मं 
सूय
को
संबो धत
आ ा क
उ ेजना
के
 लए
एक
 ाथना
ह। व ान
को
4,000
सू 
म
अ भ 
 कया
गया
था।
उनका
काम
पा ण न
इसे
 व ा म 
ने
बनाया
था। ारा
आठ
अ ाय 
म
 वभा जत
 कया
गया
था,
 जनम
से
 ेक
को
आगे
यह
मं 
ऋ ेद
म
तीसर
मंडला
से
 लया
गया
ह। चार
अ ाय 
म
 वभा जत
 कया
गया
ह।

Sol
13. Sol
18.
सही
उ र
अशोक
ह।
Important
Points कठोप नषद
म
न चकेता
का
उ ेख
ह।
249
ईसा
पूव
म
मौय
स ाट
अशोक
 ारा
 न मत
 ंभ
पर
 शलालेख
इस
बात
क काठोप नषद
यम
(मृ ु
के
 ामी)
और
न चकेता
(12
साल
के
युवा
लड़के)
गवाही
दता
ह
 क
भगवान
बु 
का
ज 
623
ईसा
पूव
म
द णी
नेपाल
के
तराई के
बीच
क 
बातचीत
ह।
मैदान 
म
 त
लु नी
के
प व 
 े 
म
 आ
था। न चकेता
मृ 
ु और
उससे
बाद
के
जीवन
का
अथ
तलाश
करने
के
 लए
म
घर
छोड़कर
चला
गया।
लु नी
म
ईसा
पूव
तीसरी
शता ी
से
बौ 
तीथ ल 
क 
 कृ त
के
बार
म उप नषद
 ाचीन
सं ृ त
 ंथ
ह
 जनम
 ह ू 
धम
क 
कुछ
क ीय
दाश नक
मह पूण
 माण
ह। अवधारणाएं 
और
 वचार
ह,
 जनम
से
कुछ
को
बौ 
धम
और
जैन
धम
जैसी
लु नी
नेपाल
के
क पलव ु
 जले
म
 त
ह। बु 
का
ज 
लुं बनी
म
 आ धा मक
परंपराओं
के
साथ
साझा
 कया
गया
ह।
था। कथा
उप नषद
 ाथ मक
उप नषद 
म
से
एक
ह
जो
कृ 
यजुवद
के
कथा
व ालय
के
अं तम
आठ
खंड 
म
स हत
था।
Sol
14.
सही
उ र
अल-ब नी
ह। Sol
19.
Key
Points
https://amzn.to/3HVqlZK ाचीन
इ तहास

भारत
म
वै दक
स ता
सर ती
नदी
के
 कनार
 वक सत
 ई। अ धकांश
ऋ ेद
स ताएं 
सर ती
के
पास
 त
थ ।
ऋ ेद
क 
पु क
6
म
'ना द ु त
सू '
नाम
का
एक
भजन
शा मल
ह, ऋ ेद
म
काबुल,
 ात,
खुरम,
गुमल,
 सधु,
झेलम,
 चनाब,
र व,
 ास
और
जसमे सर ती
क 
 ु त
करने
के
 लए उ 
"आदश
माँ,
नायाब
नदी, सतलुज
न दय 
का
भी
उ ेख
ह।
सव 
दवी"
कहा
गया ह। ऋ ेद
के
नदी ु त
सू 
म
 शंसा
को
न दय 
क 
 शंसा
का
 ोत
कहा
ऋ ेद
म
एक
श शाली,
बफ ली
नदी
सर ती
का
उ ेख
ह,
 जसके जाता
ह।
कनार 
पर
सा ह 
क 
 ु 
क 
जानी
थी। न दय 
के
पुराने
नाम:
धमा भमानी
 ह ुओ
ं ारा
प व 
माना
जाने
वाली
इस
नदी
को  "पहाड़ 
से
समु 
तक
अपने
माग
म शु 
और
अ 
सभी
न दय 
क 
म हमा
से
पर" ऋग-वै दक
नाम आधु नक
नाम
व णत
 कया
गया
ह।
सधु सधु
सर ती
का
पता
लगाने
के
 यास 
को
शु 
म
2003
म
फा 
 क
पर
रखा
गया
था। वत ा झेलम
सर ती
ह रटज
 ोजे 
को
क ीय
पयटन
और
सं ृ त
मं ालय
 ारा अ नी चनाब
शु 
 कया
गया
था
ले कन
इसे
2005
म
रोक  दया
गया
था।
प ी रव
Sol
20. वपाशा ास
सही
उ र
 वक 
-
2
अथात्
मुद 
का
टीला
ह।
शतुद सतलुज
हड़ ा
क 
खोज
के
एक
साल
बाद,
मोहनजो-दड़ो
के
पुराता क
 ल
को
1922
म
मा ता
दी
गई
थी। Sol
24.
मोहनजो-दड़ो
का
 वकास
 ागै तहा सक
 सधु
सं ृ त
से
लगभग
3,000 सही
उ र
ह
अ ,
इं ,
सोम।
ईसा
पूव
 आ
था
और
यह
 ाचीन
 सधु
घाटी
स ता
के
सबसे
बड़
शहर 
म
से
एक
था। अ ,
इं 
और
सोम
 वक 
म
से
वै दक
काल
के
मह पूण
दवताओं
के
शहरी
 नयोजन
और
बेजोड़
 स वल
इं जी नय रग
ने
इसे
अपने
समय
का
सबसे सही
समूह
ह।
उ त
और
उ ेखनीय
प र ृ त
शहर
बना
 दया।
मोहनजो-दड़ो
का
अथ
ह
'मुद 
का
टीला'।
इस
 ान
के
आधु नक
नाम
क Important
Points
ा ा
 सधी
म
"मुद 
का
टीला"
के
 प
म
क 
गई
ह।
अ 
का
सं ृ त
श 
का
अथ
अ 
ह।
सधु
घाटी अ 
 ाचीन
भारत
क 
वै दक
पौरा णक
कथाओं
म
 ू सरा
सबसे
मह
ल मह पूण
दवता
ह।
यह
एक
नवीनतम
 ल
ह
जो
गुजरात
म
मौजूद
ह। 200
ऋ ै दक
भजन
अ 
को
सम पत
ह।
इसे
अ 
 ल ,
 जनम
2
भाग
थे,
के
 वपरीत
3
भाग 
म वै दक
 ह ू 
पौरा णक
कथाओं
म
इं 
दवताओ
ं के
राजा
ह।
धोलावीरा
वभा जत
 कया
गया
था। इं 
को
वषा
दवता
माना
जाता
था।
इसम
 नचले
शहर
के
अलावा
एक
म 
शहर
मौजूद
ह। इं 
को
कभी-कभी
"
हजार
आं ख 
वाला
"
कहा
जाता
ह।
इसम
एक
कृ म
 ट
गोदी
बाड़ा
ह। सोमा
एक
पौधा
पैदा
करने
वाला
श शाली
पेय
था
जो
वै दक
ब लदान 
का
इसम
चावल
क 
सबसे
शु आती
खेती
के
 माण
ह। एक
मौ लक
 ाव
था।
लोथल 
दवता
सोमा
"पौध 
का
राजा
"
था।
यह
 सधु
घाटी
के
लोग 
के
 लए
एक
बंदरगाह
के
 प
म
काय
करता
था।
Additional
Information
सधी
भाषा
म,
इसका
अथ
ह
"मुद 
का
टीला"।
यह
सभी
 सधु
शहर 
म
सबसे
बड़ा
ह। व 
ु एक
दवता
ह
जो
तीन
चरण 
म
पृ ी
को
ढकते
ह।
मोहनजो-
द
 ेट
बाथ
भी
यहां
मौजूद
ह। ग ड़
भगवान
 व ु
के
प ी
और
वहाण
(पवत)
ह।
दड़ो
कां 
क 
नृ 
करती
 ई
लड़क 
और
पशुप त
के
 प
क यम
मृ ु
के
दवता
ह।
एक
मुहर
भी
यहां
पाई
गई
ह।
ऋ े दक
काल
के
दौरान,
आ दवासी
समाज
को
तीन
समूह 
म
 वभा जत
कया
गया
था-
यो ा,
पुजारी
और
लोग।
Sol
21. सु 
नामक
चौथा
 वभाजन
ऋ े दक
काल
के
अंत
क 
ओर
 कट
सही
उ र
 वक 
4
अथात्
चार
वण
ह। आ
 क
इसका
उ ेख
ऋ ेद
क 
दसव 
पु क
म
पहली
बार
आ
ह,
जो
 क
नवीनतम
जोड़
ह।
वै दक
काल
म,
समाज
को
4
वग 
म
 वभा जत
 कया
गया
था
 ज 
वण
रग
वै दक
काल
म
पुजा रय 
को
उपहार
के
 प
म
 दए
गए
दास 
के
संदभ
ह।
कहा
जाता
ह।
वे
मु 
 प
से
घरलू
उ 
के
 लए
 नयो जत
म हला
दास
थ 
।
चार
वण
ह:
 ा ण,
 य,
वै 
और
शू ।
यह
 
ह
 क
ऋग
वै दक
काल
म
दास 
का
उपयोग
सीधे
कृ ष
या
धमशा 
ने
वण 
के
आदश
" वसाय "
के
बार
म
 नयम 
का
उ ेख
 कया अ 
उ ादक
ग त व धय 
म
नह 
 कया
जाता
था
।
ह।
ऋ ेद
के
युग
म
 वसाय
पर
आधा रत
भेदभाव
शु 
 कया
गया
था
ले कन
ा ण
-
वेद 
का
अ यन
करना
और
 श ा
दना,
ब लदान
करना
और
दान यह
 वभाजन
ब त
तेज
नह 
था
 क
इसम
 व भ 
प रवार 
के
संदभ
हएक
दना/ ा 
करना।
ही
प रवार
के
भीतर
अलग-अलग
 वसाय
 ए
।
य
-
यु 
म
संल 
होना,
लोग 
क 
र ा
करना
और
 ाय
का
 बंधन
ऋ े दक
काल
के
दौरान,
समाज
म
आ दवासी
त 
मजबूत
थे
और
कर 
के
करना,
वेद 
का
अ यन
करना,
ब लदान
दना,
और
उपहार
बनाना।
सं ह
या
भू म
क 
संप 
के
संचय
के
आधार
पर
सामा जक
 वभाजन
वै 
-
कृ ष,
पशुचारण
और
 ापार
म
संल 
होना। अनुप त
थे।
शू 
-
उ 
केवल
एक
काय
स पा
गया
था
अथात्
तीन 
"उ "
वण 
क 
सेवा
समाज
अभी
भी
आ दवासी
और
काफ 
हद
तक
समतावादी
था।
करना।
यह
 
 प
से
इं गत
करता
ह
 क
ऋ े दक
काल
म
सामंतवाद
Sol
22.
का
कोई
सबूत
नह 
था।
ब सार
(558-491
ई.पू.): Sol
25.
ब सार हयक वंश  से
संबं धत
छठी
शता ी
ईसा
पूव
म मगध
(543-492
सभा  ारं भक
ऋ ै दक
काल दोन 
 वधानसभाओं
को
दशाती
ह।
ईसा
पूव) महाजनपद
के
पहले
शासक थे।
म हलाएं 
भी
इस
सभा
म
शा मल
होती
थ 
और
उ  सभावती
कहा
जाता
था
ब सार
ने
कई
जनजा तय 
और
 े 
को
एक
साथ
लाकर
मगध
रा 
क
ऋ ेद
सभा
के
बार
म
 ा यक
और
 शास नक
काय 
के
साथ-साथ एक
ापना
क ।
नृ 
और
जुआ
सभा
के
 प
म
बोलता
ह।
Sol
23. जब क स म त एक
लोक
सभा
थी
 जसम
जनजा त
के
लोग
आ दवासी
सही
उ र
सर वसाय
के
 लए
एक त
होते
थे।
ती
ह
।
https://amzn.to/3HVqlZK ाचीन
इ तहास

Sol
26. बु 
वैशाली
गए
और
उ ने
सां 
दशन
क 
 श ा
 ा 
क ।


वह
राजगृह
गए
और
योग
 सखा।
वह
उ वेला
गए
जहाँ
उ ने
 बोधन
यजुवद सं ृ त
के
मं 
और
छं द 
का
एक
 ाचीन
सं ह
ह,
 जसका ा 
 कया।
उपयोग  ह ू 
पूजा
और
अनु ान  म
 कया
जाता
ह। वह
 फर
सारनाथ
गए
जहाँ
उ ने
अपना
पहला
धम पदश
 दया
जो
यह
नाम
सं ृ त
क 
जड़ 
से
 लया
गया
था,
यजुर,
 जसका
अथ
ह "पूजा"
या धमच वतन
के
नाम
से
भी
जाना
जाता
ह।
" ाग" और वेद,
 जसका
अथ " ान" ह। 483
ई.पू.
म
कुशीनारा
के
 नकट
उनक 
मृ 
ु ई
और
यह
घटना
यजुर
वेद
को
कभी-कभी "ब लदान
के  ान" के  प
म
अनुवा दत
 कया
जाता महाप र नवाण
के
 प
म
जानी
जाती
ह।
ह।
Sol
34.
Sol
27. सही
उ र
मेग नीज
ह।

ऋ ेद'
वेद 
म
सबसे
पुराना ह
और
 कसी
भी
इं डो-यूरोपीय
भाषा
म
सबसे इं डका
मौयकालीन
भारत
क 
एक
पु क
ह।


पुराने
 च लत
 ंथ 
म
से
एक
ह। यह
भारतीय
उपमहा ीप
म
मौय
 शासन
और
सै 
संगठन
के
बार
म
 ववरण
ऋ ेद
 ाचीन
भारतीय
 ंथ 
का
एक
समूह
ह
 जसम
वै दक
काल
से 1028 दान
करता
ह।
सं ृ त
 ो  और ऋ े दक
दवताओं
को
सम पत
10,600
 ोक ह। इसे
मेग नीज
ने
 लखा
था।
मेग नीज
से ूकस
 नकेटर
का
 ीक
राज ू त
था।
Sol
28. उ ने
चं गु 
मौय
के
शासनकाल
के
दौरान
भारत
का
दौरा
 कया।
'इं डका'
पु क
अब
लु 
हो
गई
ह
ले कन
बाद
के
लेखक 
से
पाए
गए
ऋ ेद
म
कुल 1028
भजन ह। सा ह क
अंश 
से
आं शक
 प
से
पुन न मत
क 
गई
ह।
ऋ ेद
सबसे पुराना
वेद ह। ॉ टश
शा ीय
दाश नक
जॉन
वाटसन
मैक डल
ने
1887
म
इं डका
का
यह
 ह ू 
धम
के
चार
 व हत
प व 
 ंथ 
म
से
एक
ह
 जसे
वेद 
के
 प
म
जाना एक
पुन न मत
सं रण
 का शत
 कया।
जाता
ह। बो
एक
यूनानी
दाश नक
था
जो
रोमन
सा ा 
म
रोमन
गणरा 
के
यह
 ंथ
1,028
भजन
और
10,600
 ोक 
का
एक
सं ह
ह,
 जसे
दस सं मण
काल
के
दौरान
ए शया
माइनर
म
रहता
था।
पु क 
म
मंडल
के
 प
म
 व त
 कया
गया
ह। जयो ा फका
 बो
 ारा
 ल खत
एक
 स 
पु क
ह।
नी
 ाचीन
रोम
का
एक
लेखक
था।
Sol
29. उ 
 नी
द
यंगर
के
नाम
से
जाना
जाता
ह
सू 
और
भगवद
गीता
के
साथ
उप नषद 
को
वेदांत
या
 डपाचस
के
 प
म
जाना
जाता
ह। Sol
35.

उप नषद 
को
वेद 
से
जुड़
 ह ू 
धम
म
 ु तय 
का
अं तम
 ोत
माना
जाता
ह। "जातक" बौ 
धम
के
लोग 
का एक
प व    ह।
उप नषद 
को
भारतीय
दशनशा 
का
मह पूण
 ोत
माना
जाता
ह
और
यह जातक नामक
सा ह क
  
म 500
से
अ धक
कथाएँ 
ह
और
पाली क
मूल
 प
से
 ा,
जीव
और
जगत
के
 ान
से
संबं धत
ह। ाचीन
भारतीय भाषा
म
 लखी
गई पं ह
 ंथ 
का
दसवां
  ह,  जसम
सु
पटक
का
खु क  नकाय ( पटक
का
 ू सरा
या
बौ 
पाली
  से
संबं धत
Sol
30.
ह)
बौ 
धम
के
सं दाय
हीनयान
के  स ांत
को
 तपा दत
करता
ह।
चौथे
बौ 
प रषद को व ागामनी
अभय और क न  के तहत  मश:  ीलंका
(77
ईसा
पूव)
और
क ीर
(127
सीई)
म
आयो जत दो
अलग-अलग प रषद 
म  वभा जत  कया Sol
36.
गया
ह,  जसके
प रणाम प बौ 
धम का
 वभाजन हीनयान
(थेरवाद
बौ )
और
दयोफर,
असम
म
 त
एक
पुराता क
 ल
ह।
महायान म
 आ। 
 
 
 
 
Sol
31. यह
पुरात 
 नदशालय,
असम
सरकार
के
अंतगत
संर त
पुरात 
पाक
और
च रत भारतीय स ाट हषवधन
क 
जीवनी ह,
 ज ने
606
से
647
ई ी
तक
उ र ल-सं हालय
ह।
भारत
पर
शासन
 कया
और वधन
वंश के
शासक
थे।
इसे बाणभ  ने
 लखा
था, दयोफर
पहाड़ी,
 दयोपानी
के
आर त
वन
का
 ह ा
ह।
जो सातव 
शता ी
ई ी
के
सं ृ त
लेखक
थे। दयोपानी
वन
भी
कृ म
भांग
के
पौध 
के
 लए
 स 
ह।
दयोफर
पहाड़ी
सातव 
शता ी
ई ी
क 
थी।
Sol
32.
Sol
37.
कोणाक
सूय
मं दर एक
13व 
शता ी
का
 ह ू 
मं दर
ह
जो उड़ीसा म
 त
ह गौतम
बु
और
सूय
दव
को
सम पत
ह।
मं दर
एक  वशाल
रथ
के
आकार का
ह,
मं दर
उ म प र
क 
न ाशी के वह बौ 
धम
के
सं ापक थे
और
उनका
मूल
नाम  स ाथ था।
लए
जाना
जाता
ह
जो
पूरी
संरचना
को
समा व  करता
ह। वह
एक  य थे
और शा 
गण के
थे।
आ ान
 ा 
करने
के
बाद,
उ ने 'बु '
या
'वाइज
वन' क 
उपा ध
 ा
Sol
33. क।
सही
उ र
रायगड़
ह। अपने
आ ान
 ा 
के
बाद,
वे सारनाथ गए
और पहली
बार
 श ा दी।
Key
Points
Sol
38.
बु 
के
जीवन
से
संबं धत
अ महा ान
ह
:
लुं बनी हड़ ा क 
खुदाई राय
बहा ुर
दया
राम
साहनी
ने
1921 म
क 
थी।
बोध
गया यह पंजाब
(पा क ान)
के
म टगोमरी
 जले
म
रावी
नदी के
तट
पर
 त
ह।
सारनाथ
कुशीनगर हड़ ा
 ल
से
मह पूण
खोज।
ाव ी
मानव
शरीर रचना
क  बलुथा
प र
क 
मू तयाँ
सं क ा
अनाज
का
भंडार
राजगृह
बैलगाड़ी
वैशाली
गौतम
बु 
का
ज ,
563
ई.पू.
म,
नेपाल
के
क पलव ु
के
लुं बनी
गाँव
म, Sol
39.
शा 
 य
वंश
म
 आ
था। सही
उ र
 वक 
1
ह,
अथात
गुजरात।
लोथल
 ाचीन
 सधु
घाटी
स ता
का
एक
शहर
ह
जो
गुजरात
म
 त
ह।
Important
Points
यह
गुजरात
रा 
के
भाल
 े 
म
 त
ह
और
1954
म
खोजा
गया
था।
महा भ न मण
या
महा नवाण
उस
घटना
को
कहा
जाता
ह
जब
गौतम
बु 
ने
अपना
घर
 ाग
 दया
था।
https://amzn.to/3HVqlZK ाचीन
इ तहास

भारतीय
पुरात 
सव ण
(ASI)
ने
13
फरवरी
1955
से
19
मई
1960
तक नयात
और
आयात
थे।


इसक 
खुदाई
क । कपास
का
उ ादन
होता
था।
ाचीन
समय
म,
यह
अपने
मो तय 
और
र 
के
 ापार
स हत
मह पूण
और लोथल
म,
हड़ ा
सं ृ त
म
मौजूद
स 
के
वजन
और
माप
को
समृ 
 ापा रक
क 
था। दखा
गया
था।
इसे
यूने ो
 ारा
 व 
धरोहर
 ल
के
 प
म
ना मत
 कया
गया
ह। वजन
और
आकार
म
आमतौर
पर
घनाकार
थे।
और
चूना
प र,
ीटाइट,
आ द
से
बने
थे
Sol
40.
Sol
44.
चं गु 
मौय
ने
322
ईसा
पूव
म
मौय
सा ा 
क 
 ापना
क 
जब
उ ने सही
उ र
 वक 
3,
अथात
बोध
गया
ह।
मगध
और
उ र-प मी
मेसीडो नयन  प 
के
रा 
को
जीत
 लया
था। Key
Points
मौय
सा ा 
क 
 ापना
322
ईसा
पूव
म
चं गु 
मौय
 ारा
क
गई
थी,
 ज ने नंदा
राजवंश को
उखाड़
फका
था
और
 सकंदर महाबो ध
मं दर
बोधगया
नामक
 ान
पर
 त
ह।
महान
के
सेनाओं
 ारा
वापसी
के
म नजर
 ानीय
श य 
के यह
बोधगया
म
एक
 ाचीन
पुन न मत
और
पुन न मत
बौ 
मं दर
ह।
वघटन
का
लाभ
उठाने
के
 लए
म 
और
प मी
भारत
म
तेजी
से ऐसा
माना
जाता
ह
 क
बु 
ने
यहां
आ ान
 ा 
 कया
था।
इस
 ान
पर
अपनी
श 
का
 व ार
 कया
था। एक
बो ध
वृ 
ह
 जसके
नीचे
उ 
 ान
 ा 
 आ
और
यह
 ह ुओ
ं और
बौ
के
 लए
एक
 मुख
तीथ
 ल
ह।
Sol
41. 2002
म,
यह
मं दर
एक
यूने ो
 व 
धरोहर
 ल
बन
गया।

पाट लपु 
मगध
महाजनपद
क 
राजधानी
थी। ान बु 
से
संबं धत


पाट लपु 
से
पहले,
राजगृह
उसक 
राजधानी
 आ
करती
थी।
कुशीनगर मृ ु
 ान
(महाप र नवाण)
मगध
सा ा 
म
गया,
पटना
और
शाहाबाद
के
वतमान
 जले
शा मल
थे।
मगध
उस
समय
के
सबसे
श शाली
रा 
म
से
एक
था। लु नी ज ल
बोध
गया बोधन/
 ानोदय
महाजनपद राजधानी
सारनाथ पांच
 ा ण
(ध 
च 
 वचन)
के
 लए
पहला
उपदश
कोसल ाव ी
अंग चंपा Sol
45.
वक 
4
सही
ह,
अथात
मौय।
मगध पाट लपु
म ा पावा महान
अशोक
मौय
वंश
से
स ं धत
थे।
इस
राजवंश
क 
 ापना
चं गु 
मौय
ने
वतमान
 बहार
म,
नंद
वंश
के
Sol
42. शासक-धनानंदा
को
हराने
के
बाद
क 
थी।

प व
राजाओं ने
इन
दवताओं
के
आवास
के
 लए सातव 
और
आठव अतर 
त :


शता ी म
कई
प र
के
मं दर 
का
 नमाण
कराया।
पूर
भारत
म
14
 मुख
 शलालेख
ह
उनम
से
सबसे
 स 
चे ई
से
65
 कमी
क 
 ू री
पर महाबलीपुरम म
पाए
इनम
से
 ेक
सं रण
अशोक
क 
ध 
नी त
का
 ह ा
ह
जाने
वाले सात
रथ
मं दर ह।
वे
अशोक
के
 वषय 
को
स ह ुता,
अ हसा
और
राजस ा
जैसे
 व भ
Sol
43.
दाश नक
और
 शास नक
गुण 
पर
 सखाने
के
 लए
थे।
सही
उ र
 ट
ह Sol
46.
सधु
घाटी
स ता
के
शहर 
म
घर
पके
 ए
 ट 
से
बने
होते
थे।
इसके
 नंद
वंश
के
सं ापक
उ सेन आठ
भाइय 
म
सबसे
छोट
थे।
अलावा,
ये
घर
छोट
लेन
या
आं गन
म
खुलते
ह।
वह
चं गु 
मौय
से
हार
गया,
 जसने
मौय
सा ा 
क 
 ापना
सधु
घाटी
स ता
पहली
स ताओं
म
से
एक
ह
जो
अपनी
शहरी
योजना,
क।
धातु
 व ान,
ह कला
तकनीक,
जल
आपू त
 णाली
और
जल
 नकासी
धना
नंद  ाचीन
भारत
म
नंद
वंश
का
अं तम
शासक
था।
णाली
के
 लए
जानी
जाती
ह।
सधु
घाटी
स ता
वतमान
उ र-पूव
अफगा न ान
से
पा क ान
और
उ र- Sol
47.
प म
भारत
तक
फैली
 ई
थी।
स ता
घ र-हकरा
नदी
और
 सधु
के
नदी-नाल 
म
पनपी।
 सधु
घाटी
स ता
 ु नया
क 
चार
सबसे
पुरानी ऋ ेद
म
गाय ी
मं 
3.62.10
 ोक
म
 मलता
ह।
स ताओं
म
से
एक
ह।
इसे
हड़ ा
स ता
के
 प
म
भी
जाना
जाता
ह
और मं 
 ाचीन
सूय
दवी
सा व ी
को
सम पत
ह।
यह
 ड
 णाली
पर
आधा रत
संग ठत
योजना
के
 लए
 स 
ह।
याद
रखने ऋ ेद
ने
 ु त
के
भजन
के
 ान
का
अनुवाद
 कया
ह
और
इसम
कई
धा मक
यो 
मह पूण
त । मं 
और
 ोक
ह।
सामा जक
 वशेषताएं :
-
सधु
घाटी
स ता
भारत
म
पहला
शहरीकरण
ह। वेद
का
नाम ववरण
इसम
एक
सु नयो जत
जल
 नकासी
 णाली,
 ड
पैटन
और
टाउन
ा नग
ह। अथववेद जा ू 
सू 
का
 ान
उ ने
समाज
म
समानता
पाई
ह। सामवेद धुन 
का
 ान
धा मक
त :
-
मातृदवी
या
श 
मातृ
दवी
ह। यजुवद य ीय
सू 
का
 ान
योनी
पूजा
और
 कृ त
पूजा
मौजूद
थी।
वे
पीपल
जैस
े पेड़ 
क 
पूजा
करते
थे। Sol
48.
उ ने
हवन
कुंड
नामक
अ 
पूजा
भी
क । सही
उ र
चार,
आठ
ह
।
पशुप त
महादव
को
जानवर 
के
 ामी
के
 प
म
जाना
जाता
ह। मह पूण
 ब ु
सधु
घाटी
स ता
के
लोग
गडा
और
बैल
जैस
े जानवर 
क 
पूजा
बौ 
धम
क 
न व
चार
आय
स 
और
अ ां गक
माग
ह।
करते
थे।
चार
आय
स 
ह:
आ थक
त :
-
ुख
क 
स ाई
(
 ु ा
)
सधु
घाटी
स ता
कृ ष
पर
आधा रत
ह।
ुख
के
कारण
का
सच
(
समु ा
)
इस
अव ध
म
 ापार
और
वा ण 
का
 वकास
 आ।
ुख
क 
समा 
का
स 
(
 नरोध
)
लोथल
म
एक
डॉकयाड
 मला
ह।
पथ
का
स 
 ुख
 नरोध
(म ा)
https://amzn.to/3HVqlZK ाचीन
इ तहास

अ ां गक
माग
ह
अ ां गक
माग
- मुख
 ब ु


सही
नज़ रया
या
सही
समझ
सही
 वचार 1997
से
एक
 व 
धरोहर
 ल,
लुं बनी
ने
स दय 
से
या य 
और
उपासक
सही
भाषण को
आक षत
 कया
ह।
सही
आचरण बोधगया
म
महाबो ध
मं दर
प रसर
 बहार
रा 
के
म 
भाग
म
 त
ह।
सही
आजी वका पहला
मं दर
स ाट
अशोक
 ारा
तीसरी
शता ी
ईसा
पूव
म
बनाया
गया
सही
 यास था
सही
 दमागीपन महाबो ध
मं दर
यूने ो
क 
 व 
धरोहर
 ल
ह।
सही
 ान
या
सही
 चतन
अतर 
जानकारी
Sol
49.
महायान
बौ 
धम:
सही
उ र
 वक 
2
ह,
अथात
महाभारत
बु 
क 
मृ ु
के
बाद,
एक
समूह
 वक सत
 आ
जो
मानता
था
 क
महाभारत
 ु नया
का
सबसे
बड़ा
महाका 
ह। सभी
मानवता
के
 लए
क णा
पथ
का
एक
मह पूण
 ह ा
ह,
और
यह
 ाचीन
भारत
के
दो
 मुख
महाका 
म
से
एक
ह,
 ू सरा
रामायण
ह। यह
 श ा
पूरी
तरह
से
सभी
के
 लए
उपल 
ह।
बाइबल
एक
संकलन
के
 प
म
 कट
होती
ह,
 व भ 
 प 
के
 ंथ 
का
एक जो
लोग
नए
 व ा रत
आदश 
का
पालन
करते
थे,
वे
खुद
को
संकलन
ह
जो
सभी
इस
 व ास
से
जुड़
 ए
ह
 क
वे
सामू हक
 प
से
भगवान महायान
कहते
थे,
 जसका
अथ
ह
बड़ा
बेड़ा।
के
श 
को
समा हत
करते
ह। महायान
बौ 
धम
म,
सभी
अ ास
करने
वाले
बौ 
 श ु
अवे ा,
पारसी
धम
के
धा मक
 ंथ 
का
 ाथ मक
सं ह
ह। बो धस 
ह।
कुरान
इ ाम
का
क ीय
धा मक
पाठ
ह,
 जसे
मुसलमान
ई र
(अ ाह)
से महायान
दश 
म,
बु 
क 
छ वय 
को
मं दर 
और
घर 
म
पूजा
क
एक
रह ो ाटन
मानते
ह। व 
ु के
 प
म
 ा पत
 कया
जाता
ह।
हीनयान
बौ 
धम:
Sol
50. बु 
क 
मृ 
ु के
लगभग
100
वष 
के
बाद
से,
बौ 
धम
क 
शैली
को
अपनाने
क 
 दशा
म
एक
आं दोलन
चल
रहा
था।
स 
 ीक
राज ू त
मेग नीज चं गु 
मौय के
शासनकाल
म
आए
थे। ारं भक
 भ ुओ
ं ने
 गत
 ानोदय
पर
 ान
क त
 कया,
मेग नीज से ूकस
I
 नकेटर का
एक
 ीक
राज ू त
था। यं
के
 लए
आ ा क
उ र
क 
तलाश
क ,
और
 ू सर 
को
'इं डका' मेगा नीज
का
 त त
काय
ह। आ ान
का
माग
 सखाया।
मेग नीज
भारत
क 
दो
 मुख
न दय   सधु
और
गंगा का
भी
वणन
करता
ह। ले कन
जानकारी
गंभीर
 प
से
आम
समुदाय
तक
ही
सी मत
थी
क
यह
माना
जाता
था
 क
केवल
 मक
ही
बु 
 ा 
कर
Sol
51. सकते
ह।
सही
जवाब
ह
उ ैन। इस
खंड
को
हीनयान
या
थेरवाद
के
नाम
से
भी
जाना
जाने
लगा।
Key
Points थेरवाद
का
अथ
ह
बड़ 
का
माग।
म लदप ा:
अशोक
के
 पता
 ब ु सार
ने
उ 
उ ैन
का
रा पाल
 नयु 
 कया,
जो म लदप ा,
या
" म लडा
के
 ,"
एक
मह पूण
 ारं भक
बौ
म 
भारत
के
अवंती
 ांत
म
एक
मह पूण
 शास नक
और
वा ण क
क पाठ
ह
जो
आमतौर
पर
पाली
कैनन
म
शा मल
नह 
ह।
था। फर
भी,
 म लदप ा
को
मह 
 दया
जाता
ह
 क
यह
बौ 
धम
इ तहासकार 
के
अनुसार,
अशोक
उ ैन
जाते
समय
अपनी
भावी
प ी के
कई
सबसे
क ठन
 स ांत 
को
बु 
और
 ता
के
साथ
महादवी
से
 मले। संबो धत
करता
ह।
अशोक
के
पु 
म हना
का
ज 
उ ैन
म
 आ
था। म लदप ा
राजा
मेनडर
 थम
(पाली
म
 म लडा)
और
नागसेन
अशोक
ने
सांची,
एरन,
 व दशा
आ द
म
बौ 
 ूप
बनवाए। नामक
एक
 बु 
बौ 
 भ 
ु के
बीच
एक
संवाद
 ुत
करता
ह।
मेनडर-
म
एक
इं डो-यूनानी
राजा
था
 जसके
बार
म
माना
जाता
ह
Additional
Information
क
उसने
लगभग
160
से
130
ईसा
पूव
तक
शासन
 कया
था।
स ाट
अशोक
लगभग
सभी
भारतीय
उपमहा ीप
म
268
से
232
ईसा
पूव वह
बै या
का
एक
राजा
था,
एक
 ाचीन
सा ा 
 जसने
अब
तक
शासन
 कया। तुकमे न ान,
अफगा न ान,
उजबे क ान
और
ता ज क ान,
वह
मौय
वंश
के
सं ापक
चं गु 
मौय
के
पोते
थे। साथ
ही
पा क ान
का
एक
छोटा
सा
 ह ा
ले
 लया।
सा ा 
क 
राजधानी
पाट लपु 
(आधु नक
पटना)
थी।
Sol
54.
उ ने
पूर
ए शया
म
बौ 
धम
को
फैलाने
म
मदद
क 
।

Sol
52.
ी
गु 
गु 
वंश
के
सं ापक
थे।
चं गु 
 थम
को
 ापक
 प
से
गु 
युग
के
सं ापक
के
 प
म
जाना
हड़ ा
स ता
क 
मुहर
मु 
 प
से चौकोर
आकार
क  थ 
और
इस
मुहर
से जाता
ह
जो
320
ई ी
म
उनके
रा ा भषेक
के
साथ
शु 
होता
ह।
एक
प र
 जसे  ीटाइट कहा
जाता
था,
से
हम
 सधु
घाटी
स ता
के समु गु 
गु 
वंश
के
शासक 
क 
सबसे
बड़ी
माना
जाता
ह।
धा मक
जीवन
का
अंदाजा
लगता
ह।
मह पूण
 ब ु
मु 
पु ष
दवता पशुप त
थे,
( ोटो- सवा) तीन
चेहर 
और
दो
स ग 
के
साथ गु 
वंश:
एक
यो गक
मु ा
म
बैठने
के
 प
म
मुहर 
म
 त न ध 
करते
थे।
वह चार
जानवर  ( हाथी,
बाघ,
राइनो
और
भस )
से
 घरा
 आ
ह
और
उसके गु 
वंश
के
सं ापक
 ी
गु 
थे।
पैर 
पर दो
 हरण  दखाई
दते
ह। वह
घटो च
 ारा
सफल
 आ
था।
इन
दोन 
को
महाराजा
कहा
जाता
था।
सधु
घाटी
स ता वतमान
उ र-पूव
अफगा न ान
से
पा क ान
और
उ र- अगला
शासक
चं गु 
 थम
था
और
वह
सबसे
पहले
महाराजा धराज
प म
भारत
तक
फैली
 ई
थी।
स ता
घ र-हकरा
नदी
और
 सधु
के कहलाया।
नदी-नाल 
म
पनपी।
 सधु
घाटी
स ता
 ु नया
क 
चार
सबसे
पुरानी चं गु ,
म
लगभग
330
ई ी
म
समु गु 
 ारा
सफल
 आ,
 ज ने
लगभग
स ताओं
म
से
एक
ह।
इसे
हड़ ा
स ता
के
 प
म
भी
जाना
जाता
ह
और पचास
वष 
तक
शासन
 कया।
यह
 ड
 णाली
पर
आधा रत
संग ठत
योजना
के
 लए
 स 
ह। वह
एक
महान
सै 
 तभा
था
और
कहा
जाता
ह
 क
उसने
द न
म
एक
सै 
अ भयान
क 
कमान
संभाली
थी,
और
 व 
 े 
क 
वन
जनजा तय 
को
Sol
53.

सही
उ 
तर
ह
 
लुं बनी
।
भी
अपने
अधीन
कर
 लया
था।
समु गु 
के
उ रा धकारी
चं गु 
 तीय,
 जसे
 व मा द 
के
नाम
से
भी
जाना
जाता
ह,
ने
मालवा,
गुजरात
और
का ठयावाड़
के
 ापक
 े 
पर
भगवान
बु 
का
ज 
563
ईसा
पूव
म
द णी
नेपाल
के
तराई
मैदान 
म
वजय
 ा 
क ।
त
लुं बनी
के
प व 
 े 
म
 आ
था।
मुख
 ब ु
https://amzn.to/3HVqlZK ाचीन
इ तहास

गु 
राजाओं
के
शासन
के
दौरान
कुछ
 स 
घटनाएँ : अ भ ान
शाकु लम,
मेघ ू त,
रघुवंश
आ द
उनके
 ारा
र चत
कुछ


रचनाएँ 
ह।
स 
चीनी
तीथया ी
फा ान
चं गु 
 तीय
के
शासनकाल
के
दौरान
भारत पा णनी:
आए
थे। 
उ 
भारतीय
भाषा व ान
का
जनक
भी
माना
जाता
ह।
भारत
म
अपने
नौ
वष 
के
 वास
म
से,
उ ने
गु 
सा ा 
म
छह
वष वे
अ ा ायी
( ाकरण
पर
सं ृ त
 ंथ)
के
लेखक
ह।
बताए। वेद ास:
चं गु 
 तीय
ने
प मी
भारत
के
शक
 प 
के
 खलाफ
यु 
छड़ा। वे
महान
महाका 
महाभारत,
वेद
और
पुराण 
के
लेखक
ह।
शक
 प
के
अं तम
शासक
 सह
III
को
परा जत,
ग ी
से
उतारकर
मार
दया
गया।
प मी
मालवा
और
का ठयावाड़
 ाय ीप
म
उसके
 े 
को Sol
61.
गु 
सा ा 
म
 मला
 लया
गया
था। सही
उ र
त मल
ह।
कुमारगु 
ने
नालंदा
 व व ालय
क 
न व
रखी। Key
Points

Sol
55. तोलक यम
को
त मल
म
तो पयार
ने
 लखा
था।


सही
उ र
 वक 
1
ह
अथात
राणा
साँगा यह
त मल
 ाकरण
पर
एक
रचना
ह।
Key
Points यह
उस
समय
के
राजनी तक
और
सामा जक
प र 
का
भी
पता
लगाता
ह।
राणा
साँगा
मेवाड़
का
सबसे
बड़ा
शासक
था। त मल
भाषा
का
 योग
संगम
सा ह 
म
 कया
गया
था।
उसने
मालवा
के
शासक
को
परा जत
 कया
और
अपने
 े 
का
 ह ा सं ृ त
भाषा
म
मह पूण
काय
वेद,
पंचतं ,
राजतरं गणी
और
 यद शका
नकाल
 लया। ह।
राणा
सांगा
राणा
कुंभा
के
पोते
थे।
ब ा
रावल
ने
728
CE
म
मेवाड़
सा ा 
क 
 ापना
क । Sol
62.
मेवाड़
के
महाराणा
अमर
 सह
(16
माच
1559
-
26
जनवरी
1620),
मेवाड़
के
महाराणा
 ताप
के
सबसे
बड़
पु 
और
उ रा धकारी
थे। पु क
हषच र 
के
लेखक
बाणभ 
ह।
हषच र 
राजा
हषवधन
क 
जीवनी
ह।
Sol
56. बाणभ 
राजा
हष
के
दरबारी
क व
थे।
सही
उ र
मुंडका
ह। वे
सं ृ त
सा ह 
के
महान
संर क 
म
से
एक
थे।
Key
Points बाणभ 
 व 
के
सबसे
पुराने
उप ास
कदं बरी
के
लेखक
थे।
स मेव
जयते
क 
उ 
 ाचीन
मुंडका
उप नषद 
से
 ई
ह। Sol
63.
अथ:
स 
अकेले
 वजय
करता
ह
इस
मं 
को
भारतीय
रा ीय
 तीक
के
नीचे
अं कत
 कया
गया
ह। ीलंका:
यह
भारत
सरकार
का
रा ीय
आदश
वा 
ह
 जसे
26
जनवरी
1950
को भारत
के
बाहर
बौ 
धम
का
 चार
करने
वाला
पहला
दश
 ीलंका
अपनाया
गया
था। ह।
क लग
यु 
के
बाद
अशोक
ने
बौ 
धम
अपनाने
के
बाद
अपने
पूर
Sol
57. रा 
म
बौ 
धम
का
 चार
 कया।
अशोक
बौ 
 मशन
का
नेतृ 
उनके
पु 
म ह ा
कर
रह
थे,
 ज ने
कालीबंगा
राज ान
के
हनुमानगढ़
 जले
म
 त
एक
 सधु
घाटी
 ल
ह। अ 
 भ ुओ
ं के
साथ
 ीलंका
क 
या ा
क ।
इस
 ल
क 
खोज
अमलानंद
घोष
ने
क 
थी।
इस
 ल
का
नाम
यहां
मौजूद
काली
चू ड़य 
के
सघन
 वतरण
पर
रखा
गया Sol
64.
ह
जो
इसके
टीले
क 
सतह
पर
पाए
जाते
थे।
यह
घ र
नदी
के
तट
पर
 त
ह,
ऋ ेद
म
भी
सर ती
के
 प
म
उ ेख पु म 
शुंग
ने
शुंग
वंश
क 
 ापना
क ।
कया
गया
ह
जो
2000
ईसा
पूव
तक
सूख
गई
थी। शुंग
अं तम मौय
स ाट बृह थ
मौय
के
सेनाप त थे।
राजवंश
क 
राजधानी
 बहार
म
पाट लपु 
थी।
Sol
58. बाद
म
इसे
म 
 दश
के
 व दशा
म
 ानांत रत
कर
 दया
गया।
राखीगढ़ी
ह रयाणा
म
 त
 सधु
घाटी
 ल
ह। Sol
65.
राखीगढ़ी, राखी
गढ़ी
(राखी
शाहपुर
+
राखी
खास), भारत
म
ह रयाणा रा  म
हसार
 जले का
एक
गाँव
ह,
जो
 द ी
के
उ र-प म
म
150
 कलोमीटर वा ी क
को
सं ृ त
रामायण
के
लेखक
के
 प
म
मा ता
 ा 
ह।
क 
 ू री
पर
 त
ह। रामायण
म 24,000
 ोक और
सात
छं द
ह।
यह लगभग
6500
ईसा
पूव
क 
पूव- सधु
घाटी
स ता
क  ब ी महाका 
रामायण,
5
व 
शता ी
ईसा
पूव
से
पहली
शता ी
ईसा
पूव
तक
का
 ल
ह। व भ 
 प
से
 दनां कत।
वा ी क
एक
सं ृ त
क व
थे
और
उ 
आ द
क व
के
 प
म
मनाया
जाता
Sol
59. था।
बौ 
वा ुकला
को
 ूप,
चै 
और
 वहार
के
 प
म
दखा
जा
सकता
ह।
Sol
66.
बौ 
धम
म
पूजा
 ल
के
 लए चै  का
 नमाण
 कया
गया
था।
वहार का
 नमाण
बौ 
 भ ुओ
ं के
रहने
के
उ 
से
 कया
गया
था।
वे मौय
वंश
का
अं तम
राजा बृह थ था।
भ ुओ
ं के
 नवास
 ान
ह। अं तम
मौय
शासक
बृह थ
क 
ह ा 185
ईसा
पूव म
उनके
सेनाप त पु म
शुंग ने
क 
थी।
Sol
60. शुंग वंश
क 
 ापना
पु म 
शुंग
ने
क 
थी।
सही
उ र
 वक 
1
ह
अथात
तुलसीदास। पु म 
को
उनके
पु  अ म  ने
का लदास
के
Key
Points नाटक माल वका म  का
नायक
बनाया।
रामच रतमानस: Sol
67.
तुलसीदास
एक
महाका 
रामच रतमानस
(भगवान
राम
क 
एक हड़ ा
शहर
म
 कलेबंद
गढ़
और
शहर
 नयोजन
दो
सामा 
 वशेषताएं 
थ   क
शहर
कहानी)
के
लेखक
ह नयोजन
के
अनुसार,
शहर
को
ऊपरी
शहर
और
 नचले
शहर
म
 वभा जत
 कया
गया
यह
अवधी
भाषा
म
 लखा
गया
था। था।
का लदास:
वह
च 
गु 
 तीय
के
दरबार
म
रहते
थे। गढ़
एक
ऊंचे
चबूतर
पर
बनाया
गया
था जब क  नचला
शहर
शहर
के
 नचले
ह े
म
 त
था।
https://amzn.to/3HVqlZK ाचीन
इ तहास

नचला
शहर
गढ़
से
बड़ा
था। अपने
सै 
 वजय
के
कारण,
डॉ वी.ए.
 थ
ने
समु गु 
को
भारत
का


नेपो लयन
कहा
ह।
Sol
68.

सही
उ 
तर
ह
 
बु ,
ध ,
संघ
। Key
Points
समु गु :

बौ 
धम
:
स ाथ
गौतम
 ारा
 ा पत। उसका
 े 
उ र
म
 हमालय
से
लेकर
द ण
म
नमदा
नदी
तक
और
पूव
म
उनका
ज 
563
ईसा
पूव
नेपाल
के
 नकट
लुं बनी
म
 आ
था। पु 
नदी
से
लेकर
प म
म
यमुना
नदी
तक
फैला
 आ
था।
एक
पीपल
के
पेड़
के
नीचे
 ान
क 
49
 दन 
के
बाद,
गौतम
 बहार उसने
सात
अलग-अलग
 कार
के
 स े 
बनाने
शु 
 कए
-
म
बोधगया
म
बो ध
( ान)
क 
 ा 
 ई। डड
टाइप,
आचर
टाइप,
बैटल
ऐ 
टाइप,
अ मेध
टाइप,
बु 
ने
अपना
पहला
उपदश
सारनाथ,
यूपी
म
 दया
था
और
इस टाइगर
 ेयर
टाइप,
द
 कग
और
 ीन
टाइप
और
गीतकार
टाइप।
आयोजन
को
धम-च - वतन
के
 प
म
जाना
जाता
ह। गु 
राजा
के
पास
अपने
रा 
म
मौ क
 णाली
काफ 
थी।
उ ने
'म म
माग'
या
म 
माग
का
अनुसरण
 कया वह
पड़ोसी
राजाओं
के
साथ
 म तापूण
संबंध
रखता था।
उ ने
बौ 
 भ ओु ं
उ र
 दश
के
कुशीनगर
म
उनका
 नधन
हो
गया
और
इस
आयोजन क 
सु वधा
के
 लए
सीलोन
के
राजा
को
बोध-गया
म
एक
बौ 
मठ
बनाने
क
को
महाप र न न
के
नाम
से
जाना
जाता
ह। अनुम त
दी
थी।
बौ 
धम
के
 
र 
ह:
बु ,
ध 
(
बु 
क 
 श ाएं 
)
और
संघ
(जो
बौ 
धम
का
पालन
करते
ह)। Sol
71.
सही
उ र
हषवधन
ह।
अतर 
जानकारी Key
Points

महायान
बौ 
धम: चीनी
या ी
 न
 ांग
हषवधन
के
शासनकाल
म
भारत
आया
था।


बु 
क 
मृ ु
के
बाद,
एक
समूह
 वक सत
 आ
जो
मानता
था
 क
सभी
मानवता
के
 लए
क णा
पथ
का
एक
मह पूण
 ह ा
ह,
और Additional
Information
यह
 श ा
पूरी
तरह
से
सभी
के
 लए
उपल 
ह।
भारत
का
दौरा
करने
वाले
चीनी
या ी
जो
लोग
नए
 व ा रत
आदश 
का
पालन
करते
थे,
वे
खुद
को
फा- ान
405
ई 
-
411
ई
महायान
कहते
थे,
 जसका
अथ
ह
बड़ा
बेड़ा।
वह
एक
चीनी
बौ 
 भ ु
थे।
महायान
बौ 
धम
म,
सभी
अ ास
करने
वाले
बौ 
 श ु
उ ने
 व मा द 
(चं गु 
 तीय)
के
शासनकाल
के
बो धस 
ह।
दौरान
भारत
का
दौरा
 कया।
महायान
दश 
म,
बु 
क 
छ वय 
को
मं दर 
और
घर 
म
पूजा
क
वह
लुं बनी
क 
अपनी
या ा
के
 लए
जाने
जाते
ह।
व 
ु के
 प
म
 ा पत
 कया
जाता
ह।
उनक 
या ा
का
वणन
उनके
या ा
वृ ांत
"बौ 
रा 
का
हीनयान
बौ 
धम:
रकॉड"
म
 कया
गया
ह।
बु 
क 
मृ 
ु के
लगभग
100
वष 
के
बाद
से,
बौ 
धम
क 
शैली
न
सांग
630
ई 
-
645
ई
को
अपनाने
क 
 दशा
म
एक
आं दोलन
चल
रहा
था।
वह
एक
चीनी
या ी
था।
ारं भक
 भ ुओ
ं ने
 गत
 ानोदय
पर
 ान
क त
 कया,
उ ने
हष
वधन
क 
सव ता
के
दौरान
भारत
का
दौरा
यं
के
 लए
आ ा क
उ र
क 
तलाश
क ,
और
 ू सर 
को
कया।
आ ान
का
माग
 सखाया।
सी-यू-क 
या
‘द
 रकॉड
ऑफ
द
वे न
व ’
उनके
 ारा
ले कन
जानकारी
गंभीर
 प
से
आम
समुदाय
तक
ही
सी मत
थी
लखी
गई
थी।
क
यह
माना
जाता
था
 क
केवल
 मक
ही
बु 
 ा 
कर
सकते
ह। Sol
72.
इस
खंड
को
हीनयान
या
थेरवाद
के
नाम
से
भी
जाना
जाने
लगा। सही
 वक 
ह
2
अथात
बु
थेरवाद
का
अथ
ह
बड़ 
का
माग।
म लदप ा: गौतम
बु 
का
ज 
 स ाथ
गौतम
के
 प
म
क पलव ु
,
नेपाल
म
लगभग
म लदप ा,
या
" म लडा
के
 ,"
एक
मह पूण
 ारं भक
बौ 563
ईसा
पूव
म
 आ
था
पाठ
ह
जो
आमतौर
पर
पाली
कैनन
म
शा मल
नह 
ह। उ 
'ए शया
का
 काश
'
और
'
 बु 
 
'
के
 प
म
भी
जाना
जाता
ह।
फर
भी,
 म लदप ा
को
मह 
 दया
जाता
ह
 क
यह
बौ 
धम बु 
ने
उ वेला,
बोधगया
म
 नवाण
 ा 
 कया।
के
कई
सबसे
क ठन
 स ांत 
को
बु 
और
 ता
के
साथ उ ने
अपना
पहला
उपदश
सारनाथ
म
 दया
था
।
संबो धत
करता
ह। उ ने
अ ां गका
माग
या
अ ां गक
माग
 दया।
म लदप ा
राजा
मेनडर
 थम
(पाली
म
 म लडा)
और
नागसेन
नामक
एक
 बु 
बौ 
 भ 
ु के
बीच
एक
संवाद
 ुत
करता
ह। अतर 
जानकारी
मेनडर-
म
एक
इं डो-यूनानी
राजा
था
 जसके
बार
म
माना
जाता
ह
क
उसने
लगभग
160
से
130
ईसा
पूव
तक
शासन
 कया
था। महायान
बौ 
धम:
वह
बै या
का
एक
राजा
था,
एक
 ाचीन
सा ा 
 जसने
अब बु 
क 
मृ ु
के
बाद,
एक
समूह
 वक सत
 आ
जो
मानता
था
 क
तुकमे न ान,
अफगा न ान,
उजबे क ान
और
ता ज क ान, सभी
मानवता
के
 लए
क णा
पथ
का
एक
मह पूण
 ह ा
ह,
और
साथ
ही
पा क ान
का
एक
छोटा
सा
 ह ा
ले
 लया। यह
 श ा
पूरी
तरह
से
सभी
के
 लए
उपल 
ह।
जो
लोग
नए
 व ा रत
आदश 
का
पालन
करते
थे,
वे
खुद
को
Sol
69. महायान
कहते
थे,
 जसका
अथ
ह
बड़ा
बेड़ा।
मौय
सा ा 
का
समय
321
से
185
ईसा
पूव
तक
था। महायान
बौ 
धम
म,
सभी
अ ास
करने
वाले
बौ 
 श ु
पाट लपु  मौय
सा ा 
क 
राजधानी
थी। मौय
सा ा 
म
राजधानी
पाट लपु बो धस 
ह।
और त शला,
उ यनी,
तोसाली
और
सुवण ग र के
 ांतीय
क 
म
पांच
 मुख महायान
दश 
म,
बु 
क 
छ वय 
को
मं दर 
और
घर 
म
पूजा
क
राजनी तक
क 
थे । व 
ु के
 प
म
 ा पत
 कया
जाता
ह।
हीनयान
बौ 
धम:
Sol
70.
बु 
क 
मृ 
ु के
लगभग
100
वष 
के
बाद
से,
बौ 
धम
क 
शैली
सही
उ र
समु गु  ह।
को
अपनाने
क 
 दशा
म
एक
आं दोलन
चल
रहा
था।
चं गु 
 ारा समु गु  का
चयन
गु 
वंश
के
 हत
म
सबसे
बड़ा
एकल ारं भक
 भ ुओ
ं ने
 गत
 ानोदय
पर
 ान
क त
 कया,
कारक
सा बत
 आ। यं
के
 लए
आ ा क
उ र
क 
तलाश
क ,
और
 ू सर 
को
उनके
दरबारी
क व
हरीसेना
 ारा
 ल खत
इलाहाबाद
 पलर
 शलालेख
( याग आ ान
का
माग
 सखाया।
श )
समुं गु 
क 
 वजय
का
 व ृत
 ववरण
दता
ह। ले कन
जानकारी
गंभीर
 प
से
आम
समुदाय
तक
ही
सी मत
थी
य प
उनके
सै 
अ भयान 
ने,
समु गु 
ने
भारत
म
राजनी तक
एकता
को क
यह
माना
जाता
था
 क
केवल
 मक
ही
बु 
 ा 
कर
काफ 
हद
तक
 फर
से
 ा पत
 कया
था। सकते
ह।
https://amzn.to/3HVqlZK ाचीन
इ तहास

इस
खंड
को
हीनयान
या
थेरवाद
के
नाम
से
भी
जाना
जाने
लगा। न- ांग
ने
हषवधन
के
दरबार
म
जाकर
'शी-यू-क '
पु क
 लखी।


थेरवाद
का
अथ
ह
बड़ 
का
माग। सी-यू-क 
का
अथ
ह
"प मी
दश 
का
 रकॉड।"
म लदप ा: न
 ांग
स ाट
हष
क 
अव ध
के
दौरान
भारत
आया
था।
म लदप ा,
या
" म लडा
के
 ,"
एक
मह पूण
 ारं भक
बौ न
 ांग
नालंदा
 व व ालय
म
पाँच
वष 
तक
रह।
पाठ
ह
जो
आमतौर
पर
पाली
कैनन
म
शा मल
नह 
ह। न
 ांग
कम पा
के
शासक
भा र
वमन
के
अ त थ
थे।
फर
भी,
 म लदप ा
को
मह 
 दया
जाता
ह
 क
यह
बौ 
धम न
 ांग
ने
क ौज
को
एक
सुंदर
शहर
बताया।
के
कई
सबसे
क ठन
 स ांत 
को
बु 
और
 ता
के
साथ पु
स ग लग
एक
 कग
राजवंश
के
चीनी
लेखक
थे।
संबो धत
करता
ह। पु
स ग लग
 ज
 ोरीज
के
लेखक
थे।
म लदप ा
राजा
मेनडर
 थम
(पाली
म
 म लडा)
और
नागसेन
नामक
एक
 बु 
बौ 
 भ 
ु के
बीच
एक
संवाद
 ुत
करता
ह। बाणभ :
मेनडर-
म
एक
इं डो-यूनानी
राजा
था
 जसके
बार
म
माना
जाता
ह
क
उसने
लगभग
160
से
130
ईसा
पूव
तक
शासन
 कया
था। बाणभ 
राजा
हषवधन
के
दरबार
म
अ ाना
क व
थे।
वह
बै या
का
एक
राजा
था,
एक
 ाचीन
सा ा 
 जसने
अब काद री
बाणभ 
का
काम
ह।
तुकमे न ान,
अफगा न ान,
उजबे क ान
और
ता ज क ान, बाणभ 
 ारा
"हषच रत
"
भी
 लखा
गया
था।
साथ
ही
पा क ान
का
एक
छोटा
सा
 ह ा
ले
 लया।
Sol
77.
Sol
73. लोथल
का
बंदरगाह
शहर.
गुजरात
के
आधु नक
रा 
म
 त,
हड़ ा
स ता
का
सही
उ र
ह
पा ण न। सबसे
द णी
शहर
था।
यह
1954
म
खोजा
गया
था
और
खुदाई
भारतीय
पुरात
सव ण
(एएसआई)
 ारा
1955
से
1960
तक
क 
गई
थी।
एएसआई
के
अनुसार,
का लदास: लोथल
म
 ु नया
के
सबसे
पुराने
 ात
बंदरगाह
थे।
वह
चं 
गु 
 तीय
के
दरबार
म
रहता
था।
Sol
78.
अ भ ान
शाकु लम,
मेघ ू त,
रघुवंश
आ द
उनके
 ारा
र चत
कुछ
रचनाएँ 
ह।
थम
तीथकर
ऋषभनाथ
या
ऋषभदव
थे।

पा ण न:
इ 
जैन
धम
का
सं ापक
माना
जाता
था।
उ 
भारतीय
भाषा व ान
का
जनक
भी
माना
जाता
ह।
24व
तीथकर
वधमान
महावीर
का
ज 
540
ई.पू.
वैशाली
के
पास
कुंडल ाम
वे
अ ा ायी
( ाकरण
पर
सं ृ त
 ंथ)
के
लेखक
ह।
गाँव
म
 आ
था।
वेद ास:
वह
 ातृक
कुल
से
थे।
वे
महान
महाका 
महाभारत,
वेद
और
पुराण 
के
लेखक
ह।
इ 
अं तम
तीथकर
माना
जाता
था।
चाण :

उ 
पारंप रक
 प
से
कौ ट 
या
 व ुगु 
के
नाम
से
जाना
जाता Sol
79.
ह।
वह
चं गु 
मौय
के
शासन
म
 धानमं ी
थे। क न 
कुषाण
वंश
से
संबं धत
था।
अथशा 
एक
 ाचीन
सं ृ त
पु क
ह,
 जसे
चाण 
ने
 लखा
ह। कुषाण
वंश
क 
 ापना
कुजला
कड फसेस
ने
क 
थी।
क न 
ने
ई ी
सन्
पहली
शता ी
के
आसपास
शासन
 कया
और
कुषाण
Sol
74.
वंश
का
सबसे
महान
राजा
था।
ड ूगढ़
 सधु
घाटी
स ता
का
एक
मह पूण
 ल
नह 
ह। क न 
के
समय,
कुषाण
शासन
भारतीय
उपमहा ीप
के
उ री
भाग,
लोथल
 सधु
घाटी
स ता
के
सबसे
द णी
शहर
म
से
एक
था।
इसके
पास
 ु नया
के
अफगा न ान
और
संभवतः
क ीर
 े 
के
उ री
भाग
म
म 
ए शया
के
सबसे
पुराने
 ात
बंदरगाह
ह।
मोहनजोदड़ो
और
हड़ ा
 सधु
घाटी
स ता
क 
सबसे
बड़ी
ब य 
म
से
एक
और े 
तक
फैला
 आ
था।
ु नया
के
सबसे
पुराने
शहर 
म
से
एक
थे।
वे
अपने
 भावशाली
और
संग ठत
न े
के उसे
बौ 
धम
के
एक
महान
संर क
के
 प
म
भी
याद
 कया
जाता
ह।
लए
जाने
जाते
थे। उ ने
बौ 
क व
अ घोष
क 
छ छाया
म
बौ 
धम
का
अ यन
 कया।
क ीर
म
उसके
संर ण
म
चतुथ
बौ 
संगी त
भी
संयो जत
क 
गई
थी।
Sol
75. इस
संगी त
ने
अ भधम
पर
महान
 ट णी
का
संकलन
 कया।
सही
उ र
ह
चं गु ।
Sol
80.
चाण 
( व ुगु /कौ ट )
क 
मदद
से,
चं गु 
मौय
ने
धन
नंदा
(322 सही
उ र
क ौज
ह।
ईसा
पूव)
को
हराया
और
मौय
राजवंश
क 
 ापना
क ।
चाण 
ने
चं गु 
मौय
और
उनक 
सेना
को
मगध
का
 सहासन
संभालने राजा
हषवधन
ने
 ान
के
एक
बड़
क 
क 
 ापना
क 
 जसे
क ौज
म
'भ -
के
 लए
 ो ा हत
 कया। वहार'
कहा
जाता
था।
उनके
 वशाल
सा ा 
म
 बहार,
बंगाल,
द न
(आधु नक
केरल, राजा
हषवधन
 ज ने
606-647
ई.
तक
शासन
 कया,
वे
 भाकरवधन
के
त मलनाड
और
पूव र
भारत
के
कुछ
 ह 
को
छोड़कर),
पूव छोट
पु 
थे।
अफ़गा न ान,
बलू च ान
और
 सधु
नदी
का
प म
 े 
शा मल
था।
Important
Points
bindusara
क ौज
हषवधन
के
सा ा 
क 
राजधानी
थी
 जसे
उनके
 ारा
थाने र
से
चं गु 
का
उ रा धकार
उसके
पु 
 ब ुसार
को
 ा 
 आ। ानांत रत
 कया
गया
था।
ब ुसार
ने
लगभग
25-26
वष 
तक
शासन
 कया
और
उनका
उ रा धकार राजा
हषवधन
भारतीय
उपमहा ीप
के
सबसे
महान
राजाओं
म
से
एक
थे।
ववा दत
रहा
 जसने
अंततः
भारत
को
अशोक
नामक
एक
महान
राजा
 दया।

अशोक Additional
Information

अशोक
तीसरा
और
मौय
शासक 
म
सबसे
महान
शासक
था। बाणभ 
के
हषच रत
 ारा
हषवधन
के
शासनकाल
के
बार
म
 ापक


अशोक
को
अशोकवधन
कहा
जाता
ह। जानकारी
दी
गई
ह।
चीनी
या ी,
 न
सांग
ने
वणन
 कया
 क
हष
ने
अपने
शासनकाल
के
पहले
छह
कुणाल वष 
के
भीतर
पूर
दश
पर
 वजय
 ा 
क ,
ले कन
वह
उ र
भारत
पर
पूरी
तरह
से
क ा
नह 
कर
पाए
और
न
ही
उनके
यु 
और
 वजय
उनके
कुणाल
स ाट
अशोक
और
रानी
प ावती
और
अशोक
के
उ रा धकारी शासनकाल
के
पहले
छह
वष 
तक
सी मत
थे।
वा रस
के
पु 
थे।
Sol
81.
Sol
76. सही
उ र
ह,
अशोक
मौय।
सही
उ र
ह
 न- ांग।
https://amzn.to/3HVqlZK ाचीन
इ तहास

अशोक
मौय
को
'दवानाम या
 यदश '
के
नाम
से
जाना
जाता
था। वह
लगभग
पं ह
वष 
तक
भारत
म
रह
और
अपनी
पु क
“सी-यू-क ”


अशोक
ने
 यं
"दवानाम या
 यदश "
क 
उपा ध
धारण
क 
थी। नामक
अपने
अनुभव
को
दज
 कया।
पु क
दश
के
धम,
री त- रवाज ,
परंपराओं
आ द
पर
 काश
डालती
ह।
Important
Points उ 
"तीथया य 
के
राजकुमार"
के
 प
म
व णत
 कया
गया
ह
और
उनके
खात 
म
भारत
के
राजनी तक,
सामा जक
और
धा मक
सेट
पर
ब त
सारी
"दवानाम या"
का
अथ
ह
'दवताओं
को
 य'
और
" यदश "
से
ता य
ह, जानकारी
ह।
जो
हर
 कसी
के
बार
म
अचं भत
करता
ह। न
 ांग
ने
क ीर,
पंजाब
का
दौरा
 कया
और
क पलव ,ु 
बोध-गया,
सारनाथ
और
कुशीनगर
के
 लए
रवाना
 ए
।
Additional
Information
उ ने
नालंदा
 व व ालय
म
अ यन
 कया
और
ड न,
उड़ीसा
और
मौय
सा ा 
क 
 ापना
चं गु 
मौय
ने
क 
थी। बंगाल
से
या ा
क ।
चं गु 
मौय
ने
नंदा
सा ा 
को
उखाड़
फका
था। चूं क
वह
14
साल
तक
भारत
म
रह,
इस लए
उनके
वृ ांत
बताते
ह
 क
 ाचीन
ब ु सार
चं गु 
मौय
के
पु 
थे। भारत
एक
बार
कैसा
रहा
होगा।
अशोक
 ब ुसार
का
उ रा धकारी
था।
Sol
84.
Sol
82. गु 
काल

सही
उ 
तर
ह
 
पाली
।
इसक 
अव ध 275 ई ी से 550 ई ी तक
क 
थी।
मुख
 ब ु
बौ 
धम ंथ,
म मा
 नकाय,
तीसरी
शता ी
ईसा
पूव
-
 ू सरी
शता ी
सीई
के
बीच तीसरी
शता ी ई. के
म 
म
मौय
सा ा 
के
पतन
के
कारण
गु 
सा ा
बना
ह,
 जसका
अथ
ह
"म -लंबाई
के
 वचन 
का
सं ह",
सु 
 पटक
म
पांच
 नकाय क 
 ापना
 ई।
या
सं ह 
म
से
 ू सरा
ह,
जो
"
तीन
टोक रयाँ
”जो
थेरवाद
बौ 
धम
के
पाली
 ट पटका इस
अव ध
को
आमतौर
पर  ह ू 
धम
के
 ण
युग के
 प
म
जाना
जाता
ह।
क 
रचना
करती
ह। ी
गु  इसके
सं ापक
थे
और  ं दगु  इस
सा ा 
का
अं तम
शासक
अ त र 
जानकारी था।

महायान
बौ 
धम: Sol
85.


बु 
क 
मृ ु
के
बाद,
एक
समूह
 वक सत
 आ
जो
मानता
था
 क
सभी
मानवता
के
 लए
क णा
पथ
का
एक
मह पूण
 ह ा
ह,
और लोथल
गुजरात
म
एक
बंदरगाह
शहर
ह,
जो
साबरमती
नदी
क 
एक
सही
यह
 श ा
पूरी
तरह
से
सभी
के
 लए
उपल 
ह। सहायक
नदी,
भोगवा
नदी
के
तट
पर
 त
ह।
जो
लोग
नए
 व ा रत
आदश 
का
पालन
करते
थे,
वे
खुद
को यह
 ु नया
म
सबसे
पहले
 ात
गोदी
(डॉक)
थी
 जसका
उपयोग
मु 
 प
से
महायान
कहते
थे,
 जसका
अथ
ह
बड़ा
बेड़ा। सधु
घाटी
स ता
म
 ापार
के
उ 
के
 लए
 कया
जाता
था।
महायान
बौ 
धम
म,
सभी
अ ास
करने
वाले
बौ 
 श ु लोथल
आधु नक
गुजरात
म
अहमदाबाद
 जले
म
 त
ह।
इसे
भारतीय
बो धस 
ह। पुरात 
सव ण
के
अनुसार
एक
बंदरगाह
शहर
माना
जाता
ह।
महायान
दश 
म,
बु 
क 
छ वय 
को
मं दर 
और
घर 
म
पूजा
क
व 
ु के
 प
म
 ा पत
 कया
जाता
ह। Sol
86.
हीनयान
बौ 
धम:
बु 
क 
मृ 
ु के
लगभग
100
वष 
के
बाद
से,
बौ 
धम
क 
शैली चं गु 
मौय  ारा
सम थत
चाण 
नामक
एक
बु मान

को
अपनाने
क 
 दशा
म
एक
आं दोलन
चल
रहा
था। को नंदवंश  ारा
परा जत
 कया
गया
था
और 321
ईसा
पूव
म
मौय
वंश का
ारं भक
 भ ुओ
ं ने
 गत
 ानोदय
पर
 ान
क त
 कया, शासन
 ा पत
 कया
गया।
यं
के
 लए
आ ा क
उ र
क 
तलाश
क ,
और
 ू सर 
को कौ ट 
के
नाम
से
जाने
जाने
वाले
चाण 
को
पारंप रक
 प
से
चं गु 
का
आ ान
का
माग
 सखाया। मं ी
माना
जाता
ह।
ले कन
जानकारी
गंभीर
 प
से
आम
समुदाय
तक
ही
सी मत
थी चाण 
के
कई
 वचार 
को
सं ृ त
म
 लखे
गए अथशा  नामक
एक
पु क
क
यह
माना
जाता
था
 क
केवल
 मक
ही
बु 
 ा 
कर म
 लखा
गया
था। 
सकते
ह।
इस
खंड
को
हीनयान
या
थेरवाद
के
नाम
से
भी
जाना
जाने
लगा।
LEVEL
2 87
-
166
Questions
थेरवाद
का
अथ
ह
बड़ 
का
माग।
म लदप ा:
म लदप ा,
या
" म लडा
के
 ,"
एक
मह पूण
 ारं भक
बौ Sol
87.
पाठ
ह
जो
आमतौर
पर
पाली
कैनन
म
शा मल
नह 
ह। सही
उ र
वराह म हर
ह।
फर
भी,
 म लदप ा
को
मह 
 दया
जाता
ह
 क
यह
बौ 
धम Key
Points
के
कई
सबसे
क ठन
 स ांत 
को
बु 
और
 ता
के
साथ
संबो धत
करता
ह। वराह म हर
ने
बृह ं हता
 लखी।
म लदप ा
राजा
मेनडर
 थम
(पाली
म
 म लडा)
और
नागसेन यह
सं ृ त
म
एक
मह पूण
 व कोश
ह।
नामक
एक
 बु 
बौ 
 भ 
ु के
बीच
एक
संवाद
 ुत
करता
ह। यह
पाठ
कई
भारतीय
 ल पय 
म
मौजूद
ह
और
 ह ू ,
जैन
और
बौ 
मं दर
मेनडर-
म
एक
इं डो-यूनानी
राजा
था
 जसके
बार
म
माना
जाता
ह और
मठ 
म
संर त
ह।
क
उसने
लगभग
160
से
130
ईसा
पूव
तक
शासन
 कया
था। पु क
म
अटकल
को
शा मल
 कया
गया
ह
और
इसम
खगोल
 व ान,
 ह
वह
बै या
का
एक
राजा
था,
एक
 ाचीन
सा ा 
 जसने
अब क 
चाल,
 हण,
वषा,
वा ुकला,
फसल 
क 
वृ ,
इ 
का
 नमाण,
 ववाह,
तुकमे न ान,
अफगा न ान,
उजबे क ान
और
ता ज क ान, घरलू
संबंध,
र ,
मोती
और
अनु ान
स हत
कई
 वषय 
को
शा मल
 कया
साथ
ही
पा क ान
का
एक
छोटा
सा
 ह ा
ले
 लया। गया
ह।
ग ड़
पुराण
म
पाए
गए
र 
मू ांकन
क 
कसौटी
पर
खंड
 
ह
और
एक
Sol
83. ही
पाठ
से
प व 
नौ
मोती
पर
 व ृत
ह।
सही
उ र
ह
हषवधन। इसम
106
अ ाय
ह
और
इसे
"महान
संकलन"
के
 प
म
जाना
जाता
ह।

न
 ांग,
चीनी
तीथया ी,
 ामा णक
बौ 
 ल पय 
को
हा सल
करने
के Sol
88.


उ 
से
हषवधन
के
शासनकाल
के
दौरान
भारत
आए
थे।
तो,
 वक 
2 सही
उ र
बौ 
धम
ह।
सही
ह।
बौ 
धम:
Key
Points बौ 
धम
क 
उ 
 स ाथ
क 
कहानी
से
जुड़ी
 ई
ह
 जसे
बु
के
नाम
से
जाना
जाता
ह।
क ौज
 वधानसभा
(643
ई ी)
को
 न
 ांग
के
स ान
म
और
बौ 
धम उनके
ज दवस
के
 दन
ही
35
वष
क 
आयु
म
उ ने
पीपल
वृ
के
महायान
सं दाय
को
लोक य
बनाने
के
 लए
आयो जत
 कया
गया
था। के
नीचे
आ ान
( नवाण)
 ा 
 कया
और
वे
बु ,
महा ा
बु
To Practice 3751 More Questions
From Remaining Chapters

Buy NOW

Available at
&

Unique Features of SmartBook


Time To Answer (TTA)

Smart Answer Key

Level - Wise Difficulty

Best 4000 Questions

Free Video Lessons


https://amzn.to/3HVqlZK ाचीन
इ तहास

बन
गए। कु े 
यु 
शु 
होने
से
पहले
कृ 
एक
धनुधर
 ंथ
(भगवत
गीता)
म
अजुन


बौ 
गया
म
 ान
 ा 
के
बाद
उ ने
वाराणसी
के
पास
 त से
बात
कर
रह
ह।
सारनाथ
के
 हरण
पाक
म
अपने
पांच
 श 
को
अपना
पहला यह
 नयमावली
माना
जाता
ह
 जस
पर
एक
इं सान
सनातन
धम
के
अनुसार
उपदश
 दया। अपने
जीवन
का
पालन
करना
चा हए।
अजंता
क 
 च कला
म
बु 
के
आसपास
तीन
सुर ा क
दवताओं
म
से
एक
मंजु ी
को
भी
अजंता
क 
गुफाओं
म
दशाया
गया
ह। Important
Points
वह
बु 
क 
बु म ा
से
संबं धत
ह
और
वह
अपने
हाथ 
म
बु म ा
क 
तलवार
उठाए
एक
पु ष
बोधीस 
ह। भगव ीता
क 
रचना
 ास
नाम
के
एक
 ाचीन
ऋ ष
ने
क 
थी।
आजीवक: अ धकांश
 ह ू 
परंपराओं
म,
कृ 
 पायन
 ास,
 जसे
 ास
या
वेद- ास
इस
सं दाय
क 
 ापना
पांचव 
शता ी
ईसा
पूव
म
म ल (जो
वेद 
को
चार
भाग 
म
वग कृत
करता
ह)
के
 प
म
भी
जाना
जाता
ह,
एक
गोशाल
 ारा
क 
गई
थी। क ीय
और
पूजनीय
 
ह।
सं दाय
पूण
 नय तवाद
क 
 नय त
(भा )
 स ांत
पर
आधा रत परंपरागत
 प
से
उ 
महाभारत
का
लेखक
माना
जाता
ह,
हालां क
यह
भी
ह। आमतौर
पर
माना
जाता
ह
 क
केवल
महाका 
का
क 
भरत
 लखा
गया
था।
यह
मानता
ह
 क
कोई
 तं 
इ ा
नह 
होती
ह
और
जो
कुछ
भी उ 
पुराण 
के
मुंशी
के
 प
म
भी
जाना
जाता
ह,
 ाचीन
 ह ू 
 ंथ 
म
 व भ
आ
ह,
हो
रहा
ह
या
होगा,
पूण
 प
से
पूव- नयो जत
या
पूव- दवी-दवताओं
को
 द 
कथाओं
के
मा म
से,मु 
 प
से
 ह ू 
धम
म
 द
नधा रत
ह
और
यह
 ांडीय
 स ांत 
पर
आधा रत
ह।
इस लए मू त
भगवान
के
 प
म
 वन
 कया
जाता
ह।
इस
सं दाय
का
कम
पर
कोई
 व ास
नह 
था।
वेद
 ास
क 
त ीर:

यह
 नय त
के
 स ांत
पर
आधा रत
ह।
जैन
धम: Sol
92.
ऋषभ
दव
जैन
धम
के
सं ापक
थे। सही
उ र  वनय
 पटक ह।
वह
पहले
तीथकर
थे।। Key
Points
उ ने
आ ा कता
के
ल 
को
 ा 
 कया
और
 ू सर 
को
मो
या
मु 
पाने
का
सही
तरीका
 सखाया। बौ 
संघ
 ारा
बनाए
गए
 नयम 
के
संदभ
 वनय
 पटक
म
 मलते
ह।
पटक
या
 ी
बा े ट
एक
पारंप रक
श 
ह
 जसका
 योग  व भ 
बौ
Sol
89. धम ंथ 
के
 लए
 कया
जाता
ह।
तीन
 पटक
ह
सु 
 पटक,
 वनय
 पटक
और
अ भध 
 पटक।
मोहनजोदड़ो
क 
खुदाई
1922
म
 सधु
के
तट
पर
लरकाना
 जले
म
क 
गई 1.

स

 ु 
 पटक: 
थी। इसम
बु 
और
उनके
करीबी  श 
से
संबं धत
10
हजार
इसक 
खुदाई
आर.डी.
बनज 
के
नेतृ 
वाली
एक
टीम
ने
क 
थी। से
अ धक
सू 
शा मल
ह।
धनुषको ड
को
वह
 ान
माना
जाता
ह
जहां
भगवान
राम
ने
 ीलंका,
(राम यह
पहली
बौ 
संगी त से
भी
संबं धत
ह
जो
बु 
क 
मृ ु
सेत)ु 
को
जोड़ने
के
 लए
एक
पुल
बनाने
का
आदश
 दया
था।
यह
त मलनाड के
तुरत
ं 
बाद
आयो जत
क 
गई
थी।
म
 त
ह। यह
बु 
के
उपदश
और
उपदश 
का
सं ह
ह।
हड़ ा
क 
खुदाई
1920-1921
म
क 
गई
थी,
जब क
इसक 
खोज
वष
1826 2.
 वनय
 पटक:
म
क 
गई
थी। इसे
बुक
ऑफ
 ड स न
के
नाम
से
भी
जाना
जाता
ह।
कांची
या
कांचीपुरम
त मलनाड
का
एक
 ाचीन
शहर
ह। वनय
 पटक
क 
 वषय
व 
ु भ ुओ
ं और
नन 
के
 लए
मठवासी
 नयम
ह।
Sol
90. यह
संघ और
 भ ओ ु 
ं के
शासी
 नयम 
से
संबं धत
ह।
सही
उ र
कालाशोक
ह। 3.
अ भध 
 पटक:
Key
Points 1.
 यह
बौ 
धम
के
दशन
और
 स ांत
से
संबं धत
ह।
कालाशोक
ने
वैशाली
(383
ईसा
पूव)
म
 ू सरी
बौ 
प रषद
बुलाई। Sol
93.
प रषद
के
प रणाम-
 थ-वा दन
और
महासं घका
म
 व े द। अ ा ायी
प रषद् वष ान राजा अ ता अ ा ायी
 ाकरण
पर
एक
सं ृ त
 ंथ ह।
483
ईसा
पहली
बौ 
प रषद राजगृह अजातश ु ाक प
उपाली
पूव ना 
शा
383
ईसा यह नाटक य
कला
पर
एक
 व ृत
 ंथ
और
पु का ह।
ू सरी
बौ 
प रषद वैशाली कालाशोक सबकामी
पूव
तीसरी
बौ 250
ईसा वृहत्
जातक
पाट लपु अशोका मो लपु 
 त ा
प रषद पूव
वृहत् जातक को वै दक
 ो तष
पर
मानक
पा पु क माना
जाता
ह।
चौथी
बौ 
प रषद 72
ई कुंडलवन कन वासु म
पांचवी
बौ जगा भवंषा
और Sol
94.
1871
ई मांडले म न
प रषद समंगा मा
सुरकोटदा
गुजरात
के
क 
 जले
म
 त
ह।
काबा बमा यह
एकमा 
 सधु
घाटी
 ल
ह
जहाँ
वा व
म
घोड़
के
अवशेष
पाए
गए
ह।
छठी
बौ 
प रषद 1954
ई महासी
सयादव
आये सरकार कालीबंगन
हनुमानगढ़,
राज ान
म
 त
ह।
बनवाली
 हसार,
ह रयाणा
म
 त
ह।
Sol
91. मोहजो
दारो
 सध
म
लरकाना
म
 त
ह।
सही
उ र
वेद
 ास
ह।
भगव ीता: Sol
95.
सही
उ र
व पा ण
ह।
भगवत
गीता
 ह ू 
धम
क 
प व 
पु क 
म
से
एक
ह।
इसे
करीब
पांच
हजार
साल
पहले
महाभारत
का
 ह ा
 लखा
गया
था। बो धस ,
व पा ण,
 ह ू 
धम
के
दवता
इं 
क 
तरह
व 
धारण
करते
ह
'भगवान
का
गीत'
के
नाम
से
मश र
भगवत
गीता
महान
महाका 
महाभारत और
बुराई
और
पाप
के
 खलाफ
लड़ता
ह।
के
700
 ोक 
का
सं ह
ह।
इसम
ऐसी
 श ाएं 
ह,
 जनके
बार
म
कहा
जाता
ह
 क
परमा ा
भगवान अवलो कते र
या
प पा ण
कमल
के
वाहक
ह
ीकृ 
ने
 दए
ह। मजु ी
10
पार मता
(आ ा क
पूणता)
का
वणन
करने
वाली
पु क
रखते
ह।
https://amzn.to/3HVqlZK ाचीन
इ तहास

मै ेय
भ व 
बु 
ह Sol
101.


तभ
नरक- ा णय 
के
बो धस 
और
शु कारक
ह;
जब क
अ मताभ →
सही
उ 
तर
ह
 
महावीर।
ग
के
और
अथाह
जीवन
और
 काश
के
बु 
ह।
महावीर
24
व
जैन
तीथकर
थे।
Sol
96.
मुख
 ब ु
बुजहोम
पुराता क
 ल 
म
ग 
म बसने
के
 माण
ह।
यह
भारतीय
रा 
ज ू
और
क ीर
के
क ीर
घाटी
म
 त
ह। ऋषभदव
पहले
जैन
तीथकर
थे।
बुजहोम
साइट
ने
 नयो ल थक
लोग 
के
मातहत
और
जमीनी
 र
क 
आवास अ जतनाथ
 ू सर
जैन
तीथकर
थे।
सु वधाओं
से
सं मण
को
मेगा ल थक
लोग 
क 
मैड क
संरचनाओं
म
बदल संभवनाथ
तीसर
जैन
तीथकर
थे।
दया। अ भनंदननाथ
चौथे
जैन
तीथकर
थे।
सुम तनाथ
पांचव
जैन
तीथकर
थे।
Sol
97. पा नाथ
23
व
जैन
तीथकर
थे।
सं ृ त
म
 र 
का
अथ
ह
'तीन
र '
अतर 
जानकारी
बु
ध 
(धम):
उनक 
 श ा तीथकर:
संघ:
उन
सभी
का
समुदाय
जो
 श ाओं
का
पालन
करते
ह। तीथकर
' श ण
भगवान'
या
जैन
धम
म
'फोड
 नमाता'
के
 प
म
बु 
धम जाना
जाता
ह।
यह
 व ास
ह
जो
 स ाथ
गौतम
("बु ")
 ारा
 ा पत
 कया
गया जैन
धम
म,
यह
माना
जाता
ह
 क
 ेक
 ांडीय
युग
म
24
था। तीथकर
उ 
होते
ह।
स ाथ
गौतम,
भगवान
बु 
का ज 
563
ईसा
पूव
म
नेपाल
के कला
म
तीथकर 
को
कायो ग
मु ा
(शरीर
को
खा रज
करते
 ए)
लु नी
म
 आ
था। म
 दखाया
गया
ह।
बोध
गया म
एक
पीपल
के
पेड़
के
नीचे
 नवाण
 ा 
 कया
और 24
तीथकर 
को
 तीका क
रंग 
या
 तीक 
 ारा
एक
 ू सर
से
इस लए बु  (एक
 बु ) के
 प
म
जाना
जाता
ह
। अलग
 कया
जाता
ह।
सारनाथ (बनारस)
म
अपना पहला
उपदश  दया,
Sol
102.
जसे ध च व न कहा
जाता
ह।
सही
उ र
 वक 
4
ह,
अथात
क पला।
80
वष
क 
आयु
म
कुशीनगर
(U.P)
म
 नधन।
सां 
ने
भारतीय
दशन
क 
छह
 णा लय 
म
से
एक,
सां 
के
 वषय
म
भी
Sol
98.
बताया।
वेदांग छह
 ह ू 
धम
के
सहायक
 वषय
ह
जो
 ाचीन
काल
म
उ 
 ए
थे
और
वेद 
के
अ यन
से
संबं धत
थे। संत भारतीय
दशन
सबसे
मह पूण
 ाचीन
 ह ू 
दाश नक
 ंथ 
म
से
एक पूव मीमांसा
सू 
ह,
जो संत
क पला सं ा
ऋ ष
जै मनी
 ारा
 लखे
गए
ह।
मह ष
पतंज ल योग
यह
भारतीय
दशन
के
छह
 ढ़वादी
 ू ल 
(दशन)
के
सबसे
पुराने
मीमांसा
का
आधार
ह।
ऋ ष
जै मनी,
परंपरा
के
अनुसार,
महाभारत
के
लेखक
ऋ ष
वेद संत
गौतम ाय
ास
के
 श 
म
से
एक
थे। संत
कणाद
ऋ ष वैशे शका
नौकरी
को
12
अ 
याय
(अ 
याय)
म
 वभा जत
 कया
गया
ह,
जो
आगे
चलकर साठ
पद
(खंड) म
 वभ 
हो
जाते
ह। Sol
103.
सही
उ र
चं गु 
मौय
ह।
Sol
99.
Key
Points
वक 
4
अथात्
लोथल
सही
उ र
ह।
शा ीय
 ीक
 ोत 
म,
चं गु 
मौय
स ोकोटस
को
संद भत
करता
ह।
लोथल
 ु नया
का
पहला
मानव
 न मत
बंदरगाह
था
और
जली
 ई
 ट 
से
च गु 
मौय
(शासनकाल
322
से
298
ईसा
पूव)
पाट लपु 
म
मौय
राजवंश
बना
डॉकयाड।
के
सं ापक
थे।
चाण 
क 
मदद
से
उ ने
नंदा
राजवंश
को
उखाड़
फका।
इसक 
खोज
1953
म
एस.आर.
राव
ने
क 
थी।
उनके
 वशाल
सा ा 
म
 बहार,
बंगाल,
द न,
वतमान
त मलनाड
के
कुछ
यह
गुजरात
म
कै 
े क 
खाड़ी
के
पास
भोगवा
नदी
पर
 त
ह। ह 
े और
उ र
पूव
भारत
के
कुछ
 ह ,े 
पूव 
अफ़गा न ान,
बलू च ान
मोहनजोदड़ो
क 
खुदाई
1922
म
आरडी
बनज 
ने
क 
थी।
यह
 सधु
(अब और
 सधु
नदी
का
 े 
शा मल
था।
पा क ान
म)
नदी
के
दा हने
 कनार
पर
 सध
म
लरकाना
 जले
म
 त
ह।
से ूकस,
 सकंदर
के
जनरल 
म
से
एक
उसका
उ रा धकारी
बन
गया।
द
 ेट
 ैनरी,
द
 ेट
बाथ,
बुन
े ए
सूती
के
टकड़,
 ीटाइट
म
एक
दाढ़ी
वाले से ूकस
ने
मेगा नीज
को
चं गु 
के
दरबार
म
भेजा।

और
कां 
क 
नृ 
करती
 ई
एक
लड़क 
मोहनजोदड़ो
से
पायी
ह।
माना
जाता
ह
 क
चं गु 
मौय
ने
अपने
अं तम
 दन
 वणबेलगोला
म
 बताए
मोहनजोदड़ो
को
'मृतक 
के
पहाड़
(माउं ट
ऑफ़
डड)'
के
 प
म
भी
जाना
जाता थे।
ह।
माना
जाता
ह
 क
संथारा
का
अ ास
करने
से
उनक 
मृ 
ु हो
गई
थी।
Sol
100.
Sol
104.
सही
उ र
कौ ट 
ह
।
Key
Points वक 
म
से,
केवल
काद री
हषवधन
 ारा
नह 
 लखी
गई
ह।
काद री एक
 स 
पु क
ह
जो बाणभ   ारा
र चत
ह।
कौ ट 
को
'इं डयन
मै कयावेली'
भी
कहा
जाता
था।
हषवधन वधन (पु भू त)
वंश
का
शासक
था।
अथशा 
और
सं ृ त
म
राजनी त
पर
एक
पु क
चं गु 
मौय
के
समकालीन
कौ ट 
 ारा
 लखी
गई
थी। Sol
105.
अथशा 
क 
पांड ल प
क 
खोज
सबसे
पहले
1905
म
आर.
शमा
शा ी
ने
क 
थी। इस
काल
म
अनेक
सा ह क
कृ तय 
क 
रचना
 ई।
वह
थे:
नकोलो
 ड
बनाड 
द
मै कयावेली
एक
इतालवी
राजन यक,
दाश नक, का लदास ने
अभजन
शकुंतलम
और
मेघ ू तम जैसे
नाटक
 लखे।
राजनी त ,
इ तहासकार
और
लेखक
थे। भैरवी ने
 करातजुनीय
क  रचना
क   जसम
 करात
और
अजुन
उ 
आधु नक
राजनी तक
दशन
और
राजनी त
 व ान
का
जनक
कहा के बीच
यु 
का
वणन
ह।
जाता
ह। पंचतं 
क 
रचना  व 
ु शमा ने
क ।
यह
 वषय 
या
संदश 
के
साथ
1513
ई ी
म
 लखी
गई
मै कयावेली
क 
 स 
पु क
द
 स
(इल
 सपे) व भ 
कहा नय 
का
संकलन
ह।
थी
https://amzn.to/3HVqlZK ाचीन
इ तहास

Sol
106. Sol
112.
म 
सा ा :
जातक
कथाएँ 
सा ह 
रचनाएं 
ह
जो गौतम
बु 
के
 पछले
ज  के
बार
म
ह।
बौ 
धम: बौ 
धम
एक
मत
ह
 जसक 
 ापना  स ाथ
गौतम ("बु ") यह
कु 
के
पास
म 
भारत
म
 त
था।
ने पाचव 
शता ी
ई.पू. म
क 
थी। इसक 
 ापना
उपरीरा
वासु
के
पु 
म 
 त
ने
क 
थी।
बौ 
धम
अपने
सं ापक
 स ाथ
गौतम
क 
 श ाओं,
जीवन
के
अनुभव
पर उपरीरा
के
अ 
पु 
बृह थ
(मगध
क 
 ापना),
मावेला,
य ु,
म नवाहन
और
आधा रत
ह,
 जनका
ज 
लगभग
563
ईसा
पूव
म
 आ
था। राज 
थे।
म 
क 
राजधानी  वराटनगरी (राज ान
के
जयपुर
 जले
म
वतमान
बैराट)
Sol
107. म
थी।
'माल वका-अ म म्'
कालीदास
 ारा
 लखा
गया
था।
का लदास
एक
शा ीय
सं ृ त
लेखक
और
 ाचीन
भारत
के
महान
नाटक
 लखने
वाले Sol
113.
और
नाटककार
थे। सही
उ र
A
-
II,
B
-
I,
C
-
IV,
D
-
III
ह।
कालीदासा
क 
उ ेखनीय
कृ तयाँ
ह:

अ भ ानशाकु लम्।
 मगध
:
रघुवंशम्।
 यह
वतमान
 बहार
म
 त
था
मेघ ू तम्।
 मगध
क 
राजधानी
राजगृह
ह
।
व मोवशीयम्।
 पहली
बौ 
प रषद
राजगृह
म
आयो जत
क 
गई
थी।
कुमारासंभावाम। व 
:
Sol
108. व 
क 
राजधानी
कौशांबी
ह
।
सही
उ र
 वक 
4)
अथात
मंगो लया
ह।
यह
वतमान
इलाहाबाद
म
 त
था।
व 
रा 
यमुना
नदी
के
तट
पर
 त
था।
माण 
के
अनुसार,
हड़ ा
के
लोग
अफगा न ान,
प सया
और
ओमान
के सुरसेना
:
साथ
 ापार
करते
थे। सुरसेना
क 
राजधानी
मथुरा
ह
।
ले कन
इस
बात
के
कोई
 माण
नह 
ह
 क
वह
मंगो लया
के
साथ
भी
 ापार यह
वतमान
उ र
 दश
म
 त
ह।
करते
थे। क ोज
:
क ोज
क 
राजधानी
राजापुरा
ह
।
व ु
का
नाम े 
(जहां
से
साम ी
आयात
क 
जाती
थी) यह
क ीर
के
 ह ुकुश
पहाड़ 
के
आसपास
 त
था।

चाँदी अफगा न ान Sol
114.


फ रोज़ा प सया
अथववेद
 च क ा
के
 लए
एक
 व कोश
ह।
तांबा ओमान यह मं ,
 ाथना,
तं 
और
भजन 
का
सं ह था।
इसम
 व भ 
 योजन 
के
 लए  ाथनाएं  जैसे
 क फसल 
को
 बजली
से
Sol
109. बचाने
के
 लए,
जहरीले
सांप 
के
 खलाफ,
उपचार
मं ,
 ेम
मं ,
छ , आ द
ह।
हड़ ा
खोजा
जाने
वाला
पहला
 ल
था। तं 
और
मं 
का
उ 
बुराइय 
और
रोग 
को
 ू र
करना ह।
इसे
वष
1921
म
दया
राम
साहनी
क 
एक
टीम
ने
खोजा
था।
मोहनजोदड़ो
क 
खोज
वष
1922
म
आर.डी.
बनज 
क 
एक
टीम
ने
क 
थी। Sol
115.
लोथल,
धोलावीरा
म
एक
खोज
आर
एस
 ब 
क 
एक
टीम
ने
1990
म
क सही
उ र
1,
2
और
4
ह।
थी। Key
Points

Sol
110. भारतीय
उपमहा ीप
म
6व 
शता ी
ईसा
पूव
म
 ू सरा
शहरीकरण
शु


सही
उ र
 वक 
1
ह,
अथात्.
A
-
4,
B
-
2,
C
-
1,
D
-
3। आ।
ऋ ेद सामवेद यजुवद अथववेद आ थक
और
राजनी तक
ग त व ध
का
क 
उ र-प म
से
पूव 
यूपी
और
 बहार
म
 ानांत रत
हो
गया
जो
गंगा
बे सन
ह।
ऋ ेद
चार 
वेद 
म
सबसे
पुराना
ह। सामवेद
संगीतमय इसे
 ाथना
क इसे
जा ू 
क 
 कताब
कहा
बेहतर
उपकरण
और
ह थयार 
के
इ ेमाल
से
 े ीय
रा 
ब त
बड़
हो
ऋ ेद
भजन
और भजन 
का
सं ह पु क
कहा
जाता जाता
ह।
इसम
बुराइय गए
और
इ 
महाजनपद
के
नाम
से
जाना
जाने
लगा।
ाथनाओं
का
एक ह।
गायन
के ह
 क
इसम और
बीमा रय 
को
 ू र
करने मजबूत
शासक
को
स पे
गए
छोट
रा 
या
धीर-धीर
समा 
हो
गए।
सं ह
ह।
ऋ ेद
म शु आती
 माण भजन
और के
 लए
तं 
और
मं
10
मंडल
और
1028 यहां
 मलते
ह। अनु ान
होते
ह। शा मल
ह।
भजन
ह।

Sol
111.
सही
उ र
 मगध
ह
|

मगध
सा ा 
ने
सव थम
यु 
म
हा थय 
का
इ ेमाल
 कया
था।


मगध
के
शासक 
ने
हा थय 
को
बड़
पैमाने
पर
 वनाश
के
ह थयार
के
 प
म
नयु 
 कया
था।
मगध
द णी
 बहार
म
एक
 ाचीन
भारतीय
रा 
था।
मगध
को
सोलह
महाजनपद 
म
 गना
जाता
था।
जैन
और
बौ 
धम
के
 वकास
म
मगध
ने
भी
मह पूण
भू मका
 नभाई।
मगध
रा 
ने
आधु नक
 जल 
को
शा मल
 कया:
पटना
जहानाबाद
नालंदा
औरंगाबाद
अरवल
नवादा
गया।
https://amzn.to/3HVqlZK ाचीन
इ तहास

महाजनपद राजधानी कोट
दीजी
पा क ान
के
 सध
 ांत
म
खैरपुर
से
लगभग
24
 कलोमीटर


द ण
म
 त
ह।
अंग चंपा
यह
मोहनजोदड़ो
के
सामने
 सधु
के
पूव 
तट
पर
ह।
व वैशाली
Additional
Information
म पावा
और
कुशीनगर
काशी बनारस ारं भक
हड़ ा
और
हड़ ा
श 
(1921
म
खोजे
गए
थे)।
ये
 मशः
उस
काल
या
सं ृ तय 
से
संबं धत
ह
जो
 सधु
घाटी
म
शहरी
कौशल ाव ी
और
अयो ा
जीवन
क 
उप त
से
पहले
और
 यं
 सधु
स ता
से
संबं धत
ह।
व कौशा ी
Sol
119.
चेदी सु मती
सही
उ र
महा भ न रमण
ह।
कु इ Key
Points
पंचाल अ हछ और
महा भ न रमण
का
ता य
29
वष
क 
आयु
म
 स ाथ
के
अपने
घर
सुरसेना मथुरा से
चले
जाने
से
ह।
अव ी उ ैन
और
म ह त Important
Points
अ श क/अ क पो म
धमच वतन
सारनाथ
म
 स ाथ
के
पहले
धा मक
उपदश
को
संद भत
गांधार त शला करता
ह।
क ज हटका नवाण
का
ता य
बोधगया
म
बु 
 ारा
 ान
 ा 
से
ह।
प र नवाण
का
ता य
कुशीनगर
म
 स ाथ
क 
मृ ु
से
ह।
म वराटनगर
मगध राजगीर
(बाद
म
पाट लपु ) Sol
120.
सही
उ र
 वक 
1
ह
अथात्
राज ान।
Sol
116. सधु
 ल ल ण
दोहर
दफन
के
सा
जोधपुरा जयपुर
 जले
म
साबी
नदी के
तट
पर
 त
 ल
था,
जहां चावल
क 
भूसी
के
अवशेष
राज ान
म
 ाचीन
स ता
बसी
 ई
थी। लोथल,
गुजरात घोड़
क 
टराकोटा
मू त
के
सा
सा हबी
नदी/साबी
नदी
भारत
म
राज ान,
ह रयाणा
और
 द ी
रा 
से जहाज़
बनाने
का
 ान
होकर
बहने
वाली
एक
अ का लक,
वषा
आधा रत
नदी
ह। एक
जहाज
का
टराकोटा
मॉडल
यह
 द ी
म
यमुना
म
 गरती
ह,
जहां
इसके
चैनल
वाले
पा म
को हल
चले
 ए
भू म
के
सा
नजफगढ़
नाला
भी
कहा
जाता
ह,
जो
नजफगढ़
नाली
प ी
अभयार 
के
 प लकड़ी
के
हल
के
सा
कालीबंगन,
राज ान
म
भी
काय
करता
ह। सात
अ 
वे दय 
के
सा
जुताई
क 
 ई
खेत
क 
सतह
Sol
117.
घोड़
क 
ह याँ
ह ू 
पौरा णक
कथाओं
के
अनुसार,
"ग ड़"
भगवान
 व ु
के
दस
अवतार 
का
 ह ा अंडाकार
क
नह 
ह। सुरकोटदा,
गुजरात
दबे
 ए
 मटटी
के
बतन
मनके
बनाने
क 
 ुकान
ह ू 
पौरा णक
कथाओं
के
अनुसार,
भगवान
 व ु
के
दस
पूण
अवतार
ह
और,
म ी
के
बतन
अन गनत
अवतार
अपूण
सं ा
म
ह। पौध 
के
जीवा
आलम गरपुर,
उ र
 दश
जानवर 
क 
अ याँ
पद अवतार युग तांब
े का
औजार
पहला म सतयुग एक
अनूठी
जल
 बंधन
 णाली
धोलावीरा,
गुजरात
3
भाग 
म
 वभा जत
एकमा 
 ल
ू सरा कुम सतयुग
म 
 दश -
तीसरा वराह सतयुग
चौथा नर सह सतयुग Sol
121.
सही
उ र
महा भ न मण
ह।
पाँचवा वामन सतयुग
मुख-त
छठा परशुराम ेता
महा भ न रमण
का
ता य
29
वष
क 
आयु
म
 स ाथ
के
अपने
घर
से
सातवाँ राम ेता
चले
जाने
से
ह।
आठवाँ कृ ा ापर
Important
Points
नौवां बु क लयुग
दसवाँ क क लयुग धमच वतन
सारनाथ
म
 स ाथ
के
पहले
धा मक
उपदश
को
संद भत
करता
ह।
Sol
118. नवाण
का
ता य
बोधगया
म
बु 
 ारा
 ान
 ा 
से
ह।
↵ प र नवाण
का
ता य
कुशीनगर
म
 स ाथ
क 
मृ ु
से
ह।
सही
उ 
तर
पूव
हड़ ा
स ता
ह।
Key
Points Sol
122.
Key
Points
कोट
दीजी
का
 ाचीन
 ल
 सधु
स ता
या
ह ापन
स ता
से
पहले
पाया
गया
था। ऋ े दक
काल
म
अर णी
वन
क 
दवी
थी।
समय
अव ध:
3300
ईसा
पूव। इं 
को
वषा
का
दवता
माना
जाता
था।
अवशेष 
म
दो
भाग
होते
ह;
उ 
भू म
और
बाहरी
 े 
पर
गढ़
 े । सा व ी
 काश
क 
दवता
थी।
पा क ान
के
पुरात 
 वभाग
ने
1955
और
1957
म
कोट
दीजी
म
खुदाई
क अ द त
अनंत
काल
क 
दवी
थ ।
थी।
https://amzn.to/3HVqlZK ाचीन
इ तहास

Sol
123. माण 
के
अनुसार,
हड़ ा
के
लोग
अफगा न ान,
फारस
और
ओमान
के


साथ
 ापार
करते
थे।
सधु
घाटी
स ता
सबसे
बड़ी
 ाचीन
स ता
ह। ले कन
इस
बात
का
कोई
सबूत
नह 
ह
 क
वे
मंगो लया
के
साथ
भी
 ापार
जॉन
माशल
' सधु
घाटी
स ता'
श 
का
उपयोग
करने
वाले
पहले
 व ान करते
ह।
थे।
सधु
घाटी
स ता
आ 
ऐ तहा सक
युग
(ता पाषाण
युग/कां 
युग)
से साम ी
का
नाम े 
(जहां
से
साम ी
आयात
क 
गई
थी)
संबं धत
ह।
चांदी अफ़ग़ा न ान
दयाराम
साहनी
ने
पहली
बार
1921
म
हड़ ा
स ता
क 
खोज
क ।
फ़रोज़ा फारस
Sol
124.
तांबा ओमान
सही
उ र
लोथल
ह।

इसे
यूने ो
 ारा
 व 
धरोहर
 ल
के
 प
म
ना मत
 कया
गया
ह। Sol
128.


यह
गुजरात
रा 
के
भील
 े 
म
 त
ह।
चंपानेर-पावागढ़
पुरात 
पाक
गुजरात
म
पंचमहल
 जले
म
 त
ह। 
परी त
और
जनमेजय
कु 
सा ा 
के
 स 
शासक
थे।
भीमबेटका
गुफ़ाएँ 
(भीमबेटका
रॉक
शे र
या
भीमबैठका)
भारत
के
म - ऋ ै दक
काल
के
दौरान
आ दवासी
रा 
-
भरत,
म ,
य ु 
और
पु ।
दश
रा 
के
रायसेन
 जले
म
एक
पुरापाषा णक
पुराता क
 ल
ह। बाद
के
वै दक
 ंथ 
के
अनुसार
भारत
के
तीन
 भाग
ह
-
आयावत
(उ री
भारत),
म दश
(म 
भारत)
और
द णापथ
(द ण
भारत)।
Sol
125.
Sol
129.
ऋ ेद
के
पु ष सू 
म
वै दक
समाज
के
 वभाजन
का
 
उ ेख
ह।

पु ष
सू  वण
 व ा
और
पदानु मत क 
उ 
क 
 ा ा
करता
ह। 'रघुवंशम' का
संकलन का लदास  ारा
 कया
गया
ह।


ऋ ेद
के
10व
मंडल म पु ष
सू  का
उ ेख
ह, 'रघुवंशम'
 जसे 'रघुवंश' के
नाम
से
भी
जाना
जाता
ह,
का लदास
 ारा
र चत
एक
सं ृ त
महाका 
ह।
Sol
126. का  रघु
वंश के
बार
म 19
सग  म
बोली
जाती
ह,
 जसम रघु,
दशरथ,
सही
उ र
 वक 
3
ह। राम आ द
शा मल
ह।
पा ण न
 ारा
सं ृ त
भाषा
का
पहला
 ाकर णक
पाठ
 लखा
गया
था।
Sol
130.
पा ण न
 ाचीन
भारत
म
एक
सं ृ त ,
 ाकरण
के
 व ान
थे।
पा णनी
का
 ाकरण
एक
आदश
 त प ह,
 जसे
'अ ा ायी'
के
 प
म गु 
काल को भारतीय
इ तहास
के
' ण
युग' के
 प
म
व णत
 कया
गया
जाना
जाता
ह। ह।
'अ ा ायी'
वै दक
भाषा
के
अ धक
 नकट
ह। गु 
सा ा 
क 
समृ 
ने
सां ृ तक
और
वै ा नक
 ग त
के
 ण
युग का
पा ण न
क   ाकरण
5व 
शता ी
और
चौथी
शता ी
ई.पू.
के
बीच
क नमाण
 कया।
अव ध
क 
थी। गु 
सा ा , महाराजा
 ी
गु ा  ारा
 ा पत,
एक
 ाचीन
भारतीय
 े 
था,
का लदास
एक
शा ीय
सं ृ त
लेखक
ह,
 ज 
भारत
के
शे पयर
के जो 320-550
ई.पू.
से
भारतीय
उपमहा ीप का
अ धकांश
भाग
प
म
जाना
जाता
ह। शा मल करता
था।
"अ भ ानशाकु लम"
का लदास
 ारा
 स 
सं ृ त
नाटक
ह।
12व 
शता ी
म
सं ृ त
म
"राजतरं गणी"
क ण
 ारा
 लखी
गई
थी। Sol
131.

Sol
127. सोना ग र म 
 दश
रा 
का
एक
 ान
ह
जो द तया  जले
म
 त
ह।


जगत
प त
जोशी
(जेपी
जोशी)
एक
भारतीय
पुरात वद्
ने
धोलावीरा
के
 सधु इस
 ान
पर 9व  शता ी
से
ब त
सार
जैन
मं दर
 ा पत
 ए 
ह।
घाटी
साइट
क 
खोज
क 
ह। भ 
और
तप ी
संत 
के
बीच
यह
 ान
आ -अनुशासन,
तप ा
और
आधु नक ारा नदी
का मो 
 ा 
करने
के
 लए
लोक य
ह।
साइट पुराता क
खोज एक
 स  जैन
सं हालय भी
सोना ग र म
 त
ह।
ान अ व ृ त कनारा
ेट
 ैनरी,
 ेट
बाथ,
दाढ़ी
वाले
 
क Sol
132.
मोहनजोदड़ो सधु
शैलखटी
छ व, न
 ांग:
ुग
के
 बना
शहर
के
 प
म
भी
जाना
एन.जी. एक
चीनी
बौ 
 भ ु,
 व ान,
या ी
और
अनुवादक
 ज ने
सातव 
शता ी
म
च दड़ो जाता
ह
,
 लप क,
इं कपॉट,
मॉडल सर ती
मजूमदार भारत
क 
या ा
क ।
ऑफ
बुलॉक
गाड़ी
उ ने
शु आती तांग
राजवंश
के
दौरान
चीनी
बौ 
धम
और
भारतीय
बौ 
धम
यह
एक
डॉकयाड
था,
एक
घोड़
क गुजरात
का के
बीच
बातचीत
का
वणन
 कया।
लोथल टराकोटा
मू त,
डबल
दफन,
चावल
क अहमदाबाद भोगवा
वह
भारत
म
अपनी स ह
साल
क 
ओवरलड
या ा
के
 लए
 स 
 ए।
भूसी जला
उ ने
हषवधन
के
शासनकाल
के
दौरान
भारत
का
दौरा
 कया जो
बौ 
धम
के
जेपी त
उनक 
गहरी
भ 
के
 लए
उनक 
 शंसा
करने
आए
थे।
एक
अ तीय
जल
दोहन
 णाली,
जल गुजरात
का
धोलावीरा जोशी,
नकासी
 णाली,
एक
 े डयम क 
 जला
1966 Sol
133.
ह रयाणा
का आरएस
बनवाली फतेहाबाद ब हषवधन
एक
भारतीय
स ाट
थे,
 ज ने
606
ई ी
से
647
ई ी
तक
भारत
जला (ASI) के
उ री
 ह 
पर
शासन
 कया
उनक 
 ारं भक
राजधानी
थाने र
थी,
वतमान
ह रयाणा।
[गलती
 ब ु]
पा क ान उनके
आगमन
के
बाद,
राजा
हषवधन
ने
थानेसर
और
क ौज
के
दो
रा 
को
एक
पं 
म
अ 
भंडार,
दवी
क 
 म ी 1921,
का एकजुट
 कया।
हड़ ा क 
आकृ त,
तांबे
के
पैमाने,
 लगम
के दया
राम रव
साहीवाल
प र
के
 तीक
आ द। साहनी उ ने
अपनी
राजधानी
को थानेसर
से
क ौज भी
 ानांत रत
कर
 दया।
जला
इस
 कार,
7व 
शता ी
सीई
म,
हषवधन
के
सा ा 
क 
राजधानी क ौज
जेपी थी।
सुरकोटदा जोशी,
1966 Sol
134.

सुरकोटदा
म
घोड़
क 
ह य 
के
अवशेष
पाए
गए
थे।
https://amzn.to/3HVqlZK ाचीन
इ तहास

यह
 सधु
घाटी
 ल
ह
जो
वतमान
म
गुजरात
म
 त
ह।
 Key
Points

सधु
घाटी क न 
ने
राजा
हान
हो-टी
के
 खलाफ
लड़ाई
लड़ी,
जो
चीन
के
हान
वंश
के


मह राजा
थे
,
और
 ू सर
 यास
म
उ 
हरा
 दया।
साइट

कन :
यह
नवीनतम
साइट
ह
जो
गुजरात
म
 त
ह।
दो
भाग 
वाले
अ 
साइट 
के
 वपरीत
इसे
3
भाग 
म
 वभा जत वह
कुषाण
वंश
का
एक
स ाट
था
और
 ू सरी
शता ी
(c.120–144
CE)
म
धोलावीरा
कया
गया
था। त त
था।
नचले
शहर
के
अलावा
एक
म 
शहर
 त
ह।
कुषाण
स ाट 
ने
पहली
बार
चांदी
के
 ान
पर
सोने
के
 स 
का
उपयोग
 कया
था
उनके
सा ा 
क 
मु 
राजधानी
गांधार
के
पु अपुरा
म
 त
थी
।
इसम
एक
कृ म
 ट
डॉकयाड
ह।
उसने
द णी
उजबे क ान
और
ता ज क ान
से
लेकर
उ री
भारत
तक
इसम
चावल
क 
शु आती
खेती
के
 माण
ह।
लोथल फैले
एक
सा ा 
पर
शासन
 कया,
जहाँ
तक
द ण-पूव
म
मथुरा
था
यह
 सधु
घाटी
के
लोग 
के
 लए
एक
बंदरगाह
का
काय
करता
था। वह
कुषाण
सा ा 
के
सं ापक
कुजुला
कड फसेस
के
वंशज
थे
।
वह
बौ 
धम
के
संर क
थे
और
गांधार
से
चीन
तक
महायान
बौ 
धम
के
सारण
म
मह पूण
भू मका
 नभाई
थी
।
सधी
भाषा
म,
इसका
अथ
ह
"मृतक 
का
टीला"। उ ने
प रषद
के
 मुख
के
 प
म
क ीर
म
4
व
बौ 
प रषद
का
संचालन
यह
सभी
 सधु
शहर 
म
सबसे
बड़ा
ह। कया।
मोहनजोदाड़ो द
 ेट
बाथ
भी
यहाँ
 त
ह। इसक 
अ ता
वसु म 
और
अ घोष
ने
क 
थी।
कां 
क 
नृ ांगना
और
पशुप त
क 
एक
मोहर
भी
यहाँ
पाई
गई 32
भौ तक
 च 
पर
आधा रत
बु 
के
 च 
उनके
समय
के
दौरान
बनाए
गए
ह। थे।

Sol
140.
Sol
135.
जैन
धम
म, तीथकर एक
उ ारकता
ह
इ ोने
जीवन
क 
पुनज 
क 
धारा
को
मुख
 शलालेख
13 क लग
के
साथ
उनके
यु  के
बार
म
 ववरण
दता
ह। पार
करने
म
सफल
रह
ह
और
 ू सर 
के
अनुसरण
के
 लए
एक
माग
बनाया
ह।
चौदह
 मुख
 शलालेख ह। जैन
धम
म 24
तीथकर ह।
दो
क लग
लेख
नए
 व जत
 े 
म
पाए
जाते
ह। पा नाथ
23व
तीथकर
थे
मुख
 ंभ
लेख
को
मह पूण
शहर 
म
 ा पत
 कये
गए थे। महावीर अं तम
तीथकर
थे।
सू 
 शला
लेख और सू   ंभ
लेख ह।
Sol
141.
Sol
136.
ना सक
 शलालेख 
पर
गौतमी
पु 
सातकण 
क 
उपल यां
दज
ह।
यह
त मल
भाषा
के
 ाकरण
और
त मल
सा ह 
और
भाषा
 व ान
के
 े वो
सातवाहन
राजवंश
से
थे।
म
 ारं भक
काय
ह। ना सक
के
 शलालेख 
क 
आधार शला
उनक 
मां
गौतमी
बाल ी
ने
रखा
था।
इसे
त मल
म
टो पयार
ने
 लखा
था। शलालेख नागापण
पर
उनक 
जीत
के
बार
म
दशाता
ह।
यह त मल
 ाकरण पर
एक
काय
ह। सातवाहन
राजवंश
के
स हव
राजा,
हाल
अपनी
पु क
गाथास सती
के
यह
उस
समय
के
राजनी तक
और
सामा जक
प र 
के
 वषय
म
भी
बताता लए
 स 
थे।
ह। सातवाहन
राजवंश
के
सं ापक
 समुक
थे।
त मल
भाषा
का
 योग
संगम
सा ह 
म
 कया
गया
था।
Sol
142.
Sol
137. सही
उ र
ह,
चीनी
बौ 
 भ 
ु ेन
 ांग।
सही
उ र
 व ा म 
ह। Key
Points
Key
Points
चीनी
बौ 
 भ 
ु न
 ांग
ने
पुलके शन
 तीय
के
शासनकाल
के
दौरान
व ा म 
गाय ी
मं 
के
रचनाकार
ह। चालु 
सा ा 
का
दौरा
 कया
और
अपने
लेखन
म
उनक 
 शंसा
क ।
गाय ी
मं 
पहली
बार
ऋ ेद
म
 दखाई
 दया। चीनी
तीथया ी
 न
 ांग
ने
सातव 
शता ी
म
भारत
का
दौरा
 कया
और
इसे
सा व ी
मं 
के
 प
म
भी
जाना
जाता
ह। पं ह
वष 
तक
रह।
हष
के
शासनकाल
के
दौरान,
भारत
प च ं ने
म
लगभग
16
व श 
ने
इस
भजन
क 
रचना
क ,
जो
अ ,
इं 
और
अ 
दवताओं
को वष
लग
गए।
सम पत
ह,
। न
 ांग
ने
वणन
 कया
 क
भारत
को
तीन
वण 
 ा ण,
 य
और
शू 
(जो
वै दक
और
उ र
वै दक
 ंथ 
म
गाय ी
मं 
का
 ापक
 प
से
उ ेख
 कया कृ ष
करते
ह)
म
 वभा जत
 कया
गया
था,
इन
तीन 
के
साथ
अ त र 
वण
गया
ह। भी
थे
 ज 
अछत
के
 प
म
जाना
जाता
था।
यह
मं 
उपनयन
समारोह
का
एक
मह पूण
 ह ा
था। हष
के
शासनकाल
और
 न
 ांग
या ा
के
दौरान,
उ ने
दज
 कया
 क
बौ
गो ामी
तुलसीदास
हनुमान
चालीसा
के
संगीतकार
थे। धम
को
नालंदा
के
साथ
अठारह
सं दाय 
म
 वभा जत
 कया
गया
था,
जहां
इसके
 स 
क 
ने
10000
 भ ुओ
ं को
महायान
 ू ल
के
बौ 
दशन
क
Sol
138. श ा
दी
थी।
670
ई.
म
एक
अ 
चीनी
तीथया ी
ने
भारत
का
दौरा
 कया-
आई-
 ग-
भारत
म फा- हएन क 
या ा
चं गु 
 तीय
के
शासनकाल
के
दौरान
 ई
थी। जसने
नालंदा
को
भी
अपने
या ा
वृ ांत
म
दज
 कया।
फा- हएन एक
चीनी
बौ 
 भ ु
और
अनुवादक
थे
 ज ने
चीन
से
भारत
तक
पैदल
या ा
क ,
म 
ए शया
म
प व 
बौ 
 ल 
का
दौरा
 कया,
और
399- Additional
Information
412
के
समकालीन
भारतीय
उपमहा ीप
और
द ण
पूव
ए शया
के
बीच दरबारी
बौ 
 ंथ 
का
अ ध हण
 कया
था। राजा राजवंश सा ह कार
और
लेखक
कव
उ ने
अपने
या ा
वृतांत 'ए
 रकॉड
ऑफ़
बु 
 कगड ' म
अपनी
या ा
पुलके शन
का
वणन
 कया
ह। चालु 
वंश र वकर त ऐहोल
 शलालेख
तीय


चं गु 
 तीय,
 जसे  व मा द  भी
कहा
जाता
ह,
उ री
भारत
के
श शाली
स ाट
(380-415
ई.पू.)
थे। पु भू त
वंश
/
वधन बाणभ -हषच रत
हषवधन बाणभ 
 हषवधन- यद शका,
र ावली
और
वंश
Sol
139. नागानंद
सही
उ र
ह
क न 
। चालु 
वंश:
https://amzn.to/3HVqlZK ाचीन
इ तहास

पुलके शन
I
चालु 
वंश
का
सं ापक
था। धमेख
 ूप
क 
आकृ त
 ट 
और
प र
से
बनी
एक
ठोस
बेलनाकार


उसने
बादामी
(वातपी)
म
अपनी
राजधानी
 ा पत
क । संरचना
ह
जो
28
मीटर
के
 ास
के
साथ
43.6
मीटर
क 
ऊंचाई
तक
ह।
क तवमन
I
पुलके शन
I
का
पु 
था।
वजया द 
चालु 
वंश
का
सबसे
लंब
े समय
तक
शासन
करने
वाला
राजा Sol
150.
था। सही
उ र
 वक 
3
अथात्
पाट लपु 
ह।
पुलके शन
II
ने
प व
वंश
के
मह वमन
I
को
हराया। रण
करने
यो 
मह पूण
 ब ु :
चालु 
वंश
के
अं तम
राजा,
क तवमन
 तीय
को
द ुग
ने
हराया
था। गु 
सा ा :

Sol
143. गु 
वंश
क 
राजधानी
पाट लपु 
थी।


गु कालीन
 स ा
भी
उ ेखनीय
था।
समु गु 
ने
आठ
 कार
के
सोने
के
ऐहोल
 शलालेख
र वक त  ारा
 लखा
गया था
जो
पुलकेशी
 तीय के स े 
जारी
 कए।
शासनकाल
के
दौरान
एक
क व
था। चं गु 
 तीय
और
उसके
उ रा धका रय 
ने
 व भ 
 क 
के
सोने,
चांदी
ऐहोल
कनाटक
म
 त
ह
और
 ापक
 प से
भारतीय
वा ुकला
के
पालने और
तांबे
के
भी  स े 
जारी
 कए
थे।
के
 प
म
पहचाना
जाता
ह। च गु 
 तीय
का
दरबार
 त त
नवर 
 ारा
सुशो भत
था।
का लदास
इसने
चालु  क 
राजधानी
के
 प
म
काय
 कया। उनम
सबसे
अ णी
ह।
यह
 शलालेख
पुलके सन
क 
 वजय
के
बार
म
जानकारी
दता
ह,
 वशेष
 प पंचतं 
क 
कहा नय 
क 
रचना
गु 
काल
के
दौरान
 ई
थी।
से
उसने
हषवधन
को
कैसे
हराया। उनके
वतमान
 प
के
पुराण 
क 
रचना
इसी
काल
म
 ई
थी।
इस
अव ध
के
दौरान
महाभारत
और
रामायण
को
वतमान
 प
म
 लखा
गया
Sol
144. था
और
इसे
अं तम
 प
 दया
गया
था।
सही
उ र
 वक 
4
अथात्
वासुदव
ह।
Sol
151.
वासुदव
क 
वंश
के
सं ापक
थे। मेग नीज
वासुदव
अं तम
सुंग
शासक,
दवभू त
के
मं ी
थे।
वासुदव
ने
दवभू त
क 
ह ा
क 
और
क 
वंश
क 
 ापना
क । ीक
शासक से ूकस
 थम
 नकेटर और
भारतीय
शासक चं गु 
मौय के
क 
वंश
क 
राजधानी
पाट लपु 
थी। बीच
सं ध
के
समय,
वह
 स ब रयस
के
तहत
एक
अ धकारी
के
 प
म
काय
दामन
I
शक
वंश
से
संबं धत
थे
और
उ ैन
पर
शासन
करते
थे। कर
रह
थे।
समुका
सातवाहन
वंश
का
सं ापक
था। मेग नीज एक
 ाचीन
यूनानी
इ तहासकार,
राजन यक
और
भारतीय
खारवेल
चे द
वंश
से
संबं धत
थे
और
क लग
पर
शासन
करते
थे। नृवंश व ान
 वशेष ,
और
हले न क
काल
म
खोजकता
थे।
मेग नीज ने "इं डका" पु क
 लखी।
Sol
145.
Sol
152.
अल- ब नी
ने तराईख
अल- हद (भारत
का
इ तहास)
 लखा
ह,
जहां
वह
लखते
ह
"थानेसर
शहर
 ह ुओ
ं ारा
ब त
स ा नत
ह।
उस
 ान
क 
मू त ा ु ट स ांत  गु  का
मु 
काम
ह,
सी
 लखा
 आ।
628।
को
च ा मन
कहा
जाता
ह,
अथात
च 
का
मा लक,
एक
ह थयार
 जसे ग णतीय
खगोल
 व ान
के
इस
पाठ
म
मह पूण
ग णतीय
साम ी
ह।
हमने
पहले
ही
व णत
 कया
ह।
।
यह कां  का
ह,
और
लगभग
एक
आदमी यह
सकारा क
सं ा,
नकारा क
सं ा
और
शू 
के
बार
म
मू वान
का
आकार
ह।" वचार
 दान
करता
ह।
गु   ाचीन
भारतीय
खगोल वद 
के
सबसे
 नपुण
 य 
म
से
एक
थे।
Sol
146.
Sol
153.
कुल
108
उप नषद
 ात
ह
 जनम
से
11
मु 
उप नषद
ह। सही
उ र
समहता
ह।
इ 
मु 
उप नषद
के
नाम
से
जाना
जाता
ह।
अं तम
उप नषद
1656
म
दारा
 शकोह
 ारा
दज
 कया
गया
था,
जो
शाहजहाँ समहता
राज 
के
मु 
सं हकता
थे,
जो
चाण 
के
अथशा 
के
का
पु 
था। अनुसार
मौय
सा ा 
से
राज 
के
सं ह
क 
 नगरानी
करते
थे।

Sol
147. Key
Points

सधु
घाटी
 ल खोजकता खोज
का
वष दश ी
को
संभागीय
आयु 
के
 प
म
जाना
जाता
था।


लोथल शकारीपुरा
रंगनाथ
राव 1957 वे
आधु नक
 जला
म ज ट
और
 जले
के
 भारी
थे।
वे
हर
5
साल
म
एक
बार
दौरा
करते
थे
ता क
 नयं ण
वाले
 े 
के
पूर
कालीबंगा ए.
घोष,बी
. व
लाल,बी.के
थ र 1953
शासन
का
 नरी ण
 कया
जा
सके।
च दारो गोपाल
मजूमदार 1931 आं तीपापाल
को
सीमांत
के
रा पाल
के
 प
म
जाना
जाता
था।
रोपड़ वाई.
डी.
शमा 1953
Sol
154.
Sol
148.
गु 
राजवंश
के
कुमारगु 
 थम
 ारा
5व 
शता ी
म
नालंदा
महा वहार
व यान
 ू ल
बौ 
धम
से
संबं धत
ह। क 
 ापना
क 
गई
थी।
व यान
का
अथ
ह
"दी
 ीकल
ऑफ
द
थंडरबो "।
यह
बौ 
 ू ल
7व 
से यह
एक
 ाचीन
महा वहार,
बौ 
मठ
था।
8व 
शता ी
सीई
के
बीच
भारत
म
 वक सत
 आ। गु 
सा ा 
क 
 ापना
 ी
गु 
ने
क 
थी।
इसे
तां क
बौ 
धम
के
 प
म
भी
जाना
जाता
ह,
यह
पहली
बार
भारत
और चं गु 
I
गु 
सा ा 
का
पहला
महान
शासक
था,
 जसने
महाराजा धराज
ीलंका
के
 व भ 
 ह 
म
उभरा। क 
उपा ध
धारण
क ।
यह
महायान
परंपरा
के
 स ांत 
पर
आधा रत
ह। समु गु 
गु 
सा ा 
का
 ू सरा
शासक
था।
उ 
भारत
का
नेपो लयन
भी
कहा
जाता
था।
Sol
149.
Sol
155.
मौय
राजा
अशोक
 ारा
बनाया
गया
धमेख
 ूप,
सारनाथ
म
 त
ह
जो
वाराणसी
से
13
 कमी
 ू र
ह। ऋ ेद
म
नमदा
नदी
का
उ ेख
नह 
ह।
धमेख
 ूप
को
उस
 ान
को
 च त
करने
के
 लए
कहा
जाता
ह
जहाँ
बु 
ने ऋ ेद
म
व णत
न दयाँ
 सधु,
झेलम,
 चनाब,
र व,
 ास,
सतलुज,
गोमती,
आ ान
 ा 
करने
के
बाद
अपने
पांच
 श 
को
पहला
उपदश
 दया कुरम,
घ र
और
 ात
ह।
था। ऋ ेद
एक
 ाचीन
भारतीय
 
ह
जो
1,028
वै दक
सं ृ त
भजन 
और
दवताओं
को
सम पत
10,600
छं द 
का
सं ह
ह।
https://amzn.to/3HVqlZK ाचीन
इ तहास

Sol
156. उ ने
1014
ई।
म
अपने
 पता
राजराजा
चोल
I
को
उ रा धकारी


बनाया।
383
ईसा
पूव
म वैशाली म
 ू सरा
बौ 
प रषद
आयो जत
 कया
गया
था। चं गु 
 तीय
गु 
सा ा 
के
सबसे
श शाली
स ाट 
म
से
एक
था।
वैशाली
अब
उ र
भारत
म
 बहार
रा 
का
 ह ा
ह उ 
लोक य
 प
से
" व मा द "
के
 प
म
जाना
जाता
था।
ू सरी
बौ 
प रषद
राजा कालाशोका के
संर ण
म
 ई
और
इसक
अ ता साबकामी ने
क 
थी। Sol
161.
कुल
 मलाकर,
6
बौ 
प रषद
बुलाई
ग 
थी।
अथववेद उन
चार
वेद 
म
से
एक
ह
 जसम
जा ुई
अनु ान 
और
आकषण
के
Sol
157. बार
म
उ ेख
ह।
अथववेद
को
कभी-कभी
अ 
 व ान 
 ारा
गलत
घो षत
 कया
गया
"जा ुई
शाही
चोलाओं
के
अधीन
राज 
 शासन
के
संबंध
म
'शालाभोग'
श 
का सू 
का
वेद"
कहा
जाता
ह।
अथ
 व ालय
के
रखरखाव
के
 लए
दान
क 
गई
भू म
से
ह|
त मलनाड
म
शासन
करने
वाले
चोल 
के
 शलालेख 
से
पता
चलता
ह
 क Sol
162.
उनके
रा 
ने
 व भ 
 कार
के
कर 
के
 लए
400
से
अ धक
पद 
का
उपयोग
 कया
था। धोलावीरा
लूनी
नदी
के
तट
पर
 त
ह।
चोल
द ण
भारतीय
उपमहा ीप
म
सबसे
स 
जा त
के
थे। धोलावीरा
शहर
तीन
भाग 
म
 वभा जत
था।
गु 
सा ा 
के
काल
को
अ र
"भारत
का
 ण
युग"
कहा
जाता
ह। लोथल
भोगवा
नदी
के
तट
पर
 त
था।
चोल
 शलालेख 
के
अनुसार,
चोल
राजाओं
ने
अपनी
 जा
को
पाँच
 कार
के मोहनजोदड़ो
 सधु
नदी
के
तट
पर
 त
था।
'भू म
उपहार'
को
 दए
थे:
शालाभोग
:
यह
 व ालय
के
रखरखाव
के
 लए
दान हड़ ा
रावी
नदी
के
तट
पर
 त
था।
क 
गई
भू म
ह,
दवदान:
मं दर 
को
दान
म
दी
गई
भू म,
प ीचंदम:
जैन
सं ान 
को
दान
म
दी
गई
भू म,
 दय-
 ा ण 
को
उपहार
म
दी
गई
भू म Sol
163.
थी। सही
उ र
एक
 कार
क 
बंधुआ
मज ू री

Sol
158. द ण
भारत
म
 ाचीन
काल
के
दौरान
 व 
एक
 कार
क 
बंधुआ
मज ू री


कालीबंगन,
 सधु
घाटी
स ता
 ल 
पर
जुताई
का
 े 
और
एक
बैल
क 
कां थी।
आकृ त
पाई
गई
ह। व 
का
उपयोग
राजा
 ारा
कर
के
 प
म
 कया
जाता
था
जो
 कसान 
को
सधु
घाटी
स ता
 ल
पूर
 सध,
बलू च ान,
लगभग
पूर
पंजाब,
उ री
राज ान, करना
पड़ता
था।
का ठयावाड़
और
गुजरात
म
फैले
 ए
ह। द ण
भारतीय
 ाचीन
इ तहास
के
दौरान,
 ामीण 
को
सेना
को
पास
करने
और
अ धका रय 
का
दौरा
करने
के
 लए
 व भ 
 कार
क 
मु 
आपू त
Sol
159. दान
करने
के
 लए
मजबूर
 कया
गया
था।
सही
उ र
 वक 
2
ह
अथात्
जैन
धम। कर
का
भार
भू म
के
कर,
अनाज,
सोना,
फूल,
 ू ध
के
दान
के
 प
म
कृ ष
और
बेगार
 मक
( व 
के
 प
म
जाना
जाता
ह)
पर
लगाया
गया
था
ानकवासी:
यह
 ेतांबर
जैन
धम
का
एक
सं दाय
ह। Sol
164.
इसक 
 ापना
1653
ई.
म
लवाजी
नामक
एक
 ापारी
ने
क 
थी। गौतमीपु 
सातकण 
:
ानकवासी
मू त
पूजा
म
 व ास
नह 
करते
ह।
उनका
मानना
ह
 क
आ ा
क 
शु 
और
 नवाण/मो 
क 
 ा 
के वह
सातवाहन
वंश
का
23
वां
शासक
था।
लए
मू त
पूजा
आव क
नह 
ह। उसक 
उपल य 
का
उ ेख
उसक 
माता
गौतमी
ने
ना सक
 शलालेख
म
कया
ह।
Sol
160. उसने शक
राजा
नहपान
को
परा जत
 कया और सातवाहन
श  को
सही
उ र
हषवधन
ह। पुनज वत
 कया।
Key
Points
Sol
165.
हषवधन
भारत
के
सबसे
मह पूण
स ाट 
म
से
एक
थे। सही
उ र
इ ा हम
आ दल
शाह
II
ह।
उ ने
606
से
647
ई ी
तक
उ र
भारत
पर
शासन
 कया। Key
Points
वह
पु भू त
राजवंश
के
सबसे
सफल
स ाट 
म
से
एक
थे।
पु भू त
वंश
को
वधन
वंश
के
 प
म
भी
जाना
जाता
ह। इ ा हम
आ दल
शाह
 तीय
बीजापुर
स नत
के
राजा
थे।
गु 
सा ा 
के
पतन
के
बाद
यह
 मुखता
म
आया। वे
आ दल
शाही
वंश
के
सद 
थे।
भाकर
वधन
पु भू त
वंश
का
पहला
राजा
था। उनके
पास
द न
के
'अबला
बाबा'
(गरीब 
के
दो )
क 
उपा ध
थी।
7
व 
शता ी
के
 ारंभ
के
साथ,
हषवधन
ने
थाने र
और
क ौज
के उनके
पास
'जग 
बादशाह'
नामक
एक
उपा ध
थी।
सहासन
पर
चढ़ाई
क । आ दल
शाही
वंश
के
शासनकाल
म
इसक 
सबसे
बड़ी
अव ध
थी।
वे
16
साल
क 
उ 
म
वधन
वंश
के
 सहासन
पर
बैठ। उसने
1580
से
1626
तक
राजवंश
पर
शासन
 कया।
उसने
उ र
भारत
से
लेकर
म 
भारत
म
नमदा
नदी
तक
अपने वह
भारतीय
संगीत
के
 वशेष 
थे
और
इस
 वषय,
नौरस-नामा
पर
पु क
के
सा ा 
का
 व ार
 कया। लेखक
थे।
उसका
 शासन
गु 
सा ा 
के
 शासन
के
समान
था। उ ने
अपने
 कले
म
द ा
मं दर 
का
जीण ार
कराया
और
पूजा
अनु ान
स 
चीनी
या ी
जुआनज़ग
ने
अपने
काल
म
भारत
का
दौरा को
 ो ा हत
 कया।
कया। उ ने
 ह ू -मु म
के
सां ृ तक
स ाव
के
 लए
काम
 कया।
उ 
एक
द ण
भारतीय
शासक
पुलके शन
 तीय
ने
हराया
था। उ ने
' कताब-ए-नवरस'
नामक
एक
पु क
 लखी
और
संगीत
को
उनक 
जीवनी
"हषच रत"
 स 
क व
बाणभ 
 ारा
 लखी
गई
थी। मुसलमान 
के
बीच
लोक य
बनाने
क 
को शश
क ।
643
ई.
म
एक
भ 
बौ 
दी ांत
समारोह
के
आयोजन
के
 लए 'इ ा हम
रौज़ा'
जो
अपनी
वा ुकला
के
 लए
 स 
ह
उसे
इ ा हम
आ दल
भी
वे
 स 
थे। शाह
II
 ारा
बनाया
गया
था।
वह
वष
647
सीई
म
प व 
 नवास
के
 लए
रवाना
 आ।
Additional
Information
Additional
Information
अली
आ दल
शाह
 थम
बीजापुर
स नत
के
पांचव
सु ान
थे।
कृ दवराय
 वजयनगर
सा ा 
के
स ाट
थे। उ ने
मुगल
स ाट
अकबर
के
साथ
राजन यक
संबंध
 ा पत
 कए
उसने
1509
से
1529
तक
 वजयनगर
सा ा 
पर
शासन
 कया। और
 ू त 
का
आदान- दान
 कया
गया।
वह
तुलुवा
राजवंश
का
तीसरा
और
सबसे
बड़ा
शासक
था। मीर
जुमला
 तीय
औरंगजेब
के
अधीन
बंगाल
के
एक
 मुख
सूबेदार
थे।
राज 
चोल
 थम
चोल
सा ा 
के
सबसे
महान
शासक 
म
से
एक
था। उ ने
जनवरी
1662
म
अहोम
सा ा 
पर
आ मण
 कया।
इ ा हम
कुतुब
शाह
गोलकुंडा
रा 
के
चौथा
शासक
थे।
https://amzn.to/3HVqlZK ाचीन
इ तहास

वह
"सु ान"
शीषक
का
उपयोग
करने
वाले
कुतुब
शाही
वंश
के यह
फतेहाबाद
 जले
म
 त
ह
और
757-6200
ईसा
पूव
पूराना
ह।


पहले
 
थे। भराना
के
करीब
 त
राखीगढ़ी
 ु नया
का
सबसे
बड़ा
हड़ ा
 ल
ह।

Sol
166. Sol
172.
छं द ह
- सही
उ र
 वक 
1)
ह
अ नी।
"11 जब
उ ने
पु ष को
 वभा जत
 कया
तो
उ ने
 कतने
 ह े
 कए? आधु नक- दन
का
नाम ऋगवै दक
नाम
वे
उसके
मुंह,
उसक 
बाह 
को
 ा
कहते
ह?
वे
उसक 
जाँघ  और
पैर 
को
 ा
कहते
ह? चेनाब अ नी
12.
 ा ण
उसका
मुंह
था,
उसक 
दोन 
भुजा
राज  थी। रावी पु शनी
उनक 
जांघ
वै 
बन
ग ,
उनके
चरण 
से
शू 
उ 
 ए।”
यहाँ
पु ष
कभी
सव ,
सवश मान,
सव 
होने
का
उ ेख
करता
ह। झेलम वत ा
ास वपाशा
LEVEL
3 167
-
249
Questions
Sol
173.

Sol
167. कथास र ागर को
कहा नय 
के
महासागर
के
 प
म
भी
जाना
जाता
ह,
यह


भारतीय
 कवदं तय ,
प रय 
क 
कहा नय 
और
लोक
कथाओं
का
 स 
11व
शतपथ
 ा ण
वै दक
सा ह 
का
सबसे
 ाचीन
और
सबसे
बड़ा
 ा ण
ह। शता ी
का
सं ह
ह,
 जसे सोमदव ने
सं ृ त
म
 लखा
ह।
ा ण
वेद 
के
भजन 
क 
 ा ा
करते
ह। जब क
कथास र ागर
 यं
कई
कहा नय 
और
 कवदं तय 
का
संकलन
ह,
वे
मु तः
ग 
म
 लखे
गए
ह
और
इनक 
 कृ त
अनु ा नक
ह। ले कन
उदयन
और
उनके
बेट
क 
कहानी
पर
ब त
जोर
 दया
गया
ह।
शतपथ
 ा ण
यजुवद
से
संबं धत
ह
जब क
ऐतरय
और
सां 
 ा ण
ऋ ेद
से
संबं धत
ह। Sol
174.
गोपद
 ा ण
अथववेद
से
संबं धत
ह। सही
उ र
 वक 
3)
ह
अथात्
बृहदार क।
उप नषद
का
नाम मह पूण
 ट णी
Sol
168.
बृहदार क
( ाचीनतम या व -मै ेयी
संवाद
और
पुनज 
का
 वचार
इसी
उप नषद
गु 
युग
(320
ई.-
550
ई.)
को
कला,
सा ह ,
खगोल
 व ान,
 च क ा उप नषद) से
 लया
गया
ह।
व ान
आ द
के
संदभ
म
भारत
का
 ण
युग
माना
जाता
ह। चंडो यह
लौ कक
 
के
 स ांत
को
 सखाता
ह।
सु ुत
गु 
काल
म
 च क ा
पर
अपने
काय
के
 लए
जाने
जाते
ह।
सु ुत
के
बार
म: भारत
का
रा ीय
आदश
वा 
"स मेव
जयते",
इस
उप नषद
मुंडका
से
 लया
गया
ह।
सु ुत
गु 
काल
म
 च क ा
पर
अपने
काय
के
 लए
जाने
जाते
ह।
सु ुत पहले
भारतीय
श 
 च क क
थे।
Sol
175.
सु ुत
सं हता
श 
 च क ा
पर
एक
सं ृ त
 ंथ
ह।
सही
उ र
उद यन
ह।
Sol
169. शासक
का
नाम वंश मह पूण
त
सही
उ र
समु 
गु 
ह। 1.
वह
इस
राजवंश
के
सं ापक
थे।
2.
इसे
 े नका
के
नाम
से
भी
जाना
जाता
ह।
समु 
गु 
के
दरबारी
क व
और
मं ी
हरीसेन
ने
इलाहाबाद
 ंभ
 शलालेख
या 3.
वह
भगवान
बु 
के
समकालीन
थे।
याग
 स ी
क 
रचना
क 
थी। 4.
उसक 
तीन
प याँ
थ ;
यह
 ंभ
अशोक
 ारा
छह
शता य 
पूव
 न मत
एक
अशोकन
 ंभ
था।
यह
 शलालेख
समु 
गु 
का
 वन
ह
और
इसम
समु 
गु 
क 
 वजय
और कोशलदवी,
कोशल
के
राजा
क 
बेटी
और
ब सार
(544
-
गु 
सा ा 
क 
सीमाओं
का
उ ेख
ह। हयक सेन जत
क 
बहन।.
492
ईसा
पूव)
चेलाना,
 ल वी
शासक
चेतका
क 
बेटी
और
इस
 शलालेख
के
अनुसार,
समु 
गु 
ने
उ र
म
9
राजाओं
को
हराया,
द ण
अजातश 
ु क 
माँ।
म
12
राजाओं
को,
सभी
अटा वका
रा 
को
कम
कर
 दया।
पंजाब
के
मादरा
क 
बेटी
खेमा.
ह रशेन
 ारा
र चत
इलाहाबाद
 श 
क 
कोई
 त थ
नह 
ह
और
इसी
कारण
से,
इ तहासकार 
ने
माना
ह
 क
इसक 
रचना
संभवत:
समु गु 
 ारा
 कए
गए 5.
जब
अवंती
के
शासक,
 ोता
पी लया
से
पी ड़त
थे,
तब
अ मेध
य 
से
पहले
क 
गई
थी। उ ने
शाही
 च क क
जीवाका
को
भेजा
था।
उ ने
इस
आधार
पर
इसका
 वरोध
 कया
ह
 क
समु गु 
 ारा
पूण
 कए 1.
अपने
 पता
को
मार
डाला
और
 सहासन
को
हा सल
गए
अ मेध
य 
का
कोई
उ ेख
नह 
ह। कया।
इलाहाबाद
 स ी
को
मूल
 प
से
इलाहाबाद
के
पास
कौशांबी
म
अशोकन 2.
इसे
कु नका
और
अशोकचंदा
के
नाम
से
भी
जाना
जाता
ंभ
पर
उकेरा
गया
था।
बाद
म
इसे
इलाहाबाद
 कले
म
हटा
 दया
गया। था।
अजातश ु
(492
- 3.
उसने
अपने
मामा
 सनजीत
को
हराया
और
उसक 
बेटी
हयक
Sol
170. 460
ईसा
पूव) व जरा
से
शादी
क ।
4.
 स 
ह थयार
महा शलाकांतक
और
रथमुसाला
उनके
एरण: थे।
5.
उ ने
बु 
क 
मृ 
ु के
बाद
राजगृह
म
पहली
बौ 
प रषद
यह भारत
के
म 
 दश
के
सागर
 जले म
एक  ाचीन
शहर
और
पुराता क क 
 व ा
क ।
ल ह।
यह
भारतीय
राजवंश 
के
 लए
 ाचीन
टकसाल 
म
से
एक
था
जैसा
 क
यहां 1.
वह
अजातश ु
का
पु 
था।
उद यन
(460
-
हयक 2.
उसने
पाट लपु 
को
मगध
क 
राजधानी
बनाया
और
पटना
खुदाई
 कए
गए
 व भ 
 स 
से
 
ह। 444
ईसा
पूव)
म
गंगा
और
 पु 
के
संगम
पर
एक
 कला
भी
बनवाया।
इस
 ल
पर
5व 
और
6व 
शता ी
के
गु 
युग
के
मं दर
और
 ारक
ह,
वशेष
 प
से
मू तकला
के
शरीर
पर
 च त
ऋ षय 
और
 व ान 
के
साथ 1.
कालाशोक
को
काकवण
के
नाम
से
भी
जाना
जाता
था।

कालाशोक
(394
शशुनाग 2.
 ू सरी
बौ 
प रषद
वैशाली
म
उनके
शासनकाल
के
दौरान
वशाल
प र
के
सूअर
। -
366
ईसा
पूव)
आयो जत
क 
गई
थी।
एरण
म
पाए
जाने
वाले
 शलालेख काल म
और
गु 
सा ा 
के
इ तहास
के पुन नमाण
के
 लए
मह पूण
ह। Sol
176.
Sol
171.
वी.ए.
 थ हड़ ा
और
मोहनजो-दारो
क 
खुदाई
से
संबं धत
नह 
ह।
मोहनजो-दारोक 
खोज
1922
म आरडी
बनज  ने
क 
थी।
भराना को
सबसे
पुराना
खोजा
गया
 सधु
घाटी
स ता
 ल
माना
जाता
ह।
मोहनजो-दारो
के
पूव र
 े 
क  खुदाई के.एन.
दी त ने
क 
थी।
यह
भारतीय
पुरात 
सव ण
 ारा
 कए
गए
C-14
र डयो-ड टग
के
अनुसार
सबसे
पुराना
ह।
To Practice 3751 More Questions
From Remaining Chapters

Buy NOW

Available at
&

Unique Features of SmartBook


Time To Answer (TTA)

Smart Answer Key

Level - Wise Difficulty

Best 4000 Questions

Free Video Lessons


https://amzn.to/3HVqlZK ाचीन
इ तहास

एम.एस.
व  हड़ ा
म
1920-1921
और
1933-34
के
बीच
 कए
गए Sol
179.


पुराता क
उ नन
का
लेखा-जोखा
था।
महारा 
के
दमाबाद
म
हड़ ा
काल
के
रथ
क 
एक
मू त
पाई
गई
थी।
Sol
177. दमाबाद
एक
पुराता क
 ल
ह
 जसे
1958
म
खोजा
गया
था।
चार
 मुख
महान
स 
ह यहां
1976-79
के
बीच
खुदाई
क 
गई
थी।
लोथल
गुजरात
म
 त
एक
 ाचीन
 सधु
 ल
ह।
1.
 पीड़ा क 
स ाई ( ुख), बनवाली
एक
 ाचीन
हड़ ा
 ल
ह
जो
सर ती
क 
सूखी
नदी
के
तल
पर
2.
 पीड़ा क 
उ  का
सच (समुदाय), त
ह।
3.
 पीड़ा
के
 नमूलन का
सच ( नरोध), कालीबंगा
 जसका
अथ
काली
चू ड़याँ
ह,
राज ान
म
 त
एक
 ाचीन
4.
 पीड़ा क 
समा 
क 
राह का
सच (माग)। सधु
 ल
ह।
Sol
178.

Sol
180.
सही
उ 
तर
ह
 
 ं दगु 
।
ध -महाम ा,
ध  के
 व भ 
पहलुओ
ं को
लागू
करने
और
 चा रत
करने
के
ं दगु 
उ र
भारत
का
एक
गु 
स ाट
था। लए
 ा पत
अ धका रय 
का
एक
समूह
था।
ोत अशोक
ने
उ 
समाज
के
 व भ 
वग 
तक
अपना
संदश
प च ं ाने
के
भतरी
 ंभ
 शलालेख लए
 ज ेदार
बनाया।
उसने
अपने
 व ो हय 
या
 वदशी
आ मणका रय 
को
हराकर
गु 
श 
को बाद
म
वे
ब त
श शाली
हो
गए
और
रा 
क 
राजनी त
म
बहाल
 कया। ह ेप
करने
लगे।
उसने
 ण 
के
आ मण
को
ठकरा
 दया
।
Sol
181.
अतर 
जानकारी सही
उ र
सांप
ह। 
Key
Points
जूनागढ़
 शलालेख
यह
कहा
गया
ह
 क
 गु 
"पृ ी
के
शासक"
अपने
कौशल
से जैन
धम
क 
उ 
7व -5व 
शता ी
ईसा
पूव
पूव 
भारत
के
गंगा
जल
बन
गया। घाटी म
 ई
थी।
इससे
पता
चलता
ह
 क
 ं दगु 
ने
बल
 योग
करके
 सहासन 24 तीथकर
थे
 जनम
से
अं तम
वधमान
महावीर
थे।
ा 
 कया। जैन
धम
के
सं ापक
वधमान
महावीर (539-
467
ई.पू.)
थे। 
ं दगु 
ने
पांच
 कार
के
सोने
के
 स े 
जारी
 कए: पहले
तीथकर
ऋषभनाथ
थे
और
चौबीसव
तीथकर
महावीर
थे।
आचर
 कार जैन
धम
के
पाँच
वचन
ह:
राजा
और
रानी
 कार अ हसा
छ 
 कार स
शेर-ह ारा
 कार अचौय
या
अ ेय
घुड़सवार
 कार। चय
ं दगु 
के
चांदी
के
 स े 
चार
 कार
के
होते
ह: अप र ह 
ग ड़
 कार
बैल
 कार Sol
182.
वेदी
 कार सही
उ र
पृथक
क लग
 शला
 ापक
I
ह।
म दश
 कार
ं दगु 
ने
पुराने
दीनार
मानक
से
एक
नए
सुवण
म
 च
करते
 ए
अपनी अलग-अलग
 ापक:
वे
क लग
म
 ल 
पर
पाए
गए
थे।
मु ा
का
पुनमू ांकन
 कया। पृथक
 ापक
I:
अशोक
ने
घोषणा
क 
 क
सभी
लोग
मेर
पु 
ह।
पृथक
 ापक
II:
एक
 
को
भी
 ापक
क 
घोषणा।
मह पूण
 ब ु
गु 
वंश: Key
Points

गु 
वंश
के
सं ापक
 ी
गु 
थे। दो
 वशेष
 शला
 ापक
को
महामं 
के
 नदश
के
 प
म
लोग 
को
 न
वह
घटो च
 ारा
सफल
 आ
था।
इन
दोन 
को
महाराजा
कहा
जाता
था। और
सुलह
करने
के
 लए
अं कत
 कया
गया
था
और
उन
 स ांत 
को
अगला
शासक
चं गु 
 थम
था
और
वह
सबसे
पहले
महाराजा धराज ा पत
 कया
 जस
पर
अशोक
ने
क लग
और
इसके
सीमावत
कहलाया। जनजा तय 
के
 शासन
का
आधार
मांगा।
चं गु ,
म
लगभग
330
ई ी
म
समु गु 
 ारा
सफल
 आ,
 ज ने
लगभग पृथक
क लग
 ापक,
वा व
म,
क लग
के
लोग 
के
 लए
ह
और
कह 
और
पचास
वष 
तक
शासन
 कया। नह 
पाए
जाते
ह।
वह
एक
महान
सै 
 तभा
था
और
कहा
जाता
ह
 क
उसने
द न
म
एक पृथक
क लग
 ापक
म
अशोक
ने
 शासन
के
अपने
 स 
 स ांत
को
भी
सै 
अ भयान
क 
कमान
संभाली
थी,
और
 व 
 े 
क 
वन
जनजा तय 
को ीकार
 कया
था,
"सभी
नाग रक
मेर
ब 
े ह।
जैसा
 क
म
अपने
ब 
के
भी
अपने
अधीन
कर
 लया
था। लए
चाहता
 
ं क
वे
सभी
इस
 ु नया
म
आनंद
और
सुख
ल।"
समु गु 
के
उ रा धकारी
चं गु 
 तीय,
 जसे
 व मा द 
के
नाम
से
भी
जाना
जाता
ह,
ने
मालवा,
गुजरात
और
का ठयावाड़
के
 ापक
 े 
पर Sol
183.
वजय
 ा 
क ।
ादातर
चांदी
से
बने
पंच- च त
 स े ,
 बहार
म
ब तायत
से
पाए
जाते
मुख
 ब ु थे।
गु 
राजाओं
के
शासन
के
दौरान
कुछ
 स 
घटनाएँ : ये
पटना
शहर
के
गोलकपुर
म,
पू णया
और
अ 
 ान 
पर
पाए
गए
थे।
कुशान
 स े ,
ब र
म
खुदाई
म
पाए
गये
ह
जो
कुषाण
सा ा 
क 
सीमा
स 
चीनी
तीथया ी
फा ान
चं गु 
 तीय
के
शासनकाल
के
दौरान
भारत को
दशाता
ह।
आए
थे।
भारत
म
अपने
नौ
वष 
के
 वास
म
से,
उ ने
 बतायागु 
सा ा 
म
छह Sol
184.
साल। सही
उ र
जैन
दशन
ह।
चं गु 
 तीय
ने
प मी
भारत
के
शक
 प 
के
 खलाफ
यु 
छड़ा। Key
Points
शक
 प
के
अं तम
शासक
 सह
III
को
परा जत,
ग ी
से
उतारकर
मार ीकरण:
दया
गया।
प मी
मालवा
और
का ठयावाड़
 ाय ीप
म
उसके
 े 
को
गु 
सा ा 
म
 मला
 लया
गया
था। समा ध
मरण
या
स ेखना
 े ा
से
मौत
को
गले
लगाना
ह
जब
घरवाले
कुमारगु 
ने
नालंदा
 व व ालय
क 
न व
रखी। और
सं ासी
दोन 
यह
कहते
ह
 क
वृ ाव ा,
लाइलाज
बीमारी,
गंभीर
https://amzn.to/3HVqlZK ाचीन
इ तहास

अकाल
आ द
के
कारण
जीवन
का
अंत
ब त
 नकट
ह। यह
 ड
 णाली
पर
आधा रत
संग ठत
योजना
के
 लए
 स 
ह।
याद
रखने


यह
जैन
धम
के
आचार
सं हता
के
तहत
एक
पूरक
 त ा
ह। यो 
मह पूण
त ।
यह
शरीर
म
भोजन
के
सेवन
को
धीर-धीर
कम
करके
 ै क
 प
से
उपवास जॉन
माशल
' सधु
घाटी
स ता'
श 
का
इ ेमाल
करने
वाले
करने
क 
 था
ह। पहले
शोधकता
थे।
सधु
घाटी
स ता
र डयो-काबन
ड टग
के
अनुसार
2500
-
1750
Sol
185. ईसा
पूव
से
फैल
गई।
चंडो 
 ा ण
का
संबंध
साम
वेद
से
ह। हड़ ा
स ता
क 
सबसे
 व श 
 वशेषता
इसका
शहरीकरण
था।
इसके
अलावा,
भेड़
और
बक रयाँ,
कु ,े 
मवेशी
भस,
और
हाथी
 सधु
चंडो 
उप नषद
 ह ू 
धम
के
साम
वेद
के
चंडो 
 ा ण
म
स हत
एक घाटी
स ता
म
पालतू
थे।
सं ृ त
पाठ
ह। राजधानी
शहर
मोहनजोदड़ो
और
हड़ ा
ह।
यह
सबसे
पुराने
उप नषद 
म
से
एक
ह। बंदरगाह
शहर
सुतकागडोर,
बालाकोट,
लोथल,
अ ाहदीनो
और
यह
108
उप नषद 
के
मु का
कानून
म
सं ा
9
के
 प
म
सूचीब 
ह। कुंतसी
ह।
उप नषद
सामवेद
के
तां 
 व ालय
से
संबं धत
ह। सधु
घाटी
के
लोग
कपास
और
ऊन
दोन 
के
उपयोग
से
अ ी
तरह
उप नषद
का
नाम
चंडा
या
चांड
श 
से
 लया
गया
ह,
 जसका
अथ
ह प र चत
थे।
"का 
छं द,
छं द"। मुख
शहर 
के
नाम
नीचे
 दए
गए
ह:
मोहनजोदड़ो
( सध)
-
यह
 सधु
के
दा हने
 कनार
पर
 त
ह।
Sol
186.
कालीबंगन
(राज ान)
-यह
घ र
नदी
के
तट
पर
था।
सही
उ र
 वक 
2
ह,
अथात्
बौ 
धम। च ो
-
यह
मोहनजोदड़ो
के
द ण
म
 सधु
के
बाएं 
 कनार
पर
त
ह।
'कालच '
समारोह
बौ 
धम 
से
जुड़ा
ह।
लोथल
(गुजरात)
-
यह
कै 
े क 
खाड़ी
के
 मुख
पर
 त
ह।
'कालच '
सं ृ त
म
र चत
इस
परंपरा
के
मूलभूत
बौ 
तां क
 ंथ
का
नाम
ह। सुरकोटदा
(गुजरात)
-
यह
क 
के
रण
के
 मुख
म
ह।
बनवाली
(ह रयाणा)
-
यह
अब
 वलु 
सर ती
के
तट
पर
 त
बाद
म
इसका
 त ती
म
अनुवाद
 कया
गया।
था।
कालच 
परंपरा
के
मूल
सं ृ त
 ंथ
"11व 
शता ी
के
शु आती
दशक 
के
दौरान
उ 
 ए
थे।
धोलावीरा
(गुजरात)
-
इसक 
खुदाई
क 
 जले
म
होती
ह।
इस
परंपरा
के
सबसे
मह पूण
 ंथ 
म
कालच तं 
शा मल
ह। Sol
192.
इस
पर
 ा ा क
 ट णी
को
 वमल भा
कहा
जाता
ह।
सधु
घाटी
स ता
म
वा ुकला
क 
कुछ
मह पूण
 वशेषताएँ :
Sol
187.
शहरी
शहर-उ ेखनीय
शहर
क 
योजना,
और
जल
 नकासी
और
 ता
क 
एक
उ ृ 
 णाली।
चं गु 
 थम
महाराजा धराज
क 
उपा ध
को
अपनाने
वाला
गु 
वंश
का
बड़
शहर 
को
दो
भाग 
म
 वभा जत
 कया
गया
ह-प म
क 
ओर
 म ी- ट
पहला
शासक
था।
के
उ 
पो डयम
पर
बना
'गढ़'
टीला।
पूव
का
शहर
आवासीय
 े 
का
मु
चं गु 
 थम
गु  राजा
घटो च
का
पु 
था और
वंश
के
सं ापक
गु 
के
क 
था,
जो
एक
 वशाल
 ट
क 
दीवार
से
 घरा
 आ
था।
पौ 
थे,
दोन 
को
इलाहाबाद
 ंभ
 शलालेख
म
महाराजा
कहा जाता
ह।
सड़क- ल लत
जल
 नकासी
 णाली,
सु व त
जल
आपू त
 णाली,
द
वह अपने
अगले
उ रा धकारी
समु गु 
के
 पता
भी
थे।
ीट
लाइट् स
 णाली,
घड़ी
और
वाड
क 
 व ा,
रात
के
समय
म
लॉन
Sol
188. तोड़ने
वाल 
को
हटाने
के
 लए,
 वशेष
 प
से
 ैश
और
अप श 
पदाथ
फकने
के
 लए
 ान,
हर
गली
म
सावज नक
कुएँ ,
हर
घर
म
कुएँ ।
मु 
सड़क
9
जैन
गण
के
नेता
गणधर
के
 प
म
जाने
जाते
थे। फ ट
से
लेकर
30-34
फ ट
तक
चौड़ी
होती
ह
और
शहर 
को
 वभा जत
करने
जैन
धम
म,
गंधार
श 
का
उपयोग तीथकर
के
मु 
 श 
को संद भत
करने के
महान
कौशल
के
साथ
संक ण
ग लय 
के
नेटवक
म
 वभा जत
होती
ह।
के
 लए
 कया
जाता
ह। नमाण
साम ी-  सधु
शहर 
म
कोई
प र
से
 न मत
घर
और
बड़ी
इमारत
समवसरण
म
तीथकर
 बना
छए
 सहासन
पर
बैठ
गए। क 
सी ढ़याँ
ठोस
नह 
थ ;
छत
सपाट
थ 
और
लकड़ी
से
बनी
थ ।
तीथकर
के
चार 
ओर
गणधर
 वराजमान
ह। उपयोग
क 
गई
साम ी- जली
 ई
 ट
और
धूप
म
सूखने
वाली
 ट

Sol
189. Sol
193.
मनु ृ त
म
 दए
गए
 ववाह
के
कुल
आठ
 प
ह।
एनसीईआरटी क ा
12व 
म
इन
बहार के गया
 जले म
 त महाबो ध
 वहार रा 
से
एकमा  यूने ो
 व आठ
 ववाह
 प 
म
से
चार
का
उ ेख
ह।
य़े
ह
-
धरोहर
 ल ह।
यह बोधगया
म एक बौ 
मं दर
ह,
जहां
बु 
के
 ान
के
बार
म
कहा
जाता
ह पहला:
एक
बेटी
का
उपहार,
उसे
महंगे
कपड़
पहनाने
और
उसे
गहने
भट
करने
क
उ 
 ान
 ा 
 आ
था। के
बाद,
वेद
म
सीखे
 ए
 
को,
 जसे
 पता
 यं
आमं त
करता
ह,
को
स ा नत
करता
ह।
Sol
190. चौथा:
 पता
 ारा
बेटी
को
उपहार
दने
के
बाद
उसने
जोड़
को
पाठ
के
साथ
मेसो ल थक
आयु । संबो धत
 कया,
"आप
दोन 
अपने
कत 
को
एक
साथ
 नभा
सकते
ह",
और
ू 
े को
स ान
 दखाया
ह।
मेसो ल थक
युग
लगभग 9000
ईसा
पूव
से
4000
ईसा
पूव तक
 गरता
ह। पांचवां:
जब
 ू 
े को
अपनी
इ ा
के
अनुसार,
संबं धय  को
और
 ु न
को
यह
पुरापाषाण
युग
और
नवपाषाण
युग
के
बीच
का
सं मणकालीन
चरण खुद
के
 लए
 जतना
धन
 दया
जा
सकता
ह,
उसके
बाद
एक
युवती
 मलती
था। ह।
हालाँ क
मेसो ल थक
पु ष
अब
भी
 शकार
पर
काफ 
हद
तक
 नभर
थे, छठा: एक
युवती
और
उसके
 ेमी
का  ै क
संघ
जो
 े ा
से
हो।
ले कन
अब
वे
कु ,े 
भेड़,
बकरी,
गाय-बैल,
भस,
जंगली
घोड़ा,
आ द जानवर
का
 शकार
करना
शु 
कर
दते
थे। Sol
194.
म 
 दश
के आदमगढ़
और
राज ान
के
बागोर से
जानवर 
के
 शकार
का
सबसे
पहला
सबूत
 मला।
वे
मेसो ल थक
 ल
ह। वदह
सा ा 
क 
शु आत
 न म
 वदह
ने
क 
थी।
न म
 वदह
इ ाकु
पु 
थे।
Sol
191. यजुवद
म
इसका
उ ेख
 कया
गया
था।
सही
उ र
 वक 
3)
ह
अथात
धोलावीरा। दवी
सीता
इस
रा 
क 
राजकुमारी
थ ,
वह
 वदह
के
राजा
जनक
क 
बेटी
थ।
सधु
घाटी
स ता
वतमान
उ र-पूव
अफगा न ान
से
पा क ान
और
उ र- जनकपुर
इसक 
राजधानी
थी।
प म
भारत
तक
फैली
 ई
थी।
स ता
घ र-हकरा
नदी
और
 सधु
के
नदी-नाल 
म
पनपी।
 सधु
घाटी
स ता
 ु नया
क 
चार
सबसे
पुरानी Sol
195.
स ताओं
म
से
एक
ह।
इसे
हड़ ा
स ता
के
 प
म
भी
जाना
जाता
ह
और
https://amzn.to/3HVqlZK ाचीन
इ तहास

ऋ ेद
क 
रचना
 ारं भक
वै दक
युग
म
 ई
थी। यजुवद
और
अथववेद
म
कृ -आयस,
 ामा
और
 ाम-अयस
यह
1500
ईसा
पूव
-
1000
ईसा
पूव
के
बीच
था। श 
लोह
के
शु आती
नाम
माने
जाते
ह।
अत:
 वक 
3
सही
चार
वेद
ऋग,
यजुर,
साम
और
अथव
ह। ह।
अ 
तीन 
क 
रचना
उ र
वै दक
युग
म
 ई
थी।
उ र
वै दक
काल
1000
ईसा
पूव
-
600
ईसा
पूव
के
बीच
था। Sol
200.
ागै तहा सक

काल
उस
समय
को
संद भत
करता
ह
जब
कोई
लेखन
और सही
उ र
मथुरा
ह।
वकास
नह 
था
या
लेखन
का
कोई
सबूत
नह 
ह।
सुरसेन
रा 
उ र
 दश
म
वतमान
 ज
 े 
के
अनु प
एक
 ाचीन
भारतीय
Sol
196. े 
था।
वक 
1
सही
नह 
ह, अथात बाद
के
वै दक
काल
म
म हलाओं
को
मह पूण सुरसेन
क 
राजधानी
मथुरा
थी।
राजनी तक
 वशेषा धकार
 ा 
 ए। बौ 
 
अंगुटारा
 नकया,
सुरसेना
कहती
ह
 क
यह
सोलह
महाजनपद 
म
से
एक
था।
ारं भक
वै दक
काल
(ऋ े दक
काल)
के
 वपरीत,
बाद
म
वै दक
काल
म हलाओं
के
अ धकार 
के
 लए
उदार
नह 
था। Important
Points
म हलाओ
ं क 
 त
म
कोई
सुधार
नह 
 आ।
वे
अभी
भी
पु ष 
के
 लए सोलह
महाजनपद
इस
 कार
ह:
नीच
और
अधीन 
माने
जाते
थे।
म हलाओं
ने
 वधानसभाओं
म
भाग
लेन
े के
अपने
राजनी तक
अ धकार 
को
भी 1.
 कासी
खो
 दया।
इस लए,
 वक 
1
सही
नह 
ह। 2.
कोशल
बाद
म
वै दक
लोग
समु 
से
प र चत
थे
और
उ ने
बाबुल
जैस
े दश 
के
साथ 3.
अंगा
ापार
 कया। 4.
 मगध
वंशानुगत
 ापा रय 
(वा नया)
का
एक
वग
अ 
म
आया।
वै 
 ापार 5.
 व ी
और
वा ण 
पर
भी
काम
करते
थे।
उ ने
खुद
को
 ग 
के
तौर
पर
संग ठत 6.
 म ा
कया
ह
 जस
गनास
के
नाम
से
जाना
जाता
ह। 7.
 सले
ऋ े दक
काल
के
 न ा
के
अलावा,
सोना
और
चांदी
के
 स े 
जैसे 8.
 कु
सतामना
और
 शनाला
का
उपयोग
 व नमय
के
मी डया
के
 प
म
 कया 9.
 व
जाता
था। 10.
 पांचाल
बाल
 ववाह
आम
हो
चुके
थे। 11.
 म
ऐतरय
 ा ण
के
अनुसार,
एक
बेटी
को
 ुख
का
 ोत
बताया
गया
ह। 12.
 सुरसेन
13.
 अ ाका
Sol
197. 14.
अवंती
सही
उ र
सर ती
नदी
ह। 15.
गांधार
Key
Points 16.
 क ोज

सर ती
नदी
को
न द
तना
या
ऋ ेद
क 
सव े 
न दयाँ
भी
कहा
जाता
ह, Sol
201.


ह रयाणा
और
राज ान
म
घ र-हकरा
चैनल
से
पहचानी
जाती
ह। सही
उ र
 वक 
1
अथात्
लोथल
ह।
अफगा न ान
क 
कुछ
न दय 
कुंभ
और
 सधु
या
 सधु
नदी
और
इसक 
पांच
मु 
शाखाओं
का
उ ेख
ऋ ेद
म
 मलता
ह। सधु
घाटी
स ता:
सधु
नदी
ऋ ेद
म
सबसे
अ धक
बार
उ खत
नदी
क 
उ ृ ता
ह। सधु
शहर 
क 
योजना
एक
 ड
पैटन
पर
बनाई
गई
थी
जहाँ
सड़क
सदव
90
गंगा
नदी
का
ऋ ेद
सं हता
म
भी
कई
बार
उ ेख
 कया
गया
ह। ड ी
पर
रहती
थ ।
सबसे
उ ेखनीय
पहलू
क ीय
जल
 नकासी
 णाली
थी
जो
पूण
 प
से
Sol
198. ढक 
 ई
थी
और
बाथ म
के
साथ
जुड़ी
 ई
थी।
सधु
घाटी
के
 य 
क 
मह पूण
 वशेषता
यह
ह
 क
उ ने
 ट
क
ारं भक
वै दक
युग
के
दौरान
जा त- व ा
ने
अंतरजातीय
भोजन
और इमारत
जला
दी
थ ।
अंतरजातीय
 ववाह
पर
 तबंध
नह 
लगाया
था। डां सग
गल
क 
कां 
क 
मू त
को
 सधु
घाटी
स ता
का
मा र
पीस
कहा
1500
ई.पू.
और
1000
ई.पू.
के
बीच
क 
अव ध
को
 ारं भक
वै दक
काल
म जाता
ह।
वभा जत
 कया
जा
सकता
ह। सील
और
सी लग
आयताकार
और
गोलाकार
आकार
म
 ेटाईट
से
बने
थे।
1500
ई.पू.
और
600
ई.पू.
के
बीच
क 
अव ध
को बाद
के
वै दक
काल के
प
म
जाना
जाता
ह। Key
Points

Sol
199.
सही
उ र
यजुवद
ह।

भारत
म
लोह
क 
 ारं भक
घटना
 व भ 
कालानु मक
और
सां ृ तक


चरण 
के
साथ
कई
 ल 
से
 रपोट
क 
गई
ह।
उ र
भारत
म
मह पूण
 ल
बरली
 जले
म
अ ह ,
एटा
 जले
म
जखेड़ा
और
अतरंजीखेड़ा,
मेरठ
 जले
म
ह नापुर
और
 जला
कौशांबी
म
कौशांबी
ह।
यजुवद
और
अथववेद
म
कृ -अयस,
 ामा
और
 ाम-अयस
(काली
या
गहरी
धातु)
श 
 
 प
से
इस
धातु
का
उ ेख
करते
ह।

मुख
 ब ु

यजुवद:-
"यजुस"
का
अथ
ह
"य 
का
सू "
और
यजुवद
य 
क 
 ाथना
क 
पु क
ह।
यह
अनु ान-अपण
मं /मं 
का
संकलन
करता
ह।
इसके
दो
 कार
ह:-
1.
 कृ 
यजुवद
2.
शु 
यजुवद
https://amzn.to/3HVqlZK ाचीन
इ तहास

ान वशेषताएं महाभारत
को वेद
 ास ने
 लखा
था।

Sol
207.
दोहर
दफन
के
सा
बुराइय 
को
समा 
करने
के
 लए
वै दक
दवता
इं 
को
महान
यो ा के
 प
म
जाने
चावल
क 
भूसी
के
अवशेष
लोथल
जाते
थे।
घोड़
क 
टराकोटा
मू त
के
सा
जहाज़
बनाने
का
 ान
इं 
-
आकाश
का
 ामी,
व 
का
दवता, बा रश
और एक
महान
यो ा।
एक
जहाज
का
टराकोटा
मॉडल
इं 
का
वाहन
सफेद
हाथी
(ऐरावत)
ह और
इनका एक
ह थयार व  ह।

Sol
208.
कुंड
भू म
के
सा
लकड़ी
के
कुंड
के
सा नबाना
( नवाण): बु 
के
 श ण
का
अं तम
ल 
 नबाना
क 
 ा 
थी।
यह
कालीबंगा
सात
अ 
वे दय 
के
सा
एक
जगह
नह 
थी
ले कन
एक
अनुभव
था
और
इसी
जीवन
म
 ा 
 कया
जा
जुताई
क 
 ई
सतह
सकता
ह।
नवाण
का
अथ
ह
सभी
इ ाओं
को
 ू र
करना
और
क 
को
समा 
करना,
जो
अंततः
पुनज 
से
मु 
क 
ओर
ले
जाता
ह।
घोड़
क 
ह याँ
अंडाकार
क
सुरकोटड़ा Sol
209.
दफ़न
 कये
 ए
मटके
मो तयाँ
बनाने
वाली
 ुकान बनवाली


इस
 ल
पर
हड़ ा
स ता
के
सभी
तीन
चरण 
के
 माण
 मलते 
ह
-


ारं भक
काल,
हड़ न
काल
और
हड़ न 
के
बाद।
महान
 ानघर
बनवाली
को
 ुग
और
एक
 नचले
शहर
म
 वभा जत
 कया
गया
था।
पूरी
महान
अ 
भंडार
मोहनजोदड़ो
संरचना
को
 कलाब 
 कया
गया
था।
वधानसभा
हॉल
दाढ़ी
वाले
आदमी
क 
 ेटाईट
छ व एक
टराकोटा
हल
मॉडल
पाया
गया
था।
पशुप त
क 
मुहर बनवाली
क 
खुदाई
म
उ 
गुणव ा
वाली
जौ
 मली
ह।

Sol
210.
एक
अनूठी
जल
 बंधन
 णाली
ढोलावीरा सं हता म भगवान
के
 लए
मं ,
 ाथना
और
भजन का
सं ह
ह।
एकमा 
 ान
 जसे
3
भाग 
म
 वभा जत
 कया
गया
था
वेद 
के
भजन
भाग
को
सं हता
कहा
जाता
ह,
यह
भी
वेद 
का
 ह ा
ह।
इसक 
कुल
सं ा चार ह
- (1) ऋ ेद
सं हता, (2) सामवेद
Sol
202. सं हता (3) यजुवद
सं हता, (4) अथववेद
सं हता
सही
उ र
 वक 
4
ह।
Sol
211.
गाग ,
मै ेयी
और
लोपामु ा
वै दक
युग
क 
 मुख
म हला
दाश नक
थ ।
गाग ,
ऋ ष
वाचक ी
क 
बेटी
थी
और
वह
वै दक
समय
क 
एक
महान पां 
ने
मौय
सा ा 
के
पतन
के
बाद
द ण
 ावणकोर
के
कुछ
 ह 
के
व ान
थी। साथ
म ुर
और
 त ेवे ी
 जल 
पर
क ा
कर
 लया।
अग 
ऋ ष
क 
प ी
लोपामु ा
ने
ऋ ेद
के
दो
छं द 
क 
रचना
क । इसक  राजधानी
म ुर म
थी
जो वैगई
नदी के
तट
पर
 त
ह।
मै ेयी
ऋ ष
मै ी
क 
पु ी। पां ा
के
 ज
म
एक एकल
मछली
या
जुड़वां
मछली शा मल
थी।
ऋ ष
मै ी
 म थला
म
 वदह
सा ा 
म
रहते
थे। सबसे पहला
 ात
पां न
शासक
मु ुकुदमी था।
" म थला"
 बहार
म
मौजूद
ह
और
इसक 
मूल
भाषा
मै थली
ह। सबसे
महान
पां ा
राजा
न ुजे लयन थे।

Sol
203. Sol
212.
सही
उ र
 वक 
1
ह,
अथात
गाय।
उ ने  ाकृत
भाषा का संर ण  कया। इस लए,
 वक 
3
सही
नह 
ह।
गाय,
ऊंट,
घोड़
और
शेर
को
मुहर 
पर
नह 
 दखाया
गया
था। सातवाहन
वंश का
सं ापक  समुका था। इस लए,
 वक 
1
सही
ह।
यू नकॉन
(बैल)
सबसे
अ धक
जवान 
का
 त न ध 
करता
था। उसने कृ  का
 ान
 लया,
 जसने
प म
म
ना सक
तक
रा 
का
 व ार
वा व
म,
घोड़ा
एक
जानवर
था
जो
हड़ ा
स ता
के
 लए
अ ात
था। कया।
जानवर 
का
वच 
एक
मह पूण
घटना
थी
 जसे
स ता
म
दखा
जा
सकता
ह। Sol
213.
चोल
Sol
204.
पु षाथ
या
जीवन
के
चार
उ 
धम,
अथ,
काम
और
मो 
ह।
तीक
 च :
चीता

धम
का
अथ
कत 
या
नै तकता
ह,
अथ
समृ 
और
धन
ह,
काम
का
अथ
ह चेरा



आनंद
या
कामुक
संतु ,
मो ,
मो 
क 
खोज
ह।
तीक  च :
धनुष
और
बाण
पु षाथ
मानव
क 
पू त
का
खाका
ह।
पां ा

Sol
205.
बौ 
 
म मा
 नकाय म
बेगुसराय
का
अंगु ारपा
के
 प
म
उ ेख
ह। तीक  च :
मछली

म मा  नकाय एक
बौ 
धम ंथ
ह,
पाँच
 नकाय 
म
से
 ू सरा
या
सं ह,
सु Sol
214.


पटक
म,
जो
"तीन
टोकरी"
म
से
एक
ह
जो
थेरवादन
क 
रचना
करता
ह। सही
उ र
ह
 वक 
3,
i.e.
च गु 
II।

Sol
206.

भगवद
गीता,
महाभारत
के भी 
पव से
 नकाली गई
ह।


महाभारत
म
कुल 18
पव ह।
यह सं ृ त
भाषा म 700
 ोक
वाला
 ह ू 
 ंथ ह।
https://amzn.to/3HVqlZK ाचीन
इ तहास

गु 
राजाओं
का
नाम शलालेख
का
नाम ांड
के
 शखर
तक
जाती
ह
( स शला)
और
वहाँ
अन 
आनंद
म
 नवास


करती
ह।
इलाहाबाद
 श ,
एरन
प र जैन
धम
के
धा मक
 ंथ 
के
अनुसार,
सही
 व ास,
सही
 ान,
और
सही
समु गु ंभ,
नालंदा
कॉपर
 ेट,
गया
आचरण
एक
साथ
मु 
का
सीधा
रा ा
बनाते
ह
 जसका
अथ
ह
सभी
कम
कॉपर
 ेट
से
पूण
मु ।
च गु 
II महरौली
लौह
 ंभ क 
सू 
एक
जैन
 ंथ
ह,
 जसम
जैन
तीथकर ,
 वशेष
 प
से
पा नाथ
कुमारगु 
 थम
( शलालेख
क 
सबसे
बड़ी
सं ा
म वलसाड,
मनकुँवर,
मथुरा,
साँची, और
महावीर
क 
जीवनी
ह।
जारी)
वलसाड,
मनकुँवर,
मथुरा,
साँची,
उदय ग र
गुफा, उदय ग र
गुफा,
तुमाइन,
बगराम, क 
सू 
भ बा 
 ारा
 लखा
गया
था
जो
चं गु 
मौय
के
दरबार
म
थे।
तुमाइन,
बगराम,
आ द। आद जैन
धम
के
पहले
तीथकर
ऋषभदव
थे
और
उ 
ऋषभनाथ
और
आ दनाथ
के
नाम
से
भी
जाना
जाता
ह।
जूनागढ़
रॉक,
इं दौर
कॉपर
 ेट, भगवान
महावीर
जैन
धम
के
24
व
तीथकर
थे
और
उ 
जैन
धम
का
ं दगु
भतरी
 ंभ,
कहम,
सो पया
वा वक
सं ापक
माना
जाता
ह।
बु गु पहाड़पुर
कॉपर
 ेट पा नाथ
जैन
धम
के
23
व
तीथकर
थे।
बौ 
धम
के
तीन
र 
( र )
बु ,
ध 
और
संघ
ह।
Sol
215.
सही
उ र
 वक 
4
यानी
आ दनाथ
ह
। Sol
219.

जैन
क त
 
और
 वजय
 
दोन 
 च ौड़गढ़
(राज ान)
म
 त
ह। Sol
220.


जैन
क त
 ंभ
क 
ऊंचाई
22
मीटर
ह
और
इसम
7
मं जल
ह। सही
उ र
 वक 
3
ह
अथात्
हीनयान
सं दाय
म
बो धस 
क 
कोई
अवधारणा
नह 
ह।
इसका
 नमाण
12व 
शता ी
म
भगेरवाल
जैन
 ापारी
जीजाजी
काठोड़
ने
करवाया
था। हीनयान
बु 
के
मूल
 श ण
का
अनुसरण
करता
ह।
यह
एक
 ढ़वादी
Rishabhanatha
जैन
धम
के
 थम
तीथकर
था। व ालय
से
अ धक
ह।
इसका
अथ
ह
कम
वाहन।
अतः
 वक 
1
सही
नह 
ह।
म
मु 
 ब ु 
तीथकर:
वे
बु 
क 
मू त
या
छ व
पूजा
म
 व ास
नह 
करते
थे।
अतः
 वक 
2
तीथकर
' श ण
भगवान'
या
जैन
धम
म
'फोड
 नमाता'
के
 प
म
जाना
जाता सही
नह 
ह।
ह। हीनयान
का
अं तम
उ 
 नवाण
(मो )
ह।
वे
 गत
उ ार
म
 व ास
जैन
धम
म,
यह
माना
जाता
ह
 क
 ेक
 ांडीय
युग
म
24
तीथकर
उ करते
ह
और
आ 
अनुशासन
और
 ान
के
मा म
से
 गत
मो 
 ा
होते
ह। करने
का
 यास
करते
ह।
कला
म
तीथकर 
को
कायो ग
मु ा
(शरीर
को
खा रज
करते
 ए)
म
 दखाया महायान
सं दाय
बो धस 
क 
पूजा
करता
ह
और
महायान
सू 
का
अ यन
गया
ह। करता
ह
जब क
हीनयानवादी
इनका
 दशन
नह 
करते
ह।
अतः
 वक 
3
24
तीथकर 
को
 तीका क
रंग 
या
 तीक 
 ारा
एक
 ू सर
से
अलग
 कया सही
ह।
जाता
ह। हीनयान
 व ान 
ने
जनसाधारण
के
साथ
बातचीत
करने
के
 लए
पाली
भाषा
का
उपयोग
 कया
था।
अतः
 वक 
4
सही
नह 
ह।
अतर 
जानकारी
Sol
221.
ऋषभनाथ
 थम
जैन
तीथकर
थे। सही
उ र
 वक 
2
ह,
अथात
चं गु 
मौय।
अ जतनाथ
 ू सर
जैन
तीथकर
थे।
सुम तनाथ
पांचव
जैन
तीथकर
थे। "ए पटोम"
के
यूनानी
लेखक
ज न
ने
चं गु 
मौय
का
स ोकोट् स
के
अ भनंदननाथ
चौथे
जैन
तीथकर
थे। प
म
उ ेख
 कया
था।
पा नाथ
23
व
जैन
तीथकर
थे। इसक 
पहचान
 व लयम
जो 
ने
क 
थी।
महावीर
24
व
जैन
तीथकर
थे। ज न
ने
उ ेख
 कया
 क
चं गु 
मौय
ने
 सकंदर
के
दरबार
का
दौरा
 कया
था।
Sol
216. उनके
कई
 शलालेख 
म
अशोक
का
उ ेख
 पयादासी
के
 प
म
ह।
भारतीय-यूनानी
सा ा ,
या
यूनानी-भारतीय
सा ा ,
 जसे
ऐ तहा सक
 प
से
यवन ब ुसार
को
"अ म ो
का तस"
के
 प
म
यूना नय 
के
 लए
जाना
जाता
था।
सा ा 
(यवनराज)
के
 प
म
जाना
जाता
ह,
अफगा न ान
के
 व भ 
 ह 
और कुणाल
अशोक
का
पु 
था।
भारतीय
उपमहा ीप
के
उ र-प मी
 े 
(आधु नक
पा क ान
और
उ र-प मी
भारत
के
कुछ
 ह )
को
पूण करने
वाला
एक
यूनानी रा 
था।
जो
 पछली
दो Sol
222.
शता य 
ईसा
पूव
के
दौरान
अ 
म
था
और
30
से
अ धक
राजाओं
 ारा
शा सत सही
उ र
 वक 
4
ह,
अथात
इलंगो
आ दगल।
था,
मेनडर,इनम सबसे
शानदार
और
सफल
थे।
सल ा दकारम
त मल
म
एक
सा ह क
काम
ह,
यह
त मल
लोग 
 ारा
Sol
217. उ 
संबंध
म
आयो जत
 कया
जाता
ह।
यह
इलंगो
आ दगल
 ारा
 लखा
गया
ह,
वह
एक
राजकुमार
था।
खजुराहो
 त कंद रया
महादव
मं दर को धंगदव ने
बनवाया
था। महाका 
म
हम
क गी
के
बार
म
पता
चलता
ह,
 जसने
अपने
प त
को
कंद रया
महादव
मं दर मूल
 प
से
एक  शव
मं दर ह।
 जसक   नमाण
अव ध पां डयन
राजवंश
के
दरबार
म
 ाय
क 
 न लता
के
 लए
खो
 दया,
उसने
999
ई. ह। ोध
म
अपने
रा 
का
बदला
 लया।
मं दर
अपनी नागर
शैली के
 ाप 
और अपनी
कामुक
मू तय  के
 लए
स 
ह। Sol
223.
खजुराहो
समूह
का
 ारक,  ह ू 
मं दर 
और
जैन
मं दर 
का
एक
समूह
ह
जो
म 
 दश
के
छतरपुर
 जले
म
 त
ह। कामंदका
का
नी तसार
कौ ट 
के
अथशा 
पर
आधा रत
ह।
यह
पु क
गु 
काल
के
दौरान
 लखी
गई
थी।
Sol
218. इसम
भारत
म
 ाचीन
काल
के
राजनै तक
जीवन,
कूटनी त
और
राजक य
सही
उ र
 वक 
4
अथात
सही
 व ास,
सही
 ान

और
सही
आचरण
ह। च ण
को
दशाया
गया
ह।

इन
 -र 
को
 ीफो 
 र ूज
या
र य
भी
कहा
जाता
ह
जो
मूल
 प Sol
224.


से
स क
दशन
(सही
 व ास),
स क
 ान
(सही
 ान)
और
स क अचौय
या
अ ेय
(गैर-चोरी
करना):
चोरी न
करना
जो
अपने
आप
से
संबं धत
नह 
ह
च र 
(सही
आचरण)
ह। उसे अचौयानु त कहा
जाता
ह।
एक
अपनेपन
के
 लए
सही
ह
और
गलती
से
या
जैन
दशन
के
अनुसार,
 ीफो 
 े 
या
 -र 
आ ा
क 
शु 
और जानबूझकर
कुछ
भी
नह 
लेता
ह।
मु 
को
 ा 
करने
के
तरीके
ह
 क
केवल
मु 
शु 
आ ा
( स ) जैन
सा ह 
उस
आगम
म
समा हत
ह
 जसम
कई
जैन
 ंथ
अध-मागधी
 ाकृत
भाषा
म
ह।
https://amzn.to/3HVqlZK ाचीन
इ तहास

Sol
225. Sol
232.
सही
उ र
 वक 
2
अथात्
 भाकर
वधन
ह। सही
उ र
 वक 
3
अथात
टोल 
के
सं ाहक
ह
मौय
नगर
 शासन
म
शू ा ा
श 
टोल 
का
सं ाहक
ह।
भाकर
वधन
ने
गुजर 
को
हराया
और
पड़ोसी
रा 
पर
अपना
 नयं ण पा ा ा वा ण 
अधी क
बढ़ाया।
वह
पु भू त
प रवार
के
थे। सम ा ा बाजार
के
अधी क
लगभग
606
शता ी
म
हषवधन
को
 सहासन
 ा 
 आ
और
उसने
647 पौ ा ा वजन
और
माप
के
अधी क
शता ी
तक
शासन
 कया।
नवा ा जहाज 
के
अधी क
Sol
226. शू ा ा टोल 
के
सं ाहक

रामे रम म कृ े रा
मं दर
रा कूट
वंश के कृ 
तृतीय  ारा
बनाया
गया अकारा ा खान
अधी क


था।
कृ 
तृतीय ने 936
से
968
ई ी तक
शासन
 कया
और
अपने
अ भयान 
के Sol
233.
लए
 स 
था।
उ ने त ोलम
म
चोल  को
हराया।
ह ू 
धम
म,
लोकपाल
आठ,
नौ
और
दस
 मुख
 दशाओं
से
जुड़
 दशाओं
के
Sol
227. संर क
को
संद भत
करता
ह।
न ा
मौय 
के
 ण
का
 स ा
था। बौ 
धम
म,
लोकपाल
चार
 ग य
राजाओं
और
अ 
र क
आ ाओं
को
संद भत
करता
ह,
जब क
 दशाओं
के
संर क
को
' दक़पाल'
कहा
जाता
ह।
मोय 
क 
आ धका रक
मु ा
पान
थी। चार
 दशाओं
के
र क
पान
चांदी
से
बना
होता
था
और
वह
¾
तोला
के
बराबर
होता
था।
कौ ट 
 ारा
 ल खत
अथशा ा
म
मौय
काल
के
दौरान
 स 
के
खनन
का 1.
कुबेर
(उ र)
उ ेख
ह। 2.
यम
(द ण)
3.
इं 
(पूव)
गु 
वंश
को
भारतीय
इ तहास
म
 ण
युग
माना
जाता
ह।
4.
व ण
(प म)
गु 
काल
के
चांदी
के
 स 
को
 पका
के
नाम
से
जाना
जाता
ह।
कुषाण
राजवंश
दश
के
 स े 
म
एक
 ां त
लेकर
आया
था। Sol
234.
हष
के
चांदी
के
 स 
को
 चम
के
नाम
से
जाना
जाता
ह।
Sol
235.
Sol
228. →
सही
उ 
तर
ह
 
पाट लपु 
।

तीथकर
के
 वचन
पर
आधा रत
जैन 
का
अंगगामा-पाठ पहली
जैन
सभा
300
ईसा
पूव
म
पाट लपु 
(वतमान
पटना)
म
 ई
थी।


12
ऐसे
पाठ:
मुख
 ब ु
अचरंगा
सू सु तंग ंगांग
सू Sol
229.
म लदा
प ा: यह
बैठक
चं गु 
मौय
के
शासनकाल
के
दौरान
 ई
थी
।
समवयंग
सू ा ा त ानधमकथा यह
बैठक
एस
थूलभ 
क 
अ ता
म
 ई।
Upasakadasah अ ा शः अनु ारौपप तकदासाह यह
बौ जैन
धम
के
शरीर
के
इस
 ह े
म
12
भाग 
का
संपादन
 कया
गया
था।
भ ु नागसेना और पहले
जैन
 वधानसभा
म,
जैन
धम
को
दो
भाग 
 दगंबर
और
 ेता र
म
नवकरणी वपाकसृता ववाड़ा
(अब
 वलु ) राजा  म लदा के
बीच बांटा
गया
था।
संवाद 
क 
रचना
ह।
म लदा  प ा
"  म लदा  के
 "
 को
 संद भत
 करता
 ह।  इसे
 पाली
 भाषा
 म तीथकर:
लखा
गया
ह।
तीथकर
' श ण
भगवान'
या
जैन
धम
म
'फोड
 नमाता'
के
 प
म
जाना
जाता
अ भधममो : ह।
जैन
धम
म,
यह
माना
जाता
ह
 क
 ेक
 ांडीय
युग
म
24
तीथकर
उ
यह वसुब ु  ारा
सं ृ त
भाषा
म
 लखा
गया
ह। होते
ह।
इसम अ भधम
को लेकर
चचा
होती
ह। कला
म
तीथकर 
को
कायो ग
मु ा
(शरीर
को
खा रज
करते
 ए)
म
 दखाया
गया
ह।
महाव ा: 24
तीथकर 
को
 तीका क
रंग 
या
 तीक 
 ारा
एक
 ू सर
से
अलग
 कया
जाता
ह।
यह
पाली
भाषा
म
 लखी
गई
एक
क वता
ह। इसम बौ 
धम के  सार
क  जानकारी
ह। अतर 
जानकारी
यह
राजा  वजया के
शासनकाल
के
दौरान
 लखा
गया
था।
ऋषभनाथ
 थम
जैन
तीथकर
थे।
Sol
230. अ जतनाथ
 ू सर
जैन
तीथकर
थे।
सुम तनाथ
पांचव
जैन
तीथकर
थे।
अ नी
 चनाब
का
ऋ ै दक
कालीन
ह। अ भनंदननाथ
चौथे
जैन
तीथकर
थे।
परंपरागत
 प
से
इसे
चं भागा
नदी
के
नाम
से
भी
जाना
जाता
ह। पा नाथ
23
व
जैन
तीथकर
थे।
चेनाब
नदी
का
उ म
बारालाचा
ला
दरा
ह। महावीर
24
व
जैन
तीथकर
थे।
वत ता
,
झेलम
का
ऋ ै दक
कालीन
नाम
ह।
प ष ण
रावी
का
ऋ ै दक
कालीन
नाम
ह। Sol
236.
वपश
 ास
का
ऋ ै दक
कालीन
नाम
ह।
बु 
के
मृ ु
के
बाद,
उनक 
 श ाओं
को
उनके
 श 
 ारा
"बुजुग "
या
Sol
231. व र 
 भ ुओ
ं क 
एक
प रषद
(वतमान
 बहार
म
वैशाली
के
 लए
पाली)
म
संक लत
 कया
गया
था।
कुतुब
मीनार
के
पास
 त
महरौली
लौह
 ंभ
चं गु 
 व मा द 
के
काल इन
संकलन 
को
 ट पटका
के
नाम
से
जाना
जाता
था
-
शा क
 प
से,
का
ह। व भ 
 कार
के
 ंथ 
को
रखने
के
 लए
तीन
टोकरी।
द ी
का
लौह
 ंभ
(महरौली
लौह
 ंभ)
7.2
मीटर
ह
जो
23
फ ट
और
8 वनया
 पटक
म
सांगा
या
मठ
के
आदश
म
शा मल
होने
वाल 
के
इं च
के
बराबर
ह। लए
 नयम
और
कानून
शा मल
थे ।
इसे क त
  के
नाम
से
भी
जाना
जाता
ह। सु 
 पटक: बु 
क 
 श ाएँ ।
https://amzn.to/3HVqlZK ाचीन
इ तहास

अ भध 
 पटक ने
दाश नक
मामल 
को
 नपटाया। Sol
241.


इस
 कार
 
ह
 क  वनय
 पटक
बौ 
संघ
के
 नयम 
से
संबं धत
 ंथ
ह। सही
उ र
 वक 
3
ह
अथात्
का तकेय।

Sol
237. का लदास
का
कुमारस वम्
क 
कहानी
कुमार
(का तकेय)
के
ज 
के


इसका
सही
उ र
एक
 वदश
मं ी
ह। आसपास
व णत
थी।
कुमार
(का तकेय)
भगवान
 शव
और
पावती
के
पु 
ह।
सं ध व ह
नामक
एक
उ ा धकारी
का
उ ेख
गु 
 शलालेख 
म
 कया गणेश
या
 वनायक
भगवान
 शव
और
पावती
के
एक
और
पु 
ह।
गया
था,
जो
संभवत:
 वदश
मामल 
के
मं ी
थे। का लदास
एक
शा ीय
लेखक
और
भारत
क 
सं ृ त
भाषा
के
सबसे
बड़
क व
थे।
Additional
Information उ 
'भारतीय
शे पयर'
के
नाम
से
जाना
जाता
ह।
का लदास
राजा
 व मा द 
के
दरबारी
क व
थे।
राजा
उसके
 शासन
म
एक
सेनाप त,
एक
सेनाप त
या
सेना
के
कमांडर-इन-
का लदास
क 
उ ेखनीय
रचनाएँ :
चीफऔर
अ 
मह पूण
अ धका रय 
का
सेनाप त
होता
था
अ भ ानशाकु लम्।
राजा
ने
 ांतीय
 शासन
के
साथ
कुमारमा 
और
अयु 
नामक
रघुवंशम्
(रघु
वंश
के
राजाओं
के
बार
म
एक
महाका
अ धका रय 
के
एक
वग
के
साथ
 नकट
संपक
बनाए
रखा।
क वता)।
गु 
सा ा 
म
 ांत 
को
भु य 
और
 ांतीय
रा पाल 
के
 प
म
मेघ ू तम्।
उप रकस
के
 प
म
जाना
जाता
था
।
व मोवशीयम्।
वे
 ादातर
राजकुमार 
म
से
चुन
े गए
थे।
कुमारस वम्।
Sol
238. अ भम 
ु ाचीन
भारतीय
महाका 
महाभारत
के
एक
पा 
ह।
ु 
भगवान
कृ 
और
 णी
के
बड़
पु 
ह।
पूव 
 ाचीपथ
क 
राजधानी क लग
के
समीप तोशाली म थी। सन ु मार
 ाचीन
भारतीय
महाका 
महाभारत
के
एक
पा 
ह।
अशोक
ने
केवल
एक
बड़ा
यु 
लड़ा
 जसे क लग
यु  कहा
जाता
ह।
Sol
242.
तेरहव
 मुख
 शलालेख के
अनुसार,
इस
 म
म
100,000
लोग
मार
गए
थे,
150,000
लोग 
को
बंदी
बनाया
गया
था। जैन
धम
म
 श ा
का
अं तम
ल 
'मु '
ह
 क
उनक 
 श ाओं
पर
 ाथ मक
जोर
दया
गया
था:
इस
भयावह
घटना
ने
अशोक
को
गहराई
से
 भा वत
 कया
और
उसका
 दय
प रवतन
 कया। ज 
और
मृ ु
क 
 ृंखला
से
मो 
या
मो 
क 
 ा ।
उसने
कभी
यु 
नह 
लड़ने
का
 ण
 लया। यह
 सखाना
 क
मो 
उनके
 ारा
 ा 
 कया
जा
सकता
ह
जो
इसे
 ा
करना
चाहते
ह ।
Sol
239.
पुनज 
और
शारी रक
अ 
से
कुल
मु 
 ा 
करने
के
 व भ 
तरीके।
सही
उ र
 वक 
2
ह,
अथात्
मह वमन
-
मत वलासा
 शसन।
सभी
शारी रक
सुख-सु वधाओं
के
कुल
 ाग
 ारा
 तं ता
क 
एक
आदश
मत वलासा
 शसन
-
त
को
 ा 
करना।
ब मुखी
प व
शासक
और
महान
 व ान
मह वमन
 थम
ने
 स धीर-धीर
मु 
के
 मक
 वकास
ने
सभी
भौ तक
बंधन 
को
काट
 दया।
नौकरशाह
मत वलासा
 शसन
को
 लखा।
इस लए
 वक 
2
सही
ह।
Sol
243.
मटा वलसा
 हसन
का
अथ
ह
 ोट
ऑफ़
 ंकड् स।
यह
एक
लघु
एकांक 
सं ृ त
नाटक
ह। सही
उ र
ह
पाट लपु ।
मत वलास
 श 
एक
 ं 
ह
जो
बौ 
धम
के
 वधम 
कपा लका
सं दाय
क 
राजवंश
क 
राजधानी
पाट लपु 
थी।
और
जैन
धम
के
पासुपाता
सै वत
सं दाय 
के
अजीब
पहलुओ
ं पर
मज़ाक
वासुदव
क 
क 
वंश
के
सं ापक
थे।
उड़ाता
ह।
क 
वंश
 ा ण
वंश
था।
Sol
240.
वासुदव
का
उ रा धकार
उनके
पु 
भू म 
ने
 कया
था।
सुशमान
क 
वंश
के
अं तम
राजा
थे।
सही
उ र
हषवधन
ह।
सातवाहन
 ारा
क 
वंश
को
हा सल
 कया
गया
था।
Important
Points
ज़ुआन
ज़ांग
(
 आन
 ांग):
Sol
244.
एक
चीनी
बौ 
 भ ,ु 
 व ान,
या ी
और
अनुवादक
 ज ने
सातव सही
जवाब
भगभा ा
ह।
शता ी
म
भारत
क 
या ा
क । Key
Points
उ ने
 ारं भक
तांग
राजवंश
के
दौरान
चीनी
बौ 
धम
और
भारतीय
बौ
भागभ 
ने
उ र,
म 
और
पूव 
भारत
म
110
ईसा
पूव
के
आसपास
शासन
धम
के
बीच
बातचीत
का
वणन
 कया।
कया।
न
 ांग
क 
भारत
या ा
का
 ाथ मक
उ 
बौ 
धम
का
 ान
 ा
उ ने
म 
 दश
के
बेसनगर
म
अपना
दरबार
 ा पत
 कया।
करना
और
उसके
धा मक
 ंथ 
को
एक 
करना
था।
उ 
म 
 दश
के
 व दशा
म
ह लयोडोरस
 ंभ
के
एक
 शलालेख
से
चूँ क
उ 
भारत
आने
के
 लए
चीनी
स ाट
क 
अनुम त
नह 
 मली,
इस लए
जाना
जाता
ह।
वे
629
ई ी
म
वहाँ
से
 खसक
गए
वे
लगभग
पाँच
वष 
तक
नालंदा
 व व ालय
म
रह। Additional
Information
उ ने
 याग
म
हष
 ारा
बुलाई
गई
एक
धा मक
सभा
म
भी
भाग
 लया।
वह
भारत
म
अपनी
स ह
साल
क 
भू म
या ा
के
 लए
 स 
 ए। शुंग
वंश
ने
लगभग
185
से
75
ईसा
पूव
तक
म 
और
पूव 
भारतीय
उ ने
हषवधन
के
शासनकाल
के
दौरान
भारत
का
दौरा
 कया,
जो
बौ उपमहा ीप
पर
शासन
 कया।
धम
के
 त
उनक 
गहरी
भ 
के
 लए
उनक 
 शंसा
करने
आए
थे।
अत:, इसे
पु म 
शुंग
ने
मौय
सा ा 
के
अं तम
शासक
बृह थ
मौय
को
मारने
वक 
3
सही
ह। के
बाद
 ा पत
 कया
था।
इसके
राजधानी
शहर
पाट लपु 
और
 व दशा
थे।
Additional
Information
भारत
के
अ 
 वदशी
या ी: Sol
245.
या ी का
शासन: वै दक
स ता सर ती
नदी
के
 कनार
 वक सत
 ई।
डीइमाचोस ब ुसार
वै दक
स ता
-
मेग नीज चं गु 
मौय वै दक
काल 1500
ईसा
पूव
और
500
ईसा
पूव माना
जाता
ह।
फा ान चं गु 
 तीय इस
अव ध
को  ारं भक
वै दक
काल
(1500
ईसा
पूव
-
1100
ईसा
पूव) और बाद
म
वै दक
काल
(1100
ईसा
पूव
-
500
ईसा
पूव) म
इ 
बतूता मुह द- बन-तुगलक वभा जत
 कया
गया
ह।
https://amzn.to/3HVqlZK ाचीन
इ तहास

यह
अव ध  सधु-घाटी
स ता
के
अंत
के
बाद फलने-फूलने
वाली कला
म
तीथकर 
को
कायो ग
मु ा
(शरीर
को
खा रज
करते
 ए)


मानी
जाती ह। म
 दखाया
गया
ह।
24
तीथकर 
को
 तीका क
रंग 
या
 तीक 
 ारा
एक
 ू सर
से
Sol
246. अलग
 कया
जाता
ह।
सही
उ र
फा ान
ह।
Key
Points Sol
248.

फा ान
(405-411
ई ी):
वह
एक
चीनी
बौ 
 भ ु
थे
जो
च गु 
 तीय मेग नीज चं गु 
मौय के
दरबार
म से ूकस
 नकेटर का
राज ू त
था।
के
शासनकाल
म
भारत
आए
थे। वह
भारत
म
304
ईसा
पूव
से
299
ईसा
पूव
तक
रह।
इ ग
(671–695
ई ी):
एक
चीनी
या ी
थे,
वह
बौ 
धम
के
संबंध
म मेगैथेन
के
संदभ
मौय
काल
क 
सामा जक
और
राजनी तक
 त
पर
 काश
भारत
आये
थे
और
उ ने
हम
पहले
गु 
शासक
के
बार
म
जानकारी
दी
थी। डालते
ह।
हषवधन
(606
ई ी
से
647
ई ी): उनके
अनुसार,
भारतीय
समाज 7
जा तय  म
 वभा जत
था,
जो
दाश नक,
वह
पु भू त
वंश
का
सबसे
 मुख
शासक
था। कसान,
चरवाहा,
 ापारी,
यो ा,
पयवे क
और
पाषद
थे।
उसने
अपनी
राजधानी
क ौज
से
शासन
 कया। गुलामी
अनुप त
थी।
हष
के
तहत
पु भू त
सा ा 
उ र
और
उ र-प मी
भारत
तक
व ृत
 आ। Sol
249.
इस
सा ा 
का
 व ार
पूव
म
काम प
तक
और
द ण
म
नमदा
नदी
तक
था। हीनयान
मू त
पूजा
म
 व ास
नह 
करता
ह और
आ -अनुशासन
और
 ान
हषच रत
म
बाणभ 
 ारा
उनके
शासनकाल
को
अ ी
तरह
से के
मा म
से
 गत
मो 
 ा 
करने
क 
को शश
करते
ह।
ले खत
 कया
गया
ह। इस
 कार
हीनयान
का
अं तम
उ   नवाण ह।
चीनी
या ी
 न
 ांग
ने
इनके
काल
म
भारत
का
दौरा
 कया। महायान
या
"महान
वाहन"
बु 
के
बु 
और बु 
क 
मू त
क 
पूजा
और
बो धस  को
बु 
 कृ त
का
 तीक
मानते
ह।
Sol
247.

संभवनाथ तीसर
जैन
तीथकर
थे।
तीथकर:
तीथकर
' श ण
भगवान'
या
जैन
धम
म
'फोड
 नमाता'
के
 प
म
जाना
जाता
ह।
जैन
धम
म,
यह
माना
जाता
ह
 क
 ेक
 ांडीय
युग
24
तीथकर
पैदा
करता
ह।
To Practice 3751 More Questions
From Remaining Chapters

Buy NOW

Available at
&

Unique Features of SmartBook


Time To Answer (TTA)

Smart Answer Key

Level - Wise Difficulty

Best 4000 Questions

Free Video Lessons

You might also like